Добавил:
Upload Опубликованный материал нарушает ваши авторские права? Сообщите нам.
Вуз: Предмет: Файл:
крок 1_ЗЛП_2009.DOC
Скачиваний:
225
Добавлен:
01.05.2019
Размер:
10.29 Mб
Скачать

Правильнівідповіді на тест “Крок 1 Лікувальна справа” 1998р.

1.

E

21.

E

41.

B

61.

A

81.

B

101.

C

121.

C

141.

E

161.

E

181.

B

2.

D

22.

D

42.

E

62.

D

82.

C

102.

B

122.

E

142.

B

162.

C

182.

E

3.

C

23.

C

43.

D

63.

A

83.

C

103.

C

123.

A

143.

B

163.

A

183.

E

4.

E

24.

D

44.

C

64.

A

84.

A

104.

B

124.

E

144.

C

164.

A

184.

D

5.

E

25.

C

45.

C

65.

C

85.

D

105.

C

125.

E

145.

C

165.

E

185.

A

6.

C

26.

D

46.

D

66.

D

86.

C

106.

C

126.

C

146.

D

166.

E

186.

D

7.

D

27.

A

47.

E

67.

E

87.

C

107.

C

127.

E

147.

B

167.

A

187.

E

8.

D

28.

C

48.

E

68.

C

88.

A

108.

E

128.

B

148.

A

168.

C

188.

D

9.

E

29.

B

49.

A

69.

C

89.

B

109.

E

129.

B

149.

C

169.

C

189.

B

10.

B

30.

E

50.

B

70.

A

90.

C

110.

D

130.

D

150.

E

170.

C

190.

A

11.

B

31.

C

51.

B

71.

D

91.

C

111.

B

131.

B

151.

B

171.

D

191.

E

12.

C

32.

B

52.

A

72.

C

92.

A

112.

E

132.

A

152.

D

172.

D

192.

E

13.

B

33.

A

53.

D

73.

C

93.

D

113.

E

133.

D

153.

E

173.

C

193.

D

14.

B

34.

C

54.

B

74.

D

94.

A

114.

A

134.

C

154.

B

174.

B

194.

B

15.

D

35.

E

55.

C

75.

A

95.

D

115.

B

135.

E

155.

A

175.

E

195.

C

16.

E

36.

D

56.

B

76.

B

96.

A

116.

B

136.

E

156.

C

176.

D

196.

D

17.

C

37.

A

57.

E

77.

B

97.

A

117.

C

137.

B

157.

B

177.

E

197.

E

18.

B

38.

C

58.

A

78.

B

98.

E

118.

B

138.

C

158.

A

178.

A

198.

A

19.

B

39.

C

59.

D

79.

B

99.

A

119.

C

139.

A

159.

D

179.

D

199.

D

20.

C

40.

E

60.

E

80.

B

100.

E

120.

E

140.

B

160

D

180.

A

200.

E

Екзаменаційний тест 1999 р. “Крок 1 Лікувальна справа”

1. При обстеженні букального епітелію чоловіка був виявлений статевий хроматин. Для якої хромосомної хвороби це характерно?

A. Хвороба Дауна

B Трисомія за Х-хромосомою

C. Синдром Шерешевського-Тернера

D Гіпофосфатемічний рахіт

E. Синдром Клайнфельтера

2. Лікар-терапевт так описав свого хворого. "С.П. дуже уважний. Коли він читає книгу або слухає мої поради, то ніщо не може відвернути його уваги: ні поява стороннього в палаті, ні розмова сусідів, ні радіо і т.д. Вся його увага зайнята тим, що зараз відбувається. Тому він дуже відповідально ставиться до моїх призначень і це допомогає в процесі лікування." Про яку властивість уваги йдеться у поданому епізоді?

A. Стійкість уваги

B Переключення уваги

C. Коливання уваги

D Розподіл уваги

E. Концентрація уваги

3. У хворої мало місце розширення судин на передній медіальній поверхні гомілки. Розширенням якої судини зумовлений цей стан?

A. V. saphena parva

B V. poplitea

C. А. tibialis posterior

D А. tibialis anterior

E. V. saphena magna

4. У людини виділяється мало густої слини, знижена її ферментативна активність, збільшений вміст слизу. Найбільш імовірною причиною цього є порушення функції:

A. Привушних залоз

B Під'язикових і піднижньощелепних залоз

C. Піднижньощелепних залоз

D Під'язикових залоз

E. Власних залоз слизової оболонки

5. Хвора звернулась в клініку зі скаргами на слабкість, задишку, швидку стомлюваність, запаморочення. В крові: ер. - 1,8 10^12/л; Hb - 80 г/л; к.п. - 1,5; лейк. - 3,2 10^9/л. У мазку: анізоцитоз, пойкілоцитоз, мегалобласти, мегалоцити. Який найбільш вірогідний діагноз?

A. В_12-дефіцитна анемія

B Постгеморагічна анемія

C. Гострий лейкоз

D Залізодефіцитна анемія

E. Iмуногемолітична анемія

6. В гістопрепараті представлений орган нервової системи, що містить сіру і білу речовину. Сіра речовина розміщена по периферії. Нейрони в ній утворюють три шари: молекулярний, гангліонарний і зернистий. Якому органу належать дані морфологічні ознаки?

A. Кора великого мозку

B Спинний мозок

C. Довгастий мозок

D Міст

E. Мозочок

7. Новонароджена дитина відмовляється від їжі, в неї розвинулось блювання, пронос, а з часом виникло помутніння кришталика. При обстеженні: цукор в крові - 8,5 ммоль/л, а в сечі - 1%. Який найбільш вірогідний діагноз?

A. Галактоземія

B Фенілкетонурія

C. Тирозиноз

D Цистинурія

E. Алкаптонурія

8. З калу та блювотних мас від хворого з підозрою на холеру були виділені культури вібріонів. Проведення якої реакції дозволить визначити вид мікроба, що викликав це захворювання?

A. Преципітації

B Аглютинації Відаля

C. Аглютинації з сироватками, що містять Н-антитіла

D Аглютинації з сироватками, що містять О-антитіла

E. Пасивної аглютинації з еритроцитарним антигенним діагностикумом

9. Під час хірургічної операції виникла необхідність масивного переливання крові. Група крові потерпілого - III (В) Rh+. Якого донора треба вибрати?

A. III (В) Rh+

B III (В) Rh-

C. IV (АВ) Rh-

D II (А) Rh+

E. I (О) Rh-

10. Хворому призначено препарат з вираженими лiпофiльними властивостями. Яким буде головний механiзм його всмоктування?

A. Фiльтрацiя

B Пасивна дифузiя

C. Активний транспорт

D Пiноцитоз

E. Зв'язування з транспортними бiлками

11. Чоловік 38 років проходить курс лікування в стаціонарі з приводу шизофренії. Вихідний вміст в крові глюкози, кетонових тіл, сечовини - в нормі. Шокова терапія регулярними ін'єкціями інсуліну призвела до розвитку інсулінової коми, після чого настало покращення стану хворого. Що було найбільш вірогідною причиною інсулінової коми?

A. Глюкозурія

B Дегідратація тканин

C. Гіпоглікемія

D Метаболічний ацидоз

E. Кетонемія

12. У чоловіка 45 років після смерті співробітника розвинувся страх захворювання. Звернувся до лікаря з проханням обстежити всі органи і системи. Результати клінічного та додаткових методів обстеження – в межах норми. Хворий не заспокоївся. Він звернувся до іншого лікаря з тим же проханням. Отримавши результати обстеження, порівнює їх з попередніми і суб'єктивними відчуттями. Через певний час хворий знову звертається до лікаря і просить направити його на обстеження. Визначте ставлення пацієнта до свого стану.

A. Утилітарне

B Нозофобне

C. Негативне

D Нозофільне

E. Раціоналізація

13. Які зміни з боку ізольованого серця жаби можна очікувати після введення в перфузійний розчин додаткової кількості хлористого кальцію?

A. Зменшення сили скорочення

B Зупинка серця в діастолі

C. Збільшення частоти скорочень

D Збільшення сили скорочень

E. Збільшення частоти і сили скорочень

14. У хворого 21 року видалена пухлина лобної частки правої півкулі головного мозку діаметром 5 см, яка була нечітко відмежована від довколишньої тканини. На розрізі - однорідного вигляду, гістологічно - складається із клітин, численні відростки яких утворюють густі сплетіння. Яка пухлина мала місце у хворого?

A. Хоріоїдпапілома

B Астроцитома

C. Епендімома

D Олігодендрогліома

E. Гангліоневрома

15. Чоловіку 18 років з приводу флегмони плеча була зроблена внутрішньом'язова ін'єкція пеніциліну. Після цього у нього з'явилися тахікардія, ниткоподібний пульс, АТ знизився до 80/60 мм рт.ст. Який вид фармакологічної реакції розвинувся?

A. Потенціювання

B Рефлекторна дія

C. Центральна дія

D Анафілаксія

E. Периферична дія

16. У жінки 45 років відсутні симптоми діабету, але визначається натщесерце підвищений вміст глюкози в крові (7,5 ммоль/л). Який наступний тест необхідно провести?

A. Визначення залишкового азоту в крові

B Визначення глюкози крові натщесерце

C. Визначення толерантності до глюкози

D Визначення ацетонових тіл в сечі

E. Визначення гліколізованого гемоглобіну

17. При дії ультрафіолетових променів у людини темніє шкіра, що є захисною реакцією. Яка захисна речовина синтезується в клітинах шкіри під впливом вказаного фактора?

A. Артинін

B ДНК

C. Вітамін Д

D Триптофан

E. Меланін

18. У кішки в експерименті проводять подразнення однієї з рухових структур головного мозку, внаслідок чого спостерігається підвищення тонусу м'язів-розгиначів з боку стимуляції. У тварини проводили подразнення:

A. Nucleus reticularis medialis

B Nucleus vestibularis lateralis

C. Nucleus ruber

D Nucleus caudatus

E. Nucleus intermedius lateralis

19. Чоловік 67 років страждає на атеросклероз судин головного мозку. При обстеженні знайдена гіперліпідемія. Вміст якого класу ліпопротеїдів плазми крові найбільш вірогідно буде значно підвищений при біохімічному дослідженні?

A. Ліпопротеїди високої щільності

B Комплекси жирних кислот з альбумінами

C. Хіломікрони

D Ліпопротеїди низької щільності

E. Ліпопротеїди дуже низької щільності

20. У пацієнта, носія спадкової серповидної аномалії еритроцитів, захворювання пневмонією супроводжувалося гемолітичною кризою і розвитком анемії. Що є безпосередньою причиною гемолітичної кризи в даному випадку?

A. Гіпероксія

B Гетерозиготність за Нb S

C. Мутація структурного гена

D Гіпоксія, викликана пневмонією

E. Зміна осмолярності крові

21. Хворі на алкоголізм отримують основну масу калорій із спиртними напоями. У них може виникнути характерна недостатність тіаміну (синдром Верніке-Корсакова), при якій спостерігаються порушення функцій нервової системи, психози, втрата пам'яті. Зі зниженням активності якого ферменту пов'язаний цей процес?

A. Альдолаза

B Піруватдегідрогеназа

C. Гексокіназа

D Трансаміназа

E. Алкогольдегідрогеназа

22. В експерименті на тварині, яку утримували в повітрі спиною донизу, ногами догори, спостерігали рефлекторний поворот голови, спрямований на відновлення нормального положення голови у просторі. З подразненням яких рецепторів зв'язаний вказаний рефлекс? A. Вестибулорецепторів півколових каналів

B Вестибулорецепторів переддвер'я

C. Пропріорецепторів кінцівок

D Внутрішніх органів

E. Тактильних рецепторів кінцівок

23. До жінки 52 років була викликана бригада швидкої допомоги у зв'язку з задухою, що розвинулась на фоні бронхоспазму. Після внутрішньом'язового введення ефедрину гідрохлориду спостерігалось тимчасове полегшення дихання. Повторне введення препарату через півгодини мало слабкий ефект, а наступне третє введення не дало результату. Яке явище знаходиться в основі зниження бронхорозширюючої дії препарату?

A. Непрямий антагонізм

B Прямий антагонізм

C. Тахіфілаксія

D Однобічний антагонізм

E. Алергія

24. У чоловіка 70 років внаслідок падіння стався перелом стегнової кістки. Яке найпоширеніше місце перелому цієї кістки у даному віці?

A. Верхня третина

B Середина

C. Виростки

D Шийка

E. Нижня третина

25. У хворого на виразку шлунка, яка розташована у пілоричному відділі на малій кривизні, виникла кровотеча. Яку судину треба перев'язати для зупинки кровотечі?

A. Печінкову артерію

B Ліву шлункову артерію

C. Праву шлункову артерію

D Ліву шлунково-сальникову артерію

E. Праву шлунково-сальникову артерію

26. До жінки 28 років була викликана швидка допомога у зв'язку з нападом бронхіальної астми. Після внутрішньом'язової ін'єкції розчину ефедрину її стан поліпшився. Однак, через 49 хвилин напад повторився. Ефедрин подіяв слабше. Третя атака бронхоспазму, яка почалася через півгодини, ефедрином не знялася. Яке явище лежить в основі цього феномену?

A. Елімінація

B Тахіфілаксія

C. Наркоманія

D Адсорбція

E. Біотрансформація

27. У хворого із запаленням легень спостерігається підвищення температури тіла. Яка біологічно активна речовина відіграє провідну роль у виникненні цього прояву?

A. Лейкотрієни

B Серотонін

C. Iнтерлейкін-I

D Гістамін

E. Брадикінін

28. Хвора 36 років страждає на колагеноз. Збільшення вмісту якого метаболіту найбільш вірогідно буде встановлено у сечі?

A. Уробіліногену

B Сечовини

C. Оксипроліну

D Креатиніну

E. Iндикану

29. При розтині тіла жінки 40 років, померлої від уремії, знайдено: збільшені пістряві нирки, у нирках – потовщення капілярних мембран клубочків у вигляді “дротяних петель”, осередки фібриноїдного некрозу їх стінок та “гіалінові” тромби у просвітах, у ядрах – “гематоксилінові тільця”; у серці – ендокардит Лібмана-Сакса. Яке ураження нирок найбільш вірогідне?

A. Холерний гломерулонефрит

B Вовчаковий нефрит

C. Термінальний гломерулонефрит

D Склеротична нирка

E. Ревматичний гломерулонефрит

30. Чоловік 32 років впродовж 4 років хворіє на хронічний гломерулонефрит. Госпіталізований з ознаками анасарки: АТ - 185/105 мм рт.ст. У крові: Hb - 110 г/л, ер. - 2,6 1012/л, лейк. - 9,5 109/л, залишковий азот - 32 ммоль/л, загальний білок - 50 г/л. Яка зміна з найбільшою вірогідністю вказує на гломерулонефрит з нефротичним синдромом?

A. Анемія

B Артеріальна гіпертензія

C. Гіперазотемія

D Гіпопротеїнемія

E. Лейкоцитоз

31. Після травми гомілки хворий скаржиться на різкі болі в надп'ятковій ділянці гомілки і неможливість підняти тіло на кінчики пальців стопи. Сухожилок якого м'яза пошкоджений?

A. Трьохголового м'яза гомілки

B Переднього великогомілкового м'яза

C. Короткого малогомілкового м'яза

D Довгого малогомілкового м'яза

E. Заднього великогомілкового м'яза

32. В гістологічному препараті паренхіма органа представлена лімфоїдною тканиною, що утворює лімфоїдні вузлики, останні розміщені дифузно і містять центральну артерію. Який анатомічний утвір має дану морфологічну будову?

A. Червоний кістковий мозок

B Тимус

C. Мигдалик

D Лімфатичний вузол

E. Селезінка

33. Хворий 45 років госпіталізований в хірургічне відділення зі скаргами на раптовий гострий біль в надчеревній області. Після обстеження встановлено діагноз: перфоративна (проривна) виразка задньої стінки шлунка. Куди вилився вміст шлунка в момент перфорації?

A. В лівий брижовий синус

B В печінкову сумку

C. В правий брижовий синус

D В сальникову сумку

E. В передшлункову сумку

34. У хворого з тромбофлебітом нижніх кінцівок раптово після навантаження виникла задишка, різкий біль у грудях, ціаноз, набухання шийних вен. Яке найбільш імовірне порушення кровообігу виникло у хворого?

A. Тромбоемболія вінцевих судин

B Тромбоемболія мезентеріальних судин

C. Тромбоемболія судин головного мозку

D Тромбоемболія ворітної вени

E. Тромбоемболія легеневої артерії

35. Жінка 33 років страждає на гепатоцеребральну дистрофію (хвороба Вільсона). В крові знижений вміст церулоплазміну. В сечі різко підвищений вміст амінокислот. Ці зміни в першу чергу обумовлені посиленням процесу:

A. Комплексоутворення амінокислот з міддю

B Переамінування амінокислот

C. Глюконеогенезу

D Разпаду тканинних білків

E. Синтезу сечовини

36. У мишей з відсутнім волосяним покривом (тобто nude - голі) не було клітинних реакцій уповільненого типу. Для цієї патології найбільш вірогідним є:

A. Дефект фагоцитозу

B Порушення гемопоезу

C. Відсутність вилочкової залози

D Дефіцит компонентів системи комплементу

E. Відсутність гамаглобулінів у крові

37. Чоловіку 56 років, що страждає на безсоння, був призначений фенобарбітал. Ритм сну нормалізувався. Але поступово протягом двох місяців дія препарату почала знижуватись і безсоння відновилось. Яке явище призвело до зменшення ефективності снодійного засобу?

A. Погана абсорбція в шлунку

B Толерантність

C. Тахіфілаксія

D Погана розчинність

E. Фіксація в ліпоїдах

38. Експериментальній тварині давали надлишкову кількість глюкози, міченої по вуглецю, протягом тижня. В якій сполуці можна виявити мітку?

A. Арахідоновій кислоті

B Метіоніні

C. Холіні

D Вітаміні А

E. Пальмітиновій кислоті

39. На судово-медичну експертизу був доставлений труп невідомої жінки. На секції в яєчнику виявлено круглий утвір діаметром біля 5 см, що містить пігмент жовтого кольору. Патоолгічних змін в яєчнику не виявлено. З яких клітин складається цей утвір?

A. Iнтерстиціальних

B Міоїдних

C. Фолікулярних

D Фібробластів

E. Лютеїнових

40. Пацієнт (правша) скаржиться на втрату здатності виконувати тонкі рухи, необхідні для написання букв, слів та інших знаків (аграфія). Яка область кори головного мозку уражена?

A. Задній відділ середньої лобної звивини в лівій півкулі

B Середня частина прецентральної звивини зліва

C. Задній відділ середньої лобної звивини справа

D Кутова звивина і шпорна борозна в обох півкулях

E. Середня частина прецентральної звивини справа

41. Хвора 76 років страждає на атеросклероз судин головного мозку 2 стадії. Скаржиться на зниження пам'яті й сльозливість. Під час розмови жвавішає й посміхається, розповідаючи про онука, але через три хвилини, згадавши про смерть батька, плаче. Які емоційні порушення мають місце у пацієнтки ?

A. Слабкодухість

B Ейфорія

C. Апатія

D Депресія

E. Тривога

42. Після ампутації лівої верхньої кінцівки хворий тривалий час відчував сильний біль у ній. Який механізм формування болісних відчуттів найбільш імовірний у цьому випадку?

A. Фантомний

B Рефлекторний

C. Каузалгічний

D Гіпопродукція b-ендорфіну

E. Гіпопродукція енкефаліну

43. Кухар в результаті необачності обпік руку паром. Підвищення концентрації якої речовини викликало почервоніння, набряклість та болючість ураженої ділянки шкіри?

A. Галактозаміну

B Глутаміну

C. Гістаміну

D Тіаміну

E. Лізину

44. Проводять експеримент на спінальній жабі. Після збільшення площі шкіри, на яку діє розчин кислоти, час захисного згинального рефлексу зменшився з 10 до 6 секунд. Який з зазначених механізмів лежить в основі скорочення часу рефлексу?

A. Рециркуляція збудження

B Часова сумація збудження

C. Просторова сумація збудження.

D Iррадіація збудження дивергентними нервовими ланцюгами.

E. Принцип домінанти

45. У тварини в експерименті перерізали спинний мозок вище 5-го шийного сегменту. Як зміниться характер дихання?

A. Стане глибоким і рідким

B Припиниться

C. Стане поверхневим і рідким

D Стане поверхневим і частим

E. Стане глибоким і частим

46. Хворий переніс операцію з приводу резекції пілоричного відділу шлунку. Через рік скаржиться на слабкість, періодичну появу темних кіл під очима, задишку. В крові: Hb - 70 г/л, ер. - 3,0 $10^{12}$ /л. Які зміни еритроцитів в мазках периферичної крові характерні для даного захворювання?

A. Гіперхромні еритроцити

B Еритроцити з тільцями Кебо

C. Гіпохромні еритроцити

D Еритроцити з тільцями Жоллі

E. Макроцити

47. В каріотипі матері 45 хромосом. Встановлено, що це пов'язано з транслокацією 21-ї хромосоми на 15-у. Яке захворювання вірогідніше всього буде у дитини, якщо каріотип батька нормальний?

A. Синдром Дауна

B Синдром Патау

C. Синдром Клайнфельтера

D Синдром Морріса

E. Синдром Едвардса

48. Хворому 50 років з метою лікування черевного тифу призначений левоміцетін, але на наступний день стан хворого погіршився, температура піднялась до $39,6^o$С. Чим пояснити погіршення стану хворого?

A. Дією ендотоксинів збудника

B Реінфекцією

C. Алергічною реакцією

D Нечутливістю збудника до левоміцетину

E. Приєднанням вторинної інфекції

49. При штовханні штанги спортсмен закидає голову назад для максимального підвищення тонусу м'язів-розгиначів верхніх кінцівок. Де розташовані центри рефлексів, які при цьому виникають?

A. В червоних ядрах

B В спинному мозку

C. В ядрах Дейтерса

D В базальних гангліях

E. В руховій корі

50. В діагностиці інфаркта міокарду важлива роль належить методам ензимодіагностики. Визначення рівня вмісту в крові якого ферменту є вирішальним у перші 2-4 години після інфаркту?

A. Альдолази

B Ліпопротеїнліпази

C. Аланінамінотрансферази

D Креатинфосфокінази

E. Ацетилхолінестерази

51. У відділенні новонароджених різко зросла захворюваність дітей на ГРВI, викликану різними групами вірусів. З метою попередження розповсюдження інфекції рекомендовано призначення людського лейкоцитарного інтерферону. Яким шляхом вводять інтерферон?

A. Внутрішньом'язово

B Iнгаляційно

C. Перорально

D В носові ходи

E. Підшкірно

52. Чоловік 37 років госпіталізований до хірургічного відділення з явищами гострого панкреатиту (блювота, пронос, брадикардія, гіпотензія, слабість, явища зневоднювання організму). Який препарат найбільш доцільно використати в першу чергу?

A. Платифілін

B Контрикал

C. Етаперазин

D Ефедрин

E. Но-шпа

53. Дефіцит якого ферменту найчастіше є причиною неповного перетравлення жирів в шлунково-кишковому тракті і збільшення кількості нейтрального жиру в калі?

A. Шлункової ліпази

B Кишкової ліпази

C. Печінкової ліпази

D Ентерокінази

E. Панкреатичної ліпази

54. В клінічних умовах у пацієнта діагностована травма м'язів гомілки. Тканина регенерує повільно за рахунок:

A. Мітотичного поділу міосателітоцитів

B Поділу ядер м'язових волокон

C. Поділу та диференціації фібробластів

D Збільшення кількості міофібрил

E. Збільшення кількості саркоплазми

55. Після накладання джгута у досліджуваного спостерігаються точкові крововиливи на поверхні передпліччя (15 штук). З порушенням функції яких клітин крові це пов'язано?

A. Нейтрофілів

B Макрофагів

C. Базофілів

D Тромбоцитів

E. Еритроцитів

56. У хворого на виразкову хворобу шлунка при проведенні фіброгастроскопії взятий біоптат слизової оболонки в області виразки. З біоптату виготовлений мазок-відбиток, пофарбований за методом Грама; з рештою біоптату проведена проба на уреазну активність. Під час мікроскопії мазка-відбитка виявлені Грам-негативні спіралеподібні мікро-організми, тест на уреазну активність - позитивний. Які бактерії були виявлені?

A. Spirilla minor

B Shigella flexneri

C. Treponema pallidum

D Campylobacter jeuni

E. Helycobacter pylori

57. При розтині чоловіка 60 років у легенях та печінці знайдено багато білуватих вузликів розміром з пшонину. Мікроскопічно виявлені гранульоми з осередком некрозу у центрі, по периферії його - епітеліоїдні, лімфоїдні, плазматичні клітини, а також макрофаги і велика кількість клітин Пирогова-Лангханса, які переважають у інфільтратах. Яка з перелічених гранульом найбільш вірогідна?

A. Епітеліоїдно-клітинна

B Фагоцитомна

C. Макрофагальна

D Гігантоклітинна

E. Гранульома чужорідних тіл

58. Для оцінки придатності води для пиття проведено бактеріологічне дослідження. Який показник характеризує кількість бактерій групи кишкових паличок, що знаходяться в 1 л води?

A. Колі-індекс

B Мікробне число

C. Перфрінгенс-титр

D Колі-титр

E. Титр колі-фага

59. Жінка з I (O) Rh- групою крові вийшла заміж за чоловіка з IV (AB) Rh+ групою крові. Який варіант групи крові і резус-фактора можна очікувати у дітей?

A. I (O) Rh-

B III (B) Rh+

C. IV (AB) Rh+

D I (O) Rh+

E. IV (AB) Rh-

60. Характерними ознаками холери є втрата організмом великої кількості води та іонів натрію. Основою біохімічної дії холерного токсину є:

A. Окислення альдостерону в корі наднирників

B Активація аденілатциклази тонкого кишечника

C. Підсилення секреції реніну клітинами юкстагломерулярного апарату ниркових артеріол

D Гальмування синтезу антидіуретичного гормону в гіпоталамусі

E. Активація синтезу передсердного натрійуретичного фактора

61. До лікаря звернулися пацієнти з аналогічними скаргами: слабість, болі в кишечнику, розлад ШКТ. Після дослідження фекалій вияснилось, що терміновій госпіталізації підлягає один з пацієнтів, у якого були виявлені цисти з чотирма ядрами. Для якого найпростішого характерні такі цисти?

A. Кишкової амеби

B Балантидію

C. Дизентерійної амеби

D Трихомонади

E. Лямблії

62. У чоловіка 32 років діагностована гостра променева хвороба. Лабораторно встановлено різке зниження рівня серотоніну в тромбоцитах. Найбільш вірогідною причиною зниження тромбоцитарного серотоніну є порушення процесу декарбоксилювання:

A. Серину

B 5-окситриптофану

C. Тирозину

D Піровиноградної кислоти

E. Гістидину

63. В гістопрепараті представлена слизова оболонка органа. На поверхні ворсинок в епітеліальному пласті визначаються призматичні каймисті та келихоподібні клітини. В склад якого органа входять дані клітини?

A. Бронха

B Товстої кишки

C. Шлунка

D Тонкої кишки

E. Сечоводу

64. При клiнiчному обстеженнi жінки встановлено: пiдвищення основного обмiну на 40\%, пiдсилене потовидiлення, тахiкардiя, худорлявiсть. Функцiї якої з ендокринних залоз порушено i в якому напрямку?

A. Статевих залоз, гіпофункція

B Кіркової речовини наднирників, гіперфункція

C. Мозкової речовини наднирників, гіперфункція

D Щитовидної залози, гiперфункцiя

E. Підшлункової залози, гiперфункцiя

65. Дитина доставлена в санпропускник в стані асфіксії. При огляді в гортані виявлені білуваті плівки, що обтурують просвіт та легко видаляються. Лікар запідозрив дифтерію. Про яку форму запалення гортані йде мова?

A. Крупозне запалення

B Дифтеритичне запалення

C. Гнійне запалення

D Катаральне запалення

E. Серозне запалення

66. Чоловік 65 років, який страждає на подагру, скаржиться на болі в області нирок. При ультразвуковому обстеженні встановлена наявність ниркових каменів. Підвищення концентрації якої речовини є найбільш імовірною причиною утворення каменів в даному випадку?

A. Білірубіну

B Сечової кислоти

C. Сечовини

D Цистину

E. Холестерину

67. У кішки в експерименті спостерігається підвищений тонус м'язів-розгиначів кінцівок та спини (децеребраційна ригідність). На якому рівні зроблено переріз головного мозку?

A. Нижче вестибулярних ядер

B Між спинним та довгастим мозком

C. Спинного мозку

D Вище червоних ядер

E. Нижче червоних ядер

68. При копрологічному дослідженні встановлено, що кал знебарвлений, у ньому знайдено краплі нейтрального жиру. Найбільш імовірною причиною цього є порушення:

A. Надходження жовчі в кишечник

B Секреції кишкового соку

C. Кислотності шлункового соку

D Процесів всмоктування в кишечнику

E. Секреції підшлункового соку

69. Хвора звернулася до лікаря зі скаргами на появу в випорожненнях білих плоских рухливих утворів, які нагадують локшину. При лабораторному дослідженні виявлені членики з такою характеристикою: довгі, вузькі, з розміщеним поздовжньо каналом матки, яка має 17-35 бічних відгалужень з кожного боку. Який вид гельмінтів паразитує у кишечнику жінки?

A. Taeniarhynchus saginatus

B Echinococcus granulosus

C. Hymenolepis nana

D Taenia solium

E. Diphyllobothrium latum

70. В живильне середовище, де вирощуються клітини тварин, додали амінокислоти лейцину з радіоактивною міткою. Через деякий час методом радіоавтографії виявили високу концентрацію міченої амінокислоти біля певних органоїдів. Цими органоїдами можуть бути:

A. Рибосоми

B Лізосоми

C. Гладка ендоплазматична сітка

D Клітинний центр

E. Апарат Гольджі

71. Судово-медичний експерт при розтині трупу 20-річної дівчини встановив, що смерть настала внаслідок отруєння ціанідами. Порушення якого процесу найбільш вірогідно було причиною смерті дівчини?

A. Транспорту кисню гемоглобіном

B Тканинного дихання

C. Синтезу гемоглобіну

D Синтезу сечовини

E. Транспорту водню за допомогою малат-аспартатного механізму

72. У людини з медикаментозним отруєнням відсутня свідомість, спостерігається міоз і посилення спінальних моносинаптичних рефлексів. Яка речовина могла бути причиною отруєння?

A. Аміназін

B Морфін

C. Димедрол

D Діазепам

E. Етамінал натрію

73. У хворого після видалення щитовидної залози виникли напади судом. Який препарат треба призначити в даному випадку?

A. Преднізолон

B Iнсулін

C. Тироксин

D Соматотропін

E. Паратиреоїдин

74. У чоловіка при ураженні одного з відділів ЦНС спостерігається астенія, м'язова дистонія, порушення рівноваги. Який з відділів ЦНС уражено?

A. Мозочок

B Ретикулярна формація

C. Вестибулярні ядра

D Чорна субстанція

E. Червоні ядра

75. Жінку 59 років непокоять болі в правому колінному суглобі. Об'єктивно: шкіра в області коліна гіперемійована. Температура підвищена, тканини напружені, в ділянці колінного суглоба рентгенологічно виявлено ексудат. Який препарат найбільш ефективний для лікування цієї хворої?

A. Iндометацин

B Парацетамол

C. Промедол

D Анальгін

E. Новокаїн

76. До гінекологічного відділення госпіталізована жінка з клінікою гострого живота з підозрою на позаматкову вагітність. Крізь яке анатомічне утворення або його частину буде виконувати пункцію черевної порожнини гінеколог ?

A. Fornix vaginae posterior

B Cervix uteri

C. Fornix vaginae anterior

D Rectum

E. -

77. При визначенні групи крові по системі АВО за допомогою стандартних сироваток були отримані наступні результати: аглютинація відбулася в сироватках I, II та III груп. Яка група досліджуваної крові?

A. Неможливо визначити

B II (A)

C. III (B)

D IV (AB)

E. I (O)

78. При пункційній біопсії в трансплантованій нирці виявлена дифузна інфільтрація строми лімфоцитами, плазмоцитами, лімфобластами, плазмобластами, а також некротичний артеріїт. Який патологічний процес розвинувся у трансплантаті?

A. Iмунне відторгнення

B Iшемічне пошкодження нирки

C. Гломерулонефрит

D Пієлонефрит

E. Тубулонекроз

79. У чоловіка, який тривалий час не вживав з їжею жирів, але отримував достатню кількість вуглеводів і білків, виявлено дерматит, погане загоювання ран, погіршення зору. Яка можлива причина порушення обміну речовин?

A. Низька калорійність дієти

B Нестача олеїнової кислоти

C. Нестача вітамінів РР, Н

D Нестача пальмітинової кислоти

E. Нестача лінолевої кислоти, вітамінів А, Д, Е, К

80. При ревматизмі у хворих часто виявляється збільшення та деформація суглобів. Який вид запалення лежить в основі цих змін?

A. Альтеративне

B Проліферативне

C. Ексудативне

D Фібринозне

E. Геморагічне

81. У чоловіка 34 років з туберкульозом надниркових залоз шкіра на розтині сірувато-коричневого кольору, артеріальний тиск (прижиттєво) був знижений, спостерігалась адинамія та зниження рівня 17-оксикортикостероїдів в сечі та плазмі крові. Порушення якого пiгменту обумовило клiнiчнi прояви у хворого?

A. Гемосидерину

B Лiпохрому

C. Бiлiрубiну

D Меланiну

E. Лiпофусцину

82. Альбіноси погано переносять вплив сонця - загар не розвивається, а з'являються опіки. Порушення метаболізму якої амінокислоти лежить в основі цього явища?

A. Метіоніну

B Глутамінової

C. Гістидину

D Фенілаланіну

E. Триптофану

83. В експерименті на тварині були пошкоджені нервові шляхи, які проходять в ніжці гіпофізу, що порушило надходження в кров наступних гормонів:

A. Гормонів гіпофізу

B Вазопресину і окситоцину

C. Гормонів аденогіпофізу

D Тиреотропного гормону

E. Аденокортикотропного гормону

84. Жінка 25 років скаржиться на постійний біль в області серця, задишку при русі, загальну слабість. Об'єктивно: шкіра бліда та холодна, акроціаноз. Пульс 96 за 1 хв., АТ - 105/70 мм рт.ст. Межа серця зміщена на 2 см вліво. Перший тон над верхівкою серця послаблений, систолічний шум над верхівкою. Діагностована недостатність мітрального клапана серця. Чим обумовлене порушення кровообігу?

A. Збільшенням об'єму судинного русла

B Зниженням об'єму циркулюючої крові

C. Перевантаженням міокарда підвищеним опором відтоку крові

D Пошкодженням міокарда

E. Перевантаженням міокарда збільшеним об'ємом крові

85. У недоношених новонароджених часто спостерігається синдром дихальної недостатності. Яка найбільш вірогідна причина цього?

A. Ковтання біляплідних вод

B Незрілість альвеол легень в зв'язку з нестачею сурфактанта

C. Внутріутробна гіперкапнія

D Недосконалість нервової регуляції дихального акту

E. Внутріутробна асфіксія

86. У хворого 53 років, що тривалий час страждав на бронхоектатичну хворобу, кровохаркання, з'явилися набряки на обличчі, в поперековій області, в сечі - білок 33 мг/л. Смерть настала від легеневої кровотечі. Результат аутопсії: нирки збільшені в об'ємі, ущільнені, поверхня розрізу має сальний вигляд. Гістологічно відмічено відкладення в клубочках і по ходу канальців гомогенних еозинофільних мас, котрі вибірково зафарбовуються Конго-Рот і дають метахромазію з метиловим фіолетовим. Який патологічний процес мав місце в нирках в даному випадку?

A. Зерниста дистрофія

B Жирова дистрофія

C. Мукоїдне набухання

D Амілоїдоз

E. Гіаліноз

87. РНК, що містить вірус СНIДу, проник всередину лейкоцита і за допомогою ферменту ревертази примусив клітину синтезувати вірусну ДНК. В основі цього процесу лежить:

A. Зворотна транскрипція

B Конваріантна реплікація

C. Зворотна трансляція

D Репресія оперона

E. Дерепресія оперона

88. В лабораторiї була визначена чутливість стафiлокока до антибіотиків i отримані наступні результати дослідження - діаметр зон затримки росту дорівнює: пенiцилiн - 8 мм, оксацилiн - 8 мм, ампiцилiн - 25 мм, гентамiцин - 22 мм. Який метод дослідження був використаний?

A. Біохімічний

B Метод серійних розведень

C. Бактерiоскопiчний

D Метод паперових дисків

E. Біометричний

89. У хворого в анамнезі відмічено тривалий ріст кінцівок, видовжені "павукові" пальці, дефекти кришталика ока, аномалії серцево-судинної системи. Iнтелект в нормі. Які фенотипічні ознаки ще можуть бути у цього хворого?

A. Недорозвинення гонад

B Плоске обличчя і широке перенісся

C. Недорозвинення нижньої щелепи

D Порушення розвитку з'єднувальної тканини

E. Розщеплення м'якого і твердого піднебіння

90. У хворого на рівні 4-го грудного хребця рентгеноскопічно діагностовано чужорідне тіло стравоходу. В ділянці якого стравохідного звуження зупинилося чужорідне тіло?

A. Діафрагмального звуження

B Біфуркаційного звуження

C. Глоткового звуження

D Аортального звуження

E. Абдомінального звуження

91. Жінка 44 років скаржиться на загальну слабість, біль в ділянці серця, значне збільшення маси тіла. Об'єктивно: обличчя місяцеподібне, гірсутизм, АТ - 165/100 мм рт.ст., зріст - 164 см, вага - 103 кг, переважно накопичення жиру на шиї, верхньому плечовому поясі, животі. Що є основним патогенетичним механізмом ожиріння у жінки?

A. Зниження продукції тиреоїдних гормонів

B Зниження продукції глюкагону

C. Підвищення продукції глюкокортикоїдів

D Підвищення продукції інсуліну

E. Підвищення продукції мінералокортикоїдів

92. При електронній мікроскопії в кірковій речовині нирки визначаються структури, вистелені призматичним епітелієм, для якого характерна щіточкова облямівка і глибокі складки плазмолеми в базальній частині. Між складками розташовується велика кількість мітохондрій. Якому відділу нефрона належать описані структури?

A. Петлі Генле

B Звивистому дистальному канальцю

C. Прямому дистальному канальцю

D Нирковому тільцю

E. Проксимальному канальцю

93. У чоловіка 45 років через 3 роки після операції видалення шлунка вміст еритроцитів в крові складає 2,0 $10^{12}$/л, Hb - 85 г/л, кольоровий показник - 1,27. Порушення всмоктування якого вітаміну викликало зміни еритропоезу?

A. С

B Р

C. А

D. В6

E. В12

94. При обстеженні хворої на медіальній поверхні стегна відмічено припухлість, збільшення розмірів вен, утворення вузлів. З боку якої вени відмічається патологія?

A. V.saphena parva

B V.iliaca externa

C. V.poplitea

D V.saphena nagna

E. V.femoralis

95. Хвора звернулася в травмпункт з приводу нагноєння різаної рани. Лікар для очищення рани від гнійних виділень промив її 3\% розчином перекису водню. При цьому піна не утворилася. З чим зв'язана відсутність дії препарату?

A. Неглибока рана

B Низька концентрація Н2О2

C. Спадкова недостатність каталази

D Спадкова недостатність фосфатдегідрогенази еритроцитів

E. Наявність у рані гнійного вмісту

96. При розтині тіла жінки 50 років, померлої від уремії, знайдено аденому паращитовидної залози, деформацію кінцівок, хребта, ребер. Кісткі м'які, на розрізі підвищеної порозності, з пухлиноподібними вузликами, які на розрізі мають пістрявий вигляд, а також з кистами. При мікроскопічному дослідженні виявляється значна перебудова кісткових структур, осередки лакунарного розсмоктування остеоїдної та фіброзної тканини. Яка хвороба найбільш вірогідна?

A. Хронічний остеомієліт

B Остеопороз

C. Хвороба Педжета

D Хвороба Реклінгхаузена

E. Фіброзна дисплазія

97. У неврологічне відділення у несвідомому стані доставлено молодого чоловіка з підозрою на отруєння наркотичною речовиною. Яке з порушень зовнішнього дихання слід очікувати?

A. Асфіксію

B Альвеолярну гіпервентіляцію

C. Дихання Куссмауля

D Дихання Біота

E. Альвеолярну гіповентіляцію

98. Одним із самих небезпечних моментів в патогенезі некрозу міокарда є подальше наростання зон некрозу, дистрофії та ішемії. Важлива роль в цьому належить підвищенню споживання міокардом кисню. Які речовини сприяють даному процесу?

A. Iони хлору

B Холестерин

C. Катехоламіни

D Ацетилхолін

E. Аденозин

99. При мікроскопії шматочків епідермісу уражених ділянок шкіри, взятих з міжпальцевих складок та підошов хворих шахтарів, лікар виявив двоконтурні нитки міцелію, як круглі, так і квадратні спори. Про збудника якої грибкової хвороби може йти мова в даному випадку?

A. Епідермофітії

B Трихофітії

C. Мікроспорії

D Кандидозу

E. Парші

100. Після виробничої травми потерпілий доставлений в лікарню з пошкодженням хребта. Виявлено ураження задніх канатиків спинного мозку на рівні 1-го грудного хребця. Які провідні шляхи постраждали при цьому?

A. Больової та температурної чутливості

B Кірково-спинномозкові

C. Спинно-мозочкові

D Тактильної і пропріоцептивної чутливості

E. Екстрапірамідні

101. У жінки 34 років діагностована спадкова мікросфероцитарна гемолітична анемія (хвороба Мінковського-Шоффара). Який механізм викликав гемоліз еритроцитів у хворої?

A. Ензимопатія

B Гемоглобінопатія

C. Аутоімунне ураження

D Мембранопатія

E. Гіпоплазія кісткового мозку

102. Після інсульту з ураженням ядер гіпоталамусу у хворого виник нецукровий діабет. Що стало причиною посиленого сечовиділення у цього хворого?

A. Зменшення реабсорбції натрію

B Прискорення клубочкової фільтрації

C. Зменшення реабсорбції води

D Зниження артеріального тиску

E. Гіперглікемія

103. Хвора 25 років звернулася зі скаргами на погіршення зору. При огляді виявлено порушення акомодації, зіниця розширена, не реагує на світло. Функція яких м'язів порушена?

A. М'яз, що звужує і м'яз, що розширює зіницю

B М'яз, що звужує зіницю, війковий

C. Верхній навскісний, війковий

D М'яз, що розширює зіницю, війковий

E. Латеральний прямий, м'яз, що звужує зіницю

104. У хворої зі скаргами на болі в епігастральній області оперізуючого характеру при лабораторному обстеженні виявлено підвищений вміст діастази в сечі, а також вміст в калі великої кількості неперевареного жиру. Для якої форми патології ШКТ найбільш характерні змальовані явища?

A. Запалення товстого кишечника

B Гострий апендицит

C. Iнфекційний гепатит

D Виразкова хвороба шлунку

E. Гострий панкреатит

105. Тромбоз коронарної артерії спричинив розвиток інфаркту міокарда. Які механізми ушкодження клітин є домінуючими при цьому захворюванні?

A. Електролітно-осмотичні

B Ацидотичні

C. Протеїнові

D Ліпідні

E. Кальцієві

106. В крові хворого виявлено низький рівень альбумінів і фібриногену. Зниження активності яких органел гепатоцитів печінки найбільш вірогідно обумовлює це явище?

A. Агранулярної ендоплазматичної сітки

B Мітохондрій

C. Гранулярної ендоплазматичної сітки

D Комплексу Гольджі

E. Лізосом

107. Чоловік 52 років, що страждає на екзему, звернувся до дерматолога за консультацією з приводу застосування нового гормонального засобу із групи глюкокортикоїдів. Лікар звернув увагу на наявність фтору в молекулі цього препарату і вказав хворому приблизну різницю між новим засобом і преднізолоном. Чим відрізняється новий препарат?

A. Не збільшує концентрацію глюкози в крові

B Не викликає зниження рівня кортикотропіну

C. Порушує обмін калію

D Діє сильніше

E. Діє слабше

108. На ЕКГ пацієнта мають місце такі зміни: зубець Р – нормальний, інтервал Р-Q - вкорочений, шлуночковий комплекс QRST - розширений, зубець R - двогорбий або двофазний. Яка із форм аритмії має місце у даного пацієнта?

A. Синдром WPW (Вольфа-Паркінсона-Уайта)

B Атріо-вентрикулярна блокада

C. Миготіння шлуночків

D Синдром Фредеріка (тремтіння передсердь)

E. Миготлива аритмія

109. При дослідженні біоптату лімфатичного вузла дано заключення "Лімфогранульоматоз". На підставі наявності яких ознак був поставлений діагноз?

A. Клітин Мікулича

B Клітин Пирогова-Лангханса

C. Клітин Березовського-Штернберга

D Клітин Тутона

E. Клітин Вірхова

110. У хворого 35 років, який часто вживає алкоголь, на фоні лікування сечогінними засобами виникла сильна м'язова і серцева слабість, блювота, діарея, АТ - 100/60 мм рт.ст., депресія. Причиною такого стану є посилене виділення з сечею:

A. Хлору

B Кальцію

C. Натрію

D Фосфатів

E. Калію

111. Хворий 23 років скаржиться на головний біль, зміну зовнішнього вигляду (збільшення розмірів ніг, кистей, рис обличчя), огрубіння голосу, погіршення пам'яті. Захворювання почалося приблизно 3 роки тому без видимих причин. При огляді - збільшення надбрівних дуг, носа, язика. Аналіз сечі без особливих змін. Причиною такого стану може бути:

A. Нестача альдостерону

B Гіперпродукція кортикостероїдів

C. Нестача тироксину

D Гіперпродукція соматотропіну

E. Нестача глюкагону

112. При гнійному отиті гноєм пошкоджено верхню стінку барабанної порожнини. У яку ямку черепа розповсюдився гній із барабанної порожнини?

A. В очну ямку

B В крилопіднебінну ямку

C. В передню черепну ямку

D В задню черепну ямку

E. В середню черепну ямку

113. Зріст дитини 10 років сягає 178 см, маса - 64 кг. З порушенням діяльності якої ендокринної залози це пов'язано?

A. Статевих залоз

B Надниркових залоз

C. Паращитовидної залози

D Гіпофізу

E. Щитовидної залози

114. У хворого спостерігається типова для нападу малярії клінічна картина: озноб, жар, проливний піт. Яка стадія малярійного плазмодію найвірогідніше буде виявлена в крові хворого в цей час?

A. Спорозоїт

B Оокінета

C. Мерозоїт

D Спороциста

E. Шизонт

115. Хворий 70 років протягом кількох років перебуває на диспансерному обліку з приводу цирозу печінки. В результаті незначної травми руки виникла кровотеча. Протромбіновий індекс в нормі, в зв'язку з чим встановлено діагноз фібринолітичної кровотечі. Який з гемостатиків необхідно призначити даному хворому?

A. Кальцію хлорид

B Кислоту амінокапронову

C. Вікасол

D Тромбін

E. Губку гемостатичну

116. Жінка 40 років звернулась до лікаря зі скаргами на болі в дрібних суглобах ніг і рук. Суглоби збільшені, мають вигляд потовщених вузлів. У сироватці крові виявлено підвищений вміст уратів. Причиною є порушення обміну:

A. Амінокислот

B Пуринів

C. Вуглеводів

D Піримідинів

E. Ліпідів

117. Хворий звернувся до лікаря зі скаргами на неможливість приведення та відведення пальців в п'ястно-фалангових суглобах по відношенню до 3 пальця. Функція яких м'язів порушена?

A. Коротких згиначів пальців

B Міжкісткових

C. Довгих згиначів пальців

D Розгиначів

E. Червоподібних

118. При гістологічному дослідженні передміхурової залози, оперативно видаленої у чоловіка 72 років, який скаржився на утруднення сечовипускання, виявлено: збільшення кількості залозистих та м'язових елементів. Часточкова будова залози порушена. Який процес у передміхуровій залозі найбільш вірогідний?

A. Аденокарцинома

B М'язово-фіброзна гіперплазія

C. Залозиста гіперплазія

D Змішана форма простатопатії

E. Простатит

119. У практиці невідкладної терапії та реанімації нерідко зустрічаються стани, що супроводжуються набряком клітин мозку. Для боротьби з цим в організм хворих доцільно вводити речовини, які:

A. Змінюють кислотно-лужний баланс крові

B Понижують системний артеріальний тиск

C. Зменшують ОЦК

D Підвищують колоїдно-осмотичний тиск крові

E. Понижують центральний венозний тиск

120. Жінку 32 років вжалила оса. На шкірі лівої щоки (на місці укусу) - набряк та гіперемія. Який механізм набряку є первинним у даному випадку?

A. Зниження онкотичного тиску крові

B Утруднення лімфовідтоку

C. Підвищення онкотичного тиску тканинної рідини

D Підвищення проникливості капілярів

E. Підвищення гідростатичного тиску крові у капілярах

121. В пологовому будинку під час першого годування у новонародженого було помічено постійне витікання молока з носа. Про яку аномалію розвитку може свідчити цей симптом?

A. Бронхіогенні нориці

B Макростома

C. Мікростома

D Заяча губа

E. Вовча паща

122. У мазку дуоденального вмісту хворого з розладом травлення виявлено найпростіших розміром 10-18 мкм. Тіло грушоподібної форми, 4 пари джгутиків, у розширеній передній частині тіла два ядра, які розміщені симетрично. Який вид найпростіших найбільш імовірний?

A. Лямблія

B Трихомонада

C. Кишкова амеба

D Дизентерійна амеба

E. Балантидій

123. Хворий на гострий інфаркт міокарда в складі комплексної терапії отримував гепарин. Через деякий час з'явилась гематурія. Який препарат (антидот) купіює це ускладення?

A. Фібриноген

B Протаміна сульфат

C. Неодокумарин

D Амінокапронова кислота

E. Вікасол

124. Хворий 59 років госпіталізований у кардіологічне відділення в тяжкому стані з діагнозом: гострий інфаркт міокарда в області задньої стінки лівого шлуночка та перегородки, початковий набряк легень. Який первинний механізм, що викликає розвиток набряку легень у пацієнта?

A. Легенева артеріальна гіпертензія

B Лівошлуночкова недостатність

C. Легенева венозна гіпертензія

D Гіпоксемія

E. Зниження альвеоло-капілярної дифузії кисню

125. При обстеженні міміки хворого виявлено, що він не може скласти губи трубочкою, не може свистіти, а ротова щілина розтягується в боки (поперечна посмішка). На атрофію якого м'язу вказують дані симптоми?

A. Колового м'яза

B Великого виличного м'язу

C. Щічного м'язу

D Жувального м'язу

E. М'язу сміху

126. До жінки, що намагалась покінчити з життям, був викликаний психіатр, який виявив стан ендогенної депресії. Який препарат найбільш доцільно призначити хворій для проведення курсу лікування?

A. Кофеїн

B Амітриптилін

C. Етимізол

D Ноотропіл

E. Сиднокарб

127. У хворого з частими кровотечами з внутрішніх органів і слизових оболонок виявлені пролін і лізин в складі колагенових волокон. Через відсутність якого вітаміну порушене їх гідроксилювання?

A. Тіаміну

B Вітаміну А

C. Вітаміну Н

D Вітаміну С

E. Вітаміну К

128. Через рік після субтотальної резекції шлунка з приводу виразки малої кривизни виявлені зміни в лабораторному аналізі крові: анемія, лейко- і тромбоцитопенія, кольоровий показник - 1,3, наявність мегалобластів та мегалоцитів. Дефіцит якого фактора призводить до цих змін?

A. Гастромукопротеїну

B Гастрину

C. Пепсину

D Хлороводневої кислоти

E. Муцину

129. При бактеріологічному дослідженні випорожнень чотиримісячної дитини з симптомами гострої кишкової інфекції на середовищі Ендо виросли у великій кількості червоні колонії. Які це можуть бути мікроорганізми?

A. Шигели

B Стафілококи

C. Стрептококи

D Сальмонели

E. Ешерихії

130. У дитини зі спадково обумовленими вадами зразу ж після народження спостерігався характерний синдром, який називають "крик кішки". При цьому у ранньому дитинстві малюки мають "нявкаючий" тембр голосу. Під час дослідження каріотипу цієї дитини було виявлено:

A. Додаткову Y-хромосому

B Додаткову 21-у хромосому

C. Додаткову Х-хромосому

D Делецію короткого плеча 5-ї хромосоми

E. Нестачу Х-хромосоми

131. Чоловіку 42 років, що страждає на виразкову хворобу 12-палої кишки призначені такі препарати: де-нол, ампіцилін, метронідазол. Терапія виявилась доцільною. Яка основна причина високої ефективності цього комплексу препаратів?

A. Зменшення впливу патогенної флори кишечника

B Покращення мікроциркуляції

C. Пригнічення Helycobacter pylori

D Підсилення резистентності слизової оболонки 12-палої кишки

E. Підсилення синтезу простагландинів

132. У хворого з жовтяницею встановлено: підвищення у плазмі крові вмісту загального білірубіну за рахунок непрямого (вільного), в калі і сечі – високий вміст стеркобіліну, рівень прямого (зв'язаного) білірубіну в плазмі крові в межах норми. Про який вид жовтяниці можна стверджувати?

A. Жовтяниця немовлят

B Паренхіматозна (печінкова)

C. Хвороба Жильбера

D Гемолітична

E. Механічна

133. При обстеженні хворого була виявлена недостатня кількість імуноглобулінів. Які з перелічених клітин імунної системи виробляють імуноглобуліни?

A. Плазмобласти

B Т-кілери

C. Плазматичні

D Т-хелпери

E. Т-супресори

134. В реанімаційне відділення було доставлене немовля із такими ознаками: блювота, діарея з порушенням росту і розвитку, катаракта, розумова відсталість. Був встановлений діагноз галактоземія. Дефіцит якого ферменту має місце?

A. Гексозо-1-фосфат уридилілтрансферази

B Глюкокінази

C. Глюкозо-6-фосфат дегідрогенази

D УДФ глюкозо-пірофосфорилази

E. УДФ глюкозо-4-епімерази

135. При виконанні столярних робіт працівник випадково вдихнув предмет кулястої форми біля 0,5 см в діаметрі, що викликало реакцію у вигляді сильного кашлю. Подразнення якого відділу дихальних шляхів мало місце?

A. Правого головного бронха

B Лівого головного бронха

C. Трахеї

D Гортані нижче голосових зв'язок

E. Гортані вище голосових зв'язок

136. При вірусоскопії клітинного моношару, зараженого інфекційним матеріалом, лікар-лаборант поставив діагноз - респіраторно-синцитіальна вірусна інфекція. Які зміни викликає цей вірус у культурі клітин?

A. Утворення багатоядерних клітин

B Тотальна деструкція клітинного моношару

C. Круглоклітинна дегенерація

D Відшарування моношару

E. Наявність тілець Бабеша-Негрі

137. У шматочку шкіри 1х2 см, який доставлений для гістологічного дослідження, знайдено новоутворення бурого кольору діаметром 0,5 см. Мікроскопічно пухлина складається з невусних клітин у вигляді тяжів та гнізд, розташованих у дермі, з бурим пігментом у цитоплазмі, який дає негативну реакцію Перлса. Який пігмент найбільш вірогідний?

A. Гемосидерин

B Меланін

C. Білірубін

D Гемомеланін

E. Гематоїдин

138. Хворому для попередження нападу бронхіальної астми лікар призначив кромолiн натрію. Який із зазначених механізмів може бути найбільш характерним для препарату?

A. Зменшення вмісту iмуноглобулiнiв

B Блокада гiстамiнних рецепторів

C. Стабілізація мембран тучних клітин

D Зв'язування вільного гiстамiну

E. Зменшення вмісту iмуногенiв

139. Основна маса азоту з організму виводиться у вигляді сечовини. Зниження активності якого ферменту в печінці приводить до гальмування синтезу сечовини і нагромадження аміаку в крові і тканинах?

A. Уреази

B Пепсину

C. Аспартатамінотрансферази

D Амілази

E. Карбамоїлфосфатсинтази

140. При обстеженні юнака з розумовою відсталістю виявлено євнухоїдну будову тіла, недорозвиненість статевих органів. В клітинах порожнини рота - статевий хроматин. Який метод генетичного дослідження слід застосувати для уточнення діагнозу?

A. Клініко-генеалогічний

B Цитологічний

C. Популяційно-статистичний

D Дерматогліфіка

E. Біохімічний

141. У хворого зі скаргами на задуху проведена біопсія слизової оболонки носової порожнини. Встановлено дiагноз: риносклерома. Якi клiтини типовi для даного захворювання при мiкроскопiчному дослiдженi?

A. Плазмоцити

B Клiтини Мiкулiча

C. Тiльця Шаумана

D Клiтини Пирогова-Лангханса

E. Лімфоцити

142. Який механізм тепловіддачі найбільш ефективно спрацьовує при перебуванні людини в умовах 80\% вологості повітря та температурі навколишнього середовища +35oС?

A. Потовиділення

B -

C. Конвекція

D Радіація

E. Теплопровідність

143. Зріст дорослої людини склав 100 см при пропорційній будові і нормальному розумовому розвитку. Для недостатності вироблення якого гормону характерні вказані ознаки?

A. Соматотропного гормону

B Тироксину

C. Антидіуретичного гормону

D Мінералокофноїдів

E. Гонадотропних гормонів

144. Чоловіку 27 років була проведена туберкулінова проба Манту. Через 24 години на місці ін'єкції відмічається інфільтрат 40х35 мм, шкіра над яким гіперемійована. Яка із груп біологічно активних речовин визначила в основному розвиток алергічного запалення у пацієнта?

A. Лімфокіни

B Біогенні аміни

C. Простагландини

D Лейкотрієни

E. Кініни

145. У хворого запальний гнійний процес шкіри першого міжпальцевого проміжку ноги. Які лімфатичні вузли є регіональними для вказаної ділянки та прореагують болем та припухлістю?

A. Задні великогомілкові та підколінні

B Поверхневі пахвинні

C. Поверхневі та глибокі пахвинні

D Передні великогомілкові

E. Зовнішні клубові

146. Верхні кінцівки людини, що стоїть в стані спокою, знаходяться в легкому згинанні. Що є причиною вказаного стану кінцівок?

A. Рефлекс з м'язових веретен при розтягуванні двоголового м'яза

B Антагоністичний рефлекс зі сторони розігнутих нижніх кінцівок

C. Вроджена готовність до дії

D Тонізуючий вплив лімбічних структур і нової кори

E. Рефлекс з рецепторів переддвер'я вестибулярного апарату

147. У дитини 3 років після перенесеної важкої вірусної інфекції відмічається блювання, що повторюється, втрата свідомості, судоми. При дослідженні крові хворого виявлена гіперамоніємія. З чим може бути пов'язана зміна біохімічних показників крові у даної дитини?

A. З порушенням знешкодження біогенних амінів

B З посиленням гниття білків в кишечнику

C. З пригніченням активності ферментів трансамінування

D З активацією процесів декарбоксилювання амінокислот

E. З порушенням знешкодження аміаку в орнітиновому циклі

148. При розтині дитини віком 6 міс., померлої від сепсису, виявлена відсутність вилочкової залози, зменшення розмірів та маси селезінки. При мікроскопічному дослідженні селезінки виявилася відсутність периартеріальних Т-залежних зон фолікулів зі спустошенням червоної пульпи; у лімфатичних вузлах - відсутність перикортикальної зони, яка в основному представлена Т-лімфоцитами. В-зони у периферійних імунних органах розвинуті нормально. Який патологічний процес найбільш вірогідний?

A. Синдром Гланцмана-Рінікера (недостатність клітинного та гуморального імунітету)

B ВIЛ-інфекція

C. Синдром Брутона (недостатність гуморального імунітету)

D Синдром Дайджорджи (недостатність клітинного імунітету)

E. Акцидентальна інволюція тимусу

149. У дитини 6 місяців спостерігається різке відставання в психомоторному розвитку, бліда шкіра з екзематозними змінами, біляве волосся, блакитні очі, напади судом. Який із наступних лабораторних аналізів крові і сечі найвірогідніше дозволить встановити діагноз?

A. Визначення концентрації лейцину

B Визначення концентрації фенілпірувату

C. Визначення концентрації триптофану

D Визначення концентрації гістидину

E. Визначення концентрації валінду

150. У дівчини 18 років з'вились різкий біль при ковтанні, збільшення лімфатичних вузлів шиї, підвищення to до 39oС. На слизовій оболонці мигдаликів – біло-жовті плівки, які важко відділяються з утворенням дефекту. Стан прогресивно погіршувався. Хвора померла на 8-й день захворювання при наростаючих явищах серцевої недостатності. Які гістологічні зміни в кардіоміоцитах найімовірніше будуть виявлені?

A. Слизова дистрофія

B Жирова дистрофія

C. Гідропічна дистрофія

D Балонна дистрофія

E. Гіаліново-крапельна дистрофія

151. Після введення анальгіну у хворого почалася гостра алергічна реакція, задуха, біль за грудиною. Пульс зник, припинилися серцеві скорочення. Які засоби фармакологічної допомоги слід застосувати в першу чергу?

A. Внутрішньосерцево ввести адреналін

B Внутрішньовенно ввести преднізолон

C. Внутрішньовенно ввести димедрол

D Внутрішньовенно ввести адреналін

E. Внутрішньовенно ввести строфантин

152. У літніх людей підвищується частота виникнення пухлин. Одна з основних причин цього:

A. Зниження інтенсивності утворення антитіл

B Зростання порушення мітозів

C. Зниження активності клітинного імунітету

D Підвищення активності клітинного імунітету

E. Підвищення активності утворення антитіл

153. На гістологічному препараті у сполучній тканині знайдено великі клітини, заповнені базофільною метахроматичною зернистістю; гістохімічно встановлено, що гранули містять гепарин та гістамін. Які клітини найбільш імовірно знайдено в препараті?

A. Адипоцити

B Тучні клітини

C. Макрофаги

D Плазмоцити

E. Фібробласти

154. Людина довгий час проживала в умовах високогір'я. Які зміни в кровоносній системі будуть у неї?

A. Збільшення кількості лейкоцитів

B Збільшення діаметра кровоносних судин

C. Зниження кількості лейкоцитів

D Збільшення кількості гемоглобіну

E. Порідшання пульсу

155. Після побутової травми у пацієнта 18 років з'явились постійні запаморочення, ністагм очей, скандована мова, невпевнена хода. Це свідчить про порушення функції:

A. Чорної субстанції

B Рухової кори

C. Вестибулярних ядер

D Базальних гангліїв

E. Мозочка

156. В інфекційне відділення лікарні госпіталізовано хворого з діагнозом бактеріальної дизентерії. Лабораторними дослідженнями встановлено, що збудник чутливий до багатьох протимікробних засобів, однак у хворого виявлені явища анемії. Який препарат протипоказаний хворому?

A. Фталазол

B Фуразолідон

C. Ентеросептол

D Ампіцилін

E. Левоміцетин

157. Через 3 тижні після гострого інфаркту міокарда у хворого з'явилися болі в серці та суглобах, запалення легень. Який механізм є основним у розвитку постінфарктного синдрому Дресслера у цього хворого?

A. Iшемія міокарда

B Резорбція білків з некротизованої ділянки міокарда

C. Вторинна інфекція

D Тромбоз судин

E. Аутоімунне запалення

158. При фіброгастродуоденоскопії лікарю необхідно оглянути великий сосочок 12-палої кишки. Який анатомічний утвір може служити орієнтиром для його знаходження?

A. Печінково-дуоденальна зв'язка

B Цибулина 12-палої кишки

C. Циркулярні складки 12-палої кишки

D Повздовжня складка 12-палої кишки

E. Дуоденальні залози

159. В травматологічний пункт доставлено хворого з пошкодженням м'язів нижніх кінцівок. За рахунок яких клітин можлива репаративна регенерація м'язових волокон і відновлення функції м'язів?

A. Міоепітеліальних клітин

B Клітин-сателітів

C. Міофібробластів

D Фібробластів

E. Міобластів

160. При визначенні групи крові по системі АВО за допомогою стандартних сироваток були отримані наступні результати: аглютинація відбулася в сироватках I та II груп і не відбулася в сироватці III групи. Яка група досліджуваної крові?

A. II (A)

B I (O)

C. IV (AB)

D III (B)

E. Неможливо визначити

161. У пацієнта 36 років після дорожньої травми виникли параліч м'язів кінцівок справа, втрата больової і температурної чутливості зліва, часткове зниження тактильної чутливості з обох сторін. Для ураження якого відділу мозку вказані зміни є найбільш характерними?

A. Правої половини спинного мозку

B Задніх стовбів спинного мозку

C. Рухової кори зліва

D Лівої половини спинного мозку

E. Передніх стовбів спинного мозку

162. В клініку госпіталізований хворий 15 років з діагнозом поліомієліт. Захворювання супроводжується порушенням функції рухового апарату. Деструкцією яких нервових структур можно пояснити ці порушения?

A. Рухових нейронів спинного мозку

B Ретикулярної формації спинного мозку

C. Передніх корінців спинного мозку

D Чутливих нейронів спинно-мозкових вузлів

E. Нейронів мозочка

163. У померлого від раптової зупинки серця чоловіка 45 років знайдено симетричний тип ожиріння III ступеню, розрив стінки правого шлуночка з гемоперікардом; під епікардом надлишкові відкладення жиру. Мікроскопічно: жирова тканина з епікарду розповсюджується у міокард з атрофією м'язових волокон. Який процес найбільш вірогідний?

A. Гіпертонічна хвороба

B Iшемічна хвороба серця

C. Жирова дистрофія міокарда

D Гострий інфаркт міокарда

E. Ожиріння серця

164. На мікропрепараті зародка людини, взятого із мимовільного викидня, бачимо зародковий щиток, в якому розпізнаються два шари клітин -– енто- і ектодерма. На якому етапі ембріонального розвитку знаходився ембріон?

A. Гатруляції

B -

C. Прогенезу

D Бластуляції

E. Органогенезу

165. Одна з лінз в окулярах короткозорої людини має фокусну відстань 22 см і знаходиться на відстані 1,6 см від ока. Якою повинна бути фокусна відстань контактної лінзи для ока цієї людини?

A. 20,4 см

B 22 см

C. 20 см

D 23,6 см

E. 25 см

166. У хворого важка нефропатія з масивним набряковим синдромом, що ускладнила хронічну бронхоектатичну хворобу. Лабораторні дослідження виявляють значну протеїнурію, циліндрурію, значне зниження вмісту білка в сироватці крові, гіперліпемію, гіпокаліємію та інші відхилення. Що є первинною та найбільш суттєвою ланкою в патогенезі набряків у даного хворого?

A. Зниження онкотичного тиску крові

B Підвищення тиску позаклітинної рідини

C. Підвищення гідростатичного тиску крові

D Блокада лімфовідтоку

E. Підвищення проникності мікросудин

167. Хворий після прийому жирної їжі відчуває нудоту, млявість, з часом з'явилися ознаки стеатореї. В крові холестерин - 9,2 ммоль/л. Причиною такого стану є нестача:

A. Хіломікронів

B Тригліцеридів

C. Жовчних кислот

D Жирних кислот

E. Фосфоліпідів

168. Проводять дослідження на ізольованій збудливій клітині. Встановлено, що поріг сили подразнення клітини суттєво зменшився. Що з зазначеного може бути причиною цього?

A. Блокада енергоутворення у клітині

B Активація натрієвих каналів мембрани

C. Iнактивація кальцієвих каналів мембрани

D Активація калієвих каналів мембрани

E. Iнактивація натрієвих каналів мембрани

169. Кров, взяту у хворого з підозрою на сепсис, висіяли на цукровий бульйон. В цукровому бульйоні утворився придонний осад. При перевисіві на кров'яний агар виросли дрібні прозорі, круглі колонії, оточені зоною гемолізу. В мазку, приготованому з осаду, визначались грам-позитивні коки, розміщені у вигляді довгих ланцюжків. Які мікроорганізми присутні в крові у цього хворого?

A. Сарцини

B Стафілококи

C. Мікрококи

D Тетракоки

E. Стрептококи

170. У приймальному відділенні лікарні відбирають матеріал для бактеріологічного дослідження. З якою метою слід взяти матеріал у хворого з гнійним ураженням глибоких тканин нижньої кінцівки?

A. Для виявлення збудника, щоб попередити внутрішньолікарняне інфікування

B Для встановлення етіології гнійного процесу і визначення чутливості до антибіотиків

C. Для виявлення токсичності збудника

D Для виявлення патогенного стафілокока і визначення антибіотикограми

E. Для підтвердження анаеробної інфекції

171. При зборі анамнезу у хворого на бронхіальну астму лікар вияснив, що напади задухи виникають звичайно вночі, їм передує помірно виражена брадікардія. Яка група холінергічних засобів найбільш показана в даній ситуації?

A. Антихолінестеразні засоби

B М-холіноблокатори

C. Н-холіноміметики

D Реактиватори холінестерази

E. М-холіноміметики

172. На прийом до лікаря прийшов пацієнт дуже високого зросту, з довгими товстими пальцями рук, великою нижньою щелепою і відвислою нижньою губою. Підвищену секрецію якого гормону якої залози можна запідозрити?

A. Гормонів наднирника із групи глюкокортикоїдів

B Соматотропного гормону передньої частки гіпофіза

C. Антидіуретичного гормону задньої частки гіпофіза

D Гормонів щитовидної залози

E. Гонадотропного гормону передньої частки гіпофіза

173. Після народження у дитини виявлено позитивну реакцію сечі з 10\% хлоридом заліза. Для якої спадкової патології це характерно?

A. Цукровий діабет (спадкова форма)

B Тирозиноз

C. Фенілкетонурія

D Галактоземія

E. Алкаптонурія

174. Чоловік 65 років, який страждає на подагру, скаржиться на болі в області нирок. При ультразвуковому обстеженні встановлена наявність ниркових каменів. В результаті якого процесу утворюються ниркові камені?

A. Розпаду гема

B Орнітинового циклу

C. Катаболізму білків

D Відновлення цистеїну

E. Розпаду пуринових нуклеотидів

175. У хворого з субфебрильною температурою в біоптаті збільшеного лімфатичного вузла виявлені численні гранульоми, які містять у центрі казеозний некроз, оточений епітеліоїдними клітинами, велетенськими багатоядерними клітинами Пирогова-Лангханса і лімфоцитами. Для якого захворювання характерні такі патогістологічні зміни?

A. Банальний лімфаденіт

B Лімфолейкоз

C. Лімфосаркома

D Туберкульоз

E. Лімфогранульоматоз

176. Дитина 10-ти місячного віку, батьки якої брюнети, має світле волосся, дуже світлу шкіру і блакитні очі. Зовнішньо при народженні виглядала нормально, але протягом останніх 3 місяців спостерігалися порушення мозкового кровообігу, відставання у розумовому розвитку. Причиною такого стану може бути:

A. Глікогеноз

B Гостра порфірія

C. Гістидинемія

D Галактоземія

E. Фенілкетонурія

177. Хворому з явищами загальних набряків, ацидозу та гіперкаліємії необхідне призначення сечогінного засобу. Який з перерахованих засобів бажано порекомендувати хворому?

A. Манніт

B Тріамтерен

C. Фуросемід

D Дигоксин

E. Спіронолактон

178. У пацієнта 18 років при лабораторному обстеженні виявлено наявність глюкози в сечі при нормальній концентрації її в плазмі крові. Найвірогіднішою причиною цього є порушення:

A. Клубочкової фільтрації

B Канальцевої секреції

C. Секреції глюкокортикоїдів

D Секреції інсуліну

E. Канальцевої реабсорбції

179. На рентгенограмі таза видно, що всі три частини тазової кістки відокремлені проміжками, які відповідають невидимому на рентгенограмах хрящу. Для якого віку це характерно?

A. До 25 років

B До 30 роки

C. До 16 років

D До 40 років

E. До 50 років

180. Дитина 7 років захворіла гостро з підвищення to до 38oC, появи нежитю, кашлю, сльозотечі та крупноплямистого висипання на шкірі. Слизова оболонка зіву набрякла, гіперемійована, з білуватими плямами в області щік. Запалення якого характеру лежить в основі змін слизової оболонки щік?

A. Геморагічне

B Серозне

C. Гнійне

D Катаральне

E. Фібринозне

181. При вивченні біоптату шкіри у складі дерми виявлено судини, які містять товстий шар гладких м'язових клітин у середній оболонці. Як називаються ці судини?

A. Артерії м'язового типу

B Артеріоло-венулярні анастомози

C. Капіляри

D Артеріоли

E. Венули

182. При лабораторному дослідженні крові пацієнта 33 років виявлено реакцію аглютинації еритроцитів в стандартних сироватках I і II груп. Реакції аглютинації з сироваткою III групи і антирезусною сироваткою не відбулась. Кров якої групи, враховуючи систему СДЕ, можна переливати в разі потреби?

A. II (A) Rh-

B III (B) Rh-

C. IV (AB) Rh+

D IV (AB) Rh-

E. I (О) Rh+

183. У хворого в коматозному стані відчувається запах яблук із рота. Вміст глюкози в плазмі крові - 18 ммоль/л. Яку із ком найвірогідніше запідозрити в даному випадку?

A. Гіпоглікемічну

B Токсичну

C. Гіперсмолярну

D Кетоацидемічну

E. Лактатацидемічну

184. Дитина, хвора на фенілкетонурію, страждає на розумову відсталість. Який механізм буде головним у розвитку пошкодження центральної нервової системи?

A. Збільшення екскреції з сечею фенілкетонових тіл

B Підвищення синтезу тирозину

C. Зниження синтезу тиреоїдних гормонів

D Зниження синтезу меланіну

E. Накопичення в крові фенілаланіну і фенілкетонів

185. У хворого 25 років з численних шкірних пустул висіваєтся золотистий стафілокок в асоціації з епідермальним стафілококом, в аналізі харкотиння виявлена пневмоциста карінії, у випорожненнях - криптоспоридії, вульгарний протей і гриби роду кандіда. При якому захворюванні зустрічається таке множинне інфікування умовно-патогенними мікроорганізмами?

A. Сепсисі

B Дисбактеріозі

C. Цукровому діабеті

D СНIДі

E. Медикаментозному агранулоцитозі

186. У жінки 63 років є ознаки ревматоїдного артриту. Підвищення рівня якого з перерахованих нижче показників крові буде найбільш значимим для підтвердження діагнозу?

A. R-глікозидази

B Ліпопротеїдів

C. Кислої фосфатази

D Сумарних глікозаміногліканів

E. Загального холестерину

187. Лікар, перш ніж призначити виснаженому хворому білкове парентеральне харчування, призначив в лабораторії визначити електрофоретичний спектр білків крові. На яких фізико-хімічних властивостях білків оснований цей метод?

A. Гідрофільність і здатність до набрякання

B Наявність заряду

C. Нездатність до денатурації

D В'язкість

E. Оптична активність

188. В препараті представлений порожнистий орган. Слизова оболонка покрита дворядним війковим епітелієм, що переходить в однорядний. М'язова пластинка слизової добре розвинена по відношенню до товщини всієї стінки. Хряща і залоз немає. Який орган представлений в препараті?

A. Середній бронх

B Сечовий міхур

C. Гортань

D Трахея

E. Дрібний бронх

189. Хворий інфекційного відділення скаржився на слабкість, відсутність апетиту, підвищення температури до 38oС. На 7 добу – різкий біль у правому підребер'ї та пожовтіння шкіри. При мікроскопії біоптату печінки: порушення балкової будови, у гепатоцитах – гідропічна та балонна дистрофія, в деяких гепатоцитах – некроз, тільця Каунсільмена, на периферії часточок – збільшена кількість багатоядерних гепатоцитів. Яка форма вірусного гепатиту найбільш вірогідна?

A. Злоякісна

B Безжовтянична

C. Холестатична

D Циклічна жовтянична

E. Хронічна

190. У хворої 46 років з ревматичною вадою серця – стенозом лівого атріовентрикулярного отвору - визначаються задуха при невеликому фізичному навантаженні, серцебиття, ціаноз губ, вологі хрипи в нижніх відділах легень, набряки на нижніх кінцівках. Які гістологічні зміни будуть характерні для печінки?

A. Некроз гепатоцитів в центрі часточки, жирова дистрофія на периферії

B Некроз гепатоцитів в центрі часточки, гіаліново-крапельна дистрофія на периферії

C. Жирова дистрофія гепатоцитів в центрі часточки, некроз на периферії

D Гідропічна дистрофія гепатоцитів в центрі часточки, некроз на периферії

E. Некроз гепатоцитів в центрі часточки, гідропічна дистрофія на периферії

191. До приймального покою лікарні доставлено хворого з тяжким отруєнням невідомою речовиною в стані гострої судинної недостатності. Який із зазначених препаратів необхідно використати для термiнової допомоги?

A. Мезатон

B Нафтизин

C. Iзадрiн

D -

E. Анаприлін

192. До косметолога звернувся пацієнт з проханням позбавити його татуювання на плечі. Яка речовина, що міститься в сполучній тканині, обмежує поширення фарбника та уможливлює такий вид "живопису"?

A. Гепарин

B Фібронектин

C. Гамма-глобулін

D Еластин

E. Гіалуронова кислота

193. В результаті нещасного випадку виникла обтурація трахеї легені. Який етап дихання порушиться першим?

A. Тканинне дихання

B Вентиляція легень

C. Газообмін в легенях

D Газообмін в тканинах

E. Транспорт кисню і вуглекислого газу

194. У дитини 6 місяців на шкірі шиї виявлено плоский вузол червоного кольору, при надавлюванні склом вузол блідне. Який найбільш вірогідний діагноз?

A. Меланома

B Гемангіома

C. Лейоміома

D Лімфангіома

E. Пігментний невус

195. Жінка 25 років звернулась до лікаря зі скаргами на загальну слабість, поганий сон, зниження апетиту і сексуального потягу, біль в ділянці серця, задуху, головний біль, запаморочення, зниження зору. Такий стан настав після сварки з чоловіком. При обстеженні хвора демонстративна, театральна, перебільшує свої страждання, важко і часто дихає, емоційно лабільна, вразлива. Лікареві заявила: “Якщо так жити, то краще вмерти”. Визначте тип особистості.

A. Епілептоїдний

B Гіпертимний

C. Лабільний

D Iстероїдний

E. Астенічний

196. В одному з гірських селищ мала місце масова загибель гризунiв. Одночасно хворіло населення цієї мiсцевостi. Хвороба супроводжувалася швидким підвищенням to до 40oС, вираженою інтоксикацією, збільшенням пахових лiмфовузлiв. У препаратах-мазках з трупного матеріалу виявлені грам-негативні палички овоїдної форми з біполярним забарвленням. Якi мікроорганізми є збудниками цього інфекційного захворювання?

A. Стафілокок

B Паличка чуми

C. Клостридії

D Збудник туляремії

E. Збудник сибірки

197. Хворий на фіброзно-кавернозний туберкульоз помер при зростаючих явищах ниркової недостатності. На розтині – запах сечі, гіпертрофія лівого шлуночка, фібринозний перикардит, фібринозно-геморагічний ентероколіт. Нирки дещо зменшені в розмірах, дуже щільні, з множинними втягненнями. Гістологічно на препаратах, забарвлених Конто-рот - рожеві маси у клубочках та стінках судин, загибель і атрофія більшості нефронів, нефросклероз. Дайте характеристику нирки при даній патології.

A. Амілоїдно зморщені нирки

B Первинно зморщені нирки

C. Атеросклеротично зморщені нирки

D Вторинно зморщені нирки

E. Пієлонефритично зморщені нирки

198. До гастроентерологічного відділення госпіталізовано чоловіка 25 років зі скаргами на голодні болі, відчуття важкості за грудиною, печію. Лікар призначив гастроцепін. Яка фармакодинамічна властивість гастроцепіну примусила лікаря зробити цей вибір?

A. Розслаблення мускулатури жовчних протоків

B Зниження секреції соляної кислоти і гастрину

C. Розслаблення гладкої мускулатури шлунку

D Зниження секреції трипсину підшлунковою залозою

E. Пригнічення Helycobacter pylori

199. Лікар запідозрив у хворого бубонну форму туляремії і направив досліджуваний матеріал для проведення бактеріологічного методу діагностики. В чому особливість цього методу в даному випадку?

A. Виділяють чисту культуру на рідких живильних середовищах

B Виділяють чисту культуру від заражених лабораторних тварин

C. Iдентифікують виділену культуру за антигенною структурою

D Виділяють чисту культуру на густих живильних середовищах

E. Виділяють чисту культуру з використанням середовищ збагачення

200. У хворого, якого доставлено в клініку, чітко виражені розширені підшкірні вени в ділянці пупка (“голова медузи”). Прохідність якої з великих венозних судин порушена?

A. V.renalis

B V.portae.

C. V.iliaca interna

D V.mesenterica inferior

E. V.mesenterica superior

Правильні відповіді на тест“Крок 1 Лікувальна справа" 1999

1.

E

21.

B

41.

A

61.

C

81.

D

101.

D

121.

E

141.

B

161.

A

181.

A

2.

E

22.

B

42.

A

62.

B

82.

D

102.

C

122.

A

142.

A

162.

A

182.

B

3.

E

23.

C

43.

C

63.

D

83.

B

103.

B

123.

B

143.

A

163.

E

183.

D

4.

A

24.

D

44.

C

64.

D

84.

E

104.

E

124.

B

144.

A

164.

A

184.

E

5.

A

25.

C

45.

B

65.

A

85.

B

105.

E

125.

A

145.

B

165.

D

185.

D

6.

E

26.

B

46.

C

66.

B

86.

D

106.

C

126.

B

146.

A

166.

A

186.

D

7.

A

27.

C

47.

A

67.

E

87.

A

107.

D

127.

D

147.

E

167.

C

187.

B

8.

D

28.

C

48.

A

68.

A

88.

D

108.

A

128.

A

148.

D

168.

B

188.

E

9.

A

29.

B

49.

C

69.

A

89.

D

109.

C

129.

E

149.

B

169.

E

189.

D

10.

B

30.

D

50.

D

70.

A

90.

D

110.

E

130.

D

150.

B

170.

B

190.

A

11.

C

31.

A

51.

D

71.

B

91.

C

111.

D

131.

C

151.

A

171.

B

191.

A

12.

B

32.

E

52.

B

72.

B

92.

E

112.

E

132.

D

152.

C

172.

B

192.

E

13.

E

33.

D

53.

E

73.

E

93.

E

113.

D

133.

C

153.

B

173.

C

193.

B

14.

B

34.

E

54.

A

74.

A

94.

D

114.

C

134.

A

154.

D

174.

E

194.

B

15.

D

35.

A

55.

D

75.

A

95.

C

115.

B

135.

E

155.

E

175.

D

195.

D

16.

C

36.

C

56.

E

76.

A

96.

D

116.

B

136.

A

156.

E

176.

D

196.

B

17.

E

37.

B

57.

D

77.

D

97.

E

117.

B

137.

B

157.

E

177.

C

197.

A

18.

B

38.

E

58.

A

78.

A

98.

C

118.

D

138.

C

158.

D

178.

C

198.

B

19.

D

39.

E

59.

B

79.

E

99.

A

119.

D

139.

E

159.

B

179.

C

199.

B

20.

D

40.

A

60.

B

80.

B

100.

D

120.

D

140.

B

160

D

180.

D

200.

B

Екзаменаційний тест 2000 р.

“Крок 1 Лікувальна справа”

1. Хворий госпіталізований зі скаргами на неприємні відчуття в ділянці серця, напади гострої слабкості, потьмарення свідомості. Обстеження хворого і кардіограми виявили наявність атріовентрикулярної блокади II ступеня з нападами Моргана-Адамса-Стокса. Який препарат слід призначити в даній ситуації?

А. Анаприлін

В. Нітрогліцерин

С. Строфантин

D. Атропін

Е. Новокаїнамід

2. У хворого з простатопатією спостерігається постійне порушення виділення сечі, в зв'язку з чим проведена операція простатектомії. Під час операції стінка сечового міхура потовщена, слизова оболонка рожевого кольору, просвіт міхура зменшений. Який характер патологічного процесу в стінці сечового міхура?

А. Гіпертрофія

В. Атрофія

С. Гіперплазія

D. Метаплазія

Е. Гіпоплазія

3. До лікарні доставлено тяжкого інфекційного хворого, у якого виявлено також порушення функції печінки і нирок. Який із зазначених препаратів буде найбільш небажаним для призначення?

A. Цефалоксин

В. Ампіцилін

С. Пеніцилін

D. Левоміцетин

Е. Неоміцин

4. При лабораторному обстеженні у вагітної жінки виявлене збільшення вмісту фенілаланіну в крові. Як це може вплинути на дитину?

А. Ніякого впливу

В. Можливе народження дитини з галактоземією

С. Можливе народження дитини з олігофренією

D. Можливе народження дитини з гемофілією

Е. Можливе народження дитини з подагрою

5. У хворого з проносними явищами з'явились спастичні болі в області тонкого кишечника. Який препарат треба застосувати для терапевтичної допомоги?

А. Кислота дегідрохолева

В. Но-шпа

С. Прозерин

D. Натрію сульфат

Е. Карбохолін

6. В експерименті на нерв поклали тампон з ізотонічним розчином сахарози. В цьому місці нерв втратив збудливість і провідність. Це пов'язано з тим, що у складі розчину:

А. Відсутні іони натрію

В. Відсутні білки

С. Відсутні іони калію

D. Багато вуглеводів

Е. Відсутні іони хлору

7. Дослідження крові і сечі пацієнта показало, що концентрація сечовини в добовій сечі дорівнює 180 ммоль/л, а в крові - 1,5 ммоль/л. Порушення якого метаболічного шляху можна припустити?

А. Орнітинового циклу

В. Гліколізу

С. Циклу Кребса

D. Глюконеогенезу

Е. Пентозофосфатного циклу

8. Проводять експеримент на спінальній жабі. Після збільшення площі шкіри, на яку діє розчин кислоти, час захисного згинального рефлексу зменшився з 10 до 6 секунд. Який з зазначених механізмів лежить в основі скорочення часу рефлексу?

А. Рециркуляція збудження

В. Іррадіація збудження дивергентними нервовими ланцюгами

С. Часова сумація збудження

D. Принцип домінанти

Е. Просторова сумація збудження

9. В процесі транскрипції здійснюється синтез комплементарної молекули РНК на матриці ДНК. Який фермент каталізує цей процес?

А. Праймаза

В. ДНК-залежна РНК-полімераза

С. Хеліказа

D. ДНК-полімераза

Е. Топоізомераза

10. У жінки 42 років, яка перенесла операцію на нирці, після наркозу розвинулися явища рекураризації і припинилося дихання. Як міорелаксант був застосований дитилін. Який засіб найбільш доцільно застосовувати для відновлення тонусу м'язів?

А. Кофеїн

В. Прозерин

С. Стрихнину нітрат

D. Плазму крові

Е. Галантоміну гідробромід

11. Хворому встановлено діагноз: злоякісна пухлина легень. Яка з особливостей пухлинного росту найбільш вірогідно свідчить про злоякісність?

А. Безмежний ріст

В. Нерегульований ріст

С. Інфільтративний ріст

D. Експансивний ріст

Е. Ріст з однієї пухлини

12. Один із методів лікування при отруєнні метанолом полягає в тому, що хворому призначають етанол всередину або внутрішньовенно у кількостях, які у здорової людини викликають інтоксикацію. Чому таке лікування виявляється ефективним?

А. Етанол конкурує з метанолом за активний центр алкогольдегідрогенази

В. Етанол зв'язує алостеричний центр алкогольдегідрогенази, яка інактивується

С. Етанол конкурує з метанолом за активний центр альдегіддегідрогенази

D. Етанол блокує кофермент алкогольдегідрогенази

Е. Етанол розщепляється швидше, ніж метанол, в результаті чого утворюються менш токсичні продукти

13. У хворого після парентерального введення антибіотика через 25 хвилин відмічались: нудота, слабкість, болі в животі, серцебиття, утруднення дихання, на шкірі з'явились висипання у вигляді пухирів. Яка стадія алергічної реакції відмічається у хворого?

А. Сенсибілізації

В. Патохімічна

С. Біохімічна

D. Імунологічна

Е. Патофізіологічна

14. У хворого з деструктивними змінами в м'язовій тканині в крові виявлена велика кількість ізоферментів креатинкінази МВ-форми. Який найбільш імовірний діагноз?

А. Атрофія м'язів

В. Інфаркт міокарда

С. Дистрофія м'язів

D. Поліоміозит

Е.-

15. Жінка 24 років скаржиться на сухість у роті, зниження маси тіла, незважаючи на підвищений апетит. При обстеженні: зріст 162 см, маса тіла 65 кг, рівень глюкози в крові 8,3 ммоль/л, глюкозурія. Для якого стану найбільш характерні ці симптоми?

А. Аліментарна глюкозурія

В. Стероїдний діабет

С. Нецукровий діабет

D. Цукровий діабет

Е. Нирковий діабет

16. При розтині трупа хворого, померлого від легеневої недостатності, виявлена "велика пістрява легеня", некротичний трахеобронхіт, фібринозно-геморагічний плеврит. На яку хворобу найбільш вірогідно страждав хворий?

А. Часточкова зливна пневмонія

В. Крупозна пневмонія

С. Рак легені з параканкрозною пневмонією

D. Грип, що ускладнився пневмонією

Е. Бронхоектатична хвороба

17. При гепатиті, інфаркті міокарда в плазмі крові хворих різко зростає активність аланін- і аспартамамінотрасфераз. Які причини зростання активності цих ферментів в крові?

А. Збільшення швидкості розпаду амнокислот в тканинах

В. Підвищення активності ферментів гормонами

С. Нестача піридоксину

D. Зростання швидкості синтезу амінокислот в тканинах

Е. Пошкодження мембран клітин і вихід ферментів в кров

18. При розтині тіла жінки 50 років, померлої від уремії, знайдено аденому па-ращитовидної залози, деформацію кінцівок, хребта, ребер. Кісткі м'які, на розрізі підвищеної порозності, з пухлиноподібними вузликами, які на розрізі мають пістрявий вигляд, а також з кистами. При мікроскопічному дослідженні виявляється значна перебудова кісткових структур, осередки лакунарного розсмоктування остеоїдної та фіброзної тканини. Яка хвороба найбільш вірогідна?

А. Остеопороз

В. Хронічний остеомієліт

С. Хвороба Реклінгхаузена

D. Фіброзна дисплазія

Е. Хвороба Педжета

19. Хворий 56 років на роботі мав контакт з діетилнітрозаміном, скаржиться на біль в правому підребер'ї, слабкість, втрату апетиту, зниження працездатності. Об'єктивно: поверхня печінки горбиста, відмічається спленомегалія, асцит. Температура тіла 37,2°С, ШОЕ -25 мм/год., нейтрофільний лейкоцитоз, гіпохромна анемія. Яке захворювання розвинулося у пацієнта?

А. Рак печінки

В. Гепатит

С. Цироз печінки

D. Жовчно-кам'яна хвороба

Е. Дискінезія жовчних проток

20. Внаслідок посиленого потовиділення та зневоднення організму у пацієнта зросла осмолярність сечі та зменшився діурез. Зміна продукції якого гормону забезпечує в першу чергу компенсаторну затримку води?

А. Кортикостерону

В. Альдостерону

С. Антидіуретичного

D. Тироксину

Е.Інсуліну

21. На ЕКГ виявлено значне збільшення інтервалу PQ. Це означає, що сповільнено проведення збудження:

А. Волокнами Пуркін'є

В. Передсердями

С. Пучком Гіса

D. Атріовентрикулярним вузлом

Е. Шлуночками

22. У дитини 6 місяців уповільнення моторного та психічного розвитку, посвітління шкірних покривів, волосся, райдужної оболонки очей, позитивна проба Фелінга. Яке спадкове захворювання виявлено у дитини?

А. Фенілкетонурія

В. Альбінізм

С. Хвороба Дауна

D. Алкаптонурія

Е. Галактоземія

23. При запальних процесах в жовчному міхурі порушуються колоїдні властивості жовчі, що може призвести до утворення жовчних каменів. Кристалізація якої речовини є головною причиною їх утворення?

А. Хлориди

В. Урати

С. Холестерин

D. Оксалати

Е. Фосфати

24. Хворий безконтрольно використовував велику кількість лікарських засобів. Як називається це явище?

А. Ідіосинкразія

В. Медикаментозна хвороба

С. Поліпрагмазія

D. Медикаментозна залежність

Е. Тахіфілаксія

25. У чоловіка 38 років, який знаходиться у відділенні реанімації, знайдено зменшення гематокриту до 30%. Який процес найбільш вірогідно призвів до такого стану?

А. Посилене сечоутворення

В. Блювота

С. Пронос

D. Інтенсивне потовиділення

Е. Кровотеча

26. При пункційній біопсії в трансплантованій нирці виявлена дифузна інфільтрація строми лімфоцитами, плазмоцитами, лімфобластами, плазмобластами, а також некротичний артеріїт. Який патологічний процес розвинувся у трансплантаті?

А. Пієлонефрит

В. Гломерулонефрит

С. Ішемічне пошкодження нирки

D. Тубулонекроз

Е. Імунне відторгнення

27. В лікарню швидкої допомоги доставили дитину 7 років в стані алергічного шоку, який розвинувся після укусу оси. В крові підвищена концентрація гістаміну. В результаті якої реакції утворюється цей амін?

А. Декарбоксилювання

В. Гідрооксилювання

С. Дегідрування

D. Дезамінування

Е. Відновлення

28. В період падіння температури тіла у хворого на малярію настало зниження AT до 80/45 мм рт.ст., послаблення серцевої діяльності, тахікардія. Як називається таке ускладнення?

А. Бактеріальний шок

В. Інфекційно-токсичний колапс

С. Геморагічний колапс

D. Ортостатичний колапс

Е. Геморагічний шок

29. При профогляді у людини, що не має скарг на стан здоров'я, виявлено лейкоцитоз. Причиною цього може бути те, що кров для аналізу здана після:

А. Фізичного навантаження

В. Розумової праці

С. Відпочинку на курорті

D. Значного вживання води

Е. Вживання алкоголю

30. В селищі зареєстрований спалах діарейного захворювання. В зв'язку з підозрою на холеру випорожнення хворих направлені в бактеріологічну лабораторію для підтвердження цього припущення. Якими експрес-методами можна скористатись в даному випадку?

А. Реакція зв'язування комплементу

В. Реакція преципітації

С. Реакція аглютинації

D. Реакція імунофлюоресценції

Е. Реакція кільцепреципітації

31. При огляді зіву у хворого на ангіну визначається гіперемія слизової оболонки піднебіння, мигдалики збільшені в розмірах, червоні, на їх поверхні помітні дрібні біло-жовті осередки. Який клініко-морфологічний варіант ангіни найбільш вірогідний в даному випадку?

А. Гнійна

В. Катаральна

С. Лакунарна

D. Фібринозна

Е. Некротична

32. У чоловіка 60 років під час операції з приводу резекції шлунка розвинулася серцева слабкість. AT знизився до 70/50 мм рт.ст., пульс ниткоподібний. Проте анестезіолог не ввів адреналін. Який загальний анестетик, при якому протипоказано введення адреналіну, був використаний?

А. Фторотан

В. Гексенал

С. Ефір

D. Кетамін

Е. Пропанідид

33. При обстеженні у хворого був встановлений діагноз алкаптонурії. Дефіцитом якого фермента зумовлена ця патологія?

А. Тироксингідроксилази

В. Фенілаланінгідроксилази

С. Тирозинази

D. Оксидази гомогентизинової кислоти

Е. Моноамінооксидази

34. Який механізм тепловіддачі найбільш ефективно спрацьовує при перебуванні людини в умовах 80% вологості повітря та температурі навколишнього середовища +35°С?

А. Потовиділення

В. Радіація

С. Теплопровідність

D. Конвекція

Е.-

35. У хворого колаптоїдний стан через зниження тонусу периферичних судин. Який лікарський засіб показаний для усунення гострої судинної недостатності периферичного походження?

А. Каптопріл

В. Мезатон

С. Прозерин

D. Ізадрин

Е. Празозин

36. Куди треба провести катетер для забору лімфи з грудної лімфатичної протоки?

А. У ліву пахвову вену

В. У правий венозний кут

С. У верхню порожнисту вену

D. У нижню порожнисту вену

Е. У лівий венозний кут

37. Пацієнту з гострим інфарктом міокарда внутрішньовенне крапельно введено 1500 мл різних розчинів протягом 8 годин, кисень інтраназально. Смерть настала від набряку легень. Що стало причиною набряку легень?

А. Перевантаження лівого шлуночка об'ємом

В. Зменшення онкотичного тиску за рахунок гемодилюції

С. Алергічна реакція

D. Нейрогенна реакція

Е. Інгаляція кисню

38. Хворий 2 роки тому переніс операцію резекції пілоричного відділу шлунка. Спостерігається слабкість, періодична поява темних кіл перед очима, задишка. В аналізі крові: НЬ - 70 г/л, ер. - 3,0 * 1012/л, к. п. - 0,7 Які зміни еритроцитів в мазках крові найбільш характерні для даного стану?

А. Овалоцити

В. Мегалоцити

С. Шизоцити

D. Мікроцити

Е. Макроцити

39. При обстеженні хворого, що страждає на гострий панкреатит, в крові виявлено підвищений вміст хіломікронів. Активність якого фермента різко знижена при даній патології?

А. Ліпази підшлункової залози

В. Ліпопротеїнліпази

С. Панкреатичної фосфоліпази

D. Тканинної тригліцеридліпази

Е. Тканинної дигліцеридліпази

40. При перкусії правої легені визначено, що її нижній край проектується по середній ключичній лінії на VI ребро, по передній пахвовій - на VII ребро, по середній пахвовій - на VIII ребро, по задній пахвовій - на IX ребро, по лопатковій - на X ребро, по паравертебральній - на XI ребро. Як розташований нижній край легені відносно норми?

А. Не зміщений

В. Зміщений догори по передній пахвовій лінії

С. Зміщений догори по середній пахвовій лінії

D. Зміщений догори по задній пахвовій лінії

Е. Зміщений догори по середній ключичній лінії

41. У пологовий будинок ургентно доставлена вагітна жінка с загрозою передчасних пологів. Який препарат необхідно призначити в даній ситуації?

А. Пахікарпіну гідрохлорид

В. Катарніну хлорид

С. Партусістен

D. Окситоцин

Е. Ерготаміну гідротартрат

42. До лікаря звернувся хворий зі скаргами на гнійничкові висипання на шкірі обличчя. При бактеріологічному аналізі вмісту гнійничків знайдено золотистий стафілокок і встановлено діагноз: стафілококова піодермія. Який найбільш ефективний препарат для обробки гнійничкових висипань?

А. Фурацилін

В. Бриліантовий зелений

С. Метиленовий синій

D. Дегміцид

Е. Спирт етиловий

43. При хворобі Аддісона спостерігається гіперпігментація шкірних покривів. Це пов'язують зі спільністю джерел розвитку меланоцитів шкіри і мозкової речовини наднирників. Що є джерелом їх розвитку?

А. Нервовий гребінь

В. Мезодерма

С. Мезенхіма

D. Ектодерма

Е. Ентодерма

44. У хворого з обмороженням просліджуються явні ознаки гіпотермії. Разом з іншими першочерговими реанімаційними засобами йому призначається гарячий та дуже солодкий чай. Яка мета цього призначення?

А. Профілактика шоку

В. Стимуляція енергетичного обміну

С. Стимуляція секреції інсуліну

D. Пригнічення секреції інсуліну

Е. Стимуляція секреції шлункового соку

45. Чоловік 32 років знаходиться в стані стресу через виробничий конфлікт. Який гормон забезпечив запуск стресової реакції організму?

А. Меланотропін

В. Тирокальцитонін

С. Паратгормон

D. Тестостерон

Е. Адреналін

46. При тривалому голодуванні у людей з'являються "гол одні "набряки. В чому причина їх появи?

А. Зниження осмотичного тиску крові

В. Підвищення онкотичного тиску крові

С. Зниження секреції вазопресину

D. Підвищення секреції вазопресину

Е. Зниження онкотичного тиску крові

47. В медико-генетичну консультацію звернулася вагітна жінка, яка працювала на шкідливому виробництві. Після проведення амніоцентезу стало питання про переривання вагітності. Лікарі пояснили жінці, що її майбутня дитина не буде життєздатною і матиме вади в будові серця, нирок, травної системи, розщеплення м'якого і твердого піднебіння, недорозвиток або відсутність очей. Про яке порушення в каріотипі йшла мова в цьому випадку?

А. Трисомія -13

В. Трисомія - 21

С. Моносомія - X

D. Трисомія - У

Е. Полісемія - X

48. Хворий 65 років, що страждав на атеросклероз, госпіталізований до хірургічного відділення з приводу розлитого гнійного перитоніту. Під час операції діагностовано тромбоз брижових артерій. Яка найбільш вірогідна причина перитоніту?

А. Стаз

В. Ішемія ангіоспастична

С. Ішемічний інфаркт

D. Геморагічний інфаркт

Е. Ішемія компресійна

49. У чоловіка 38 років з жовтушною шкірою відмічається анемія, збільшена селезінка, гіпербілірубінемія (білірубін непрямий), уробілінурія, гіперхолічний кал. Для якого стану найбільш характерні ці зміни?

А. Надпечінкова жовтяниця

В. Підпечінкова жовтяниця

С. Клітинно-печінкова жовтяниця

D. Синдром Жильбера

Е. Синдром печінкової недостатності

50. За результатами аналізів шлункового соку хворого встановлено: загальна кислотність - 28 ммоль/л, вільна соляна килота - 1,5 ммоль/л, вміст гастромукопротеїну знижений. Нестача якого вітаміну спостерігається в організмі?

А. Біофлавоноїдів

В. Фолієвої кислоти

С. Пантотенової кислоти

D. Нікотинаміду

Е. Кобаламіну

51. При медогляді хлопчика 5 років виявили значне підвищення числа еозинофілів в одиниці об'єму крові. Що із вказаного може бути причиною еозинофілії?

А. Ожиріння

В. Глистні інвазії

С. Гіподинамія

D. Гіпотермія

Е. Фізичне навантаження

52. У пацієнта з захворюванням нирок, що супроводжується ішемією паренхіми, відмічається високий рівень артеріального тиску. Який провідний фактор є причиною підвищення AT у даного хворого?

А. Надлишок антидіуретичного гормону

В. Надлишок ангіотензину II

С. Збільшення серцевого викиду

D. Підвищення тонусу симпатичної нервової системи

Е. Гіперкатехолемія

53. Альбінізм успадковується як аутосомна рецесивна ознака. У родині, де обоє батьків здорові, народилася дитина-альбінос. Яка ймовірність народження здорової дитини у цієї пари?

А. 50%

В. 75%

С. 100%

D. 25%

Е. 10%

54. При бактеріологічному дослідженні випорожнень чотиримісячної дитини з симптомами гострої кишкової інфекції на середовищі Ендо виросли у великій кількості червоні колонії. Які це можуть бути мікроорганізми?

А. Ешерихії

В. Сальмонели

С. Стафілококи

D. Стрептококи

Е. Шигели

55. До лікаря-інфекціоніста на прийом прийшов хворий зі скаргами на гарячку, яка триває три дні, загальну слабкість, безсоння, погіршення апетиту. Лікар запідозрив черевний тиф. Який метод лабораторної діагностики найдоцільніше призначити для підтвердження діагнозу?

А. Виділення білікультури

В. Виділення копрокультури

С. Виділення уринокультури

D. Виділення гемокультури

Е. Виділення мієлокультури

56. Після операції апендектомії у хворого залишилась знижена чутливість шкіри над пахвинною складкою, в ділянці лобка та зовнішніх статевих органів на стороні проведеної операції. Гілки якого нерва були ушкоджені хірургом в процесі оперативного втручання?А. Статево-стегнового

В. Затульного

С. Стегнового

D. Клубово-пахвинного

Е. Клубово-поперекового

57. У хворого 50 років з'явились болі при ковтанні, відчуття печіння. При візуальному дослідженні в області кореня язика видно білувату бляшку з тріщинами, виступаючу над поверхнею слизової. При дослідженні біоптату: епітелій слизової сплощений за рахунок проліферації базального шару, визначаються явища паракератозу, лімфоплазмоцитарні інфільтрати. Який патологічний процес спостерігається в порожнині рота?

А. Фіброма

В. Рак

С. Лейкоплакія

D. Глосит

Е. Папілома

58. У хворого, що страждає на ішемічну хворобу серця, після проведеного лікування в серцевому м'язі значно зменшився вміст лактату. Активацією якого біохімічного процесу в міокарді можна це пояснити?

А. Глюкогенезу

В. Глюконеогенезу

С. Глікогенолізу

D. Аеробного окислення глюкози

Е. Пентозо-фосфатного циклу

59. При дослідженні функціонального стану мозку методом електроенцефалографії у чоловіка 40 років виявлено дельта-ритм. При якому функціональному стані кори великих півкуль він реєструється?

А. Судомна активність мозкових структур

В. Висока активність кори великих півкуль

С. Стан глибокого сну

D. Стан легкого сну

Е. Стан помірної активності кори великих півкуль

60. При обстеженні чоловіка 45 років, що тривалий час перебував на рослинній дієті, виявлено негативний азотистий баланс. Яка особливість раціону стала причиною цього явища?

А. Надмірна кількість вуглеводів

В. Надмірна кількість води

С. Недостатня кількість білків

D. Недостатня кількість жирів

Е. Недостатня кількість жирів і білків

61. Після автомобільної аварії у чоловіка рентгенологічно встановлено пошкодження хребта на рівні І - II грудних сегментів зі зміщенням. Неврологічно визначається порушення рухової функції та випадання глибокої чутливості праворуч і випадання поверхневої чутливості ліворуч. Який можливий варіант пошкодження спинного мозку знайдено у хворого?

А. Синдром Броун-Секара

В. Центральний параліч

С. Пошкодження передніх рогів спинногомозку

D. Пошкодження пірамідного шляху

Е. Тотальне пошкодження спинного мозку

62. При обстеженні хворого виявлені наступні клінічні прояви: шкірні покриви рожеві, теплі на дотик, сухі, ЧСС- 92/хв.,ЧД - 22/хв., температура тіла - 39,2°С. Яке співвідношення процесів утворення і віддачі тепла в описаному періоді гарячки?

А. Теплопродукція дорівнює тепловіддачі

В. Теплопродукція перевищує тепловіддачу

С. Теплопродукція нижче тепловіддачі

D. Зниження тепловіддачі на фоні незміненої теплопродукції

Е. Посилення теплопродукції без зміни тепловіддачі

63. Лікар запідозрив у хворого бубонну форму туляремії і направив досліджуваний матеріал для проведення бактеріологічного методу діагностики. В чому особливість цього методу в даному випадку?

А. Виділяють чисту культуру з використанням середовищ збагачення

В. Виділяють чисту культуру на густих живильних середовищах

С. Виділяють чисту культуру від заражених лабораторних тварин

D. Ідентифікують виділену культуру за антигенною структурою

Е. Виділяють чисту культуру на рідких живильних середовищах

64. При дослідженні сечі у хворого виявлена протеїнурія (5 г/л) за рахунок низькомолекулярних білків, гематурія зі вилуженими еритроцитами. Порушення якої функції нирок відображають ці показники?

А. Позаниркові порушення

В. Підвищення секреції канальців

С. Зміна екскреції клубочків

D. Зниження реабсорбції канальців

Е. Підвищення проникливості клубочків

65. Проводять дослідження на ізольованій збудливій клітині. Встановлено, що поріг сили подразнення клітини суттєво зменшився. Що з зазначеного може бути причиною цього?

А. Блокада енергоутворення у клітині

В. Інактивація натрієвих каналів мембрани

С. Інактивація кальцієвих каналів мембрани

D. Активація калієвих каналів мембрани

Е. Активація натрієвих каналів мембрани

66. Мати звернулась до педіатра з дитиною, у якої виникли скарги на сильне свербіння в ділянці анального отвору, що посилюється вночі. Дитина неспокійна. При дослідженні калу на я/г знайдені яйця гостриків. Який протигельмінтний засіб найбільш доцільний?

А. Мебендазол

В. Фенасал

С. Дитразин

D. Аміноакрихін

Е. Трихлорофен

67. У хворого з жовтяницею при пункційній біопсії печінки виявлено порушення структури часточок, розповсюджена балонна дистрофія і некроз гепатоцитів, багато тілець Каунсілмена, дифузна інфільтрація строми печінки лімфоцитами. Про яке захворювання свідчать виявлені патологічні зміни?

А. Гнійний гепатит

В. Цироз печінки

С. Рак печінки

D. Вірусний гепатит

Е. Мікотичний гепатит

68. В діагностиці інфаркта міокарда важлива роль належить методам ензимодіагностики. Визначення рівня вмісту в крові якого ферменту є вирішальним у перші 2-4 години після інфаркта?

А. Креатинфосфокінази

В. Ацетилхолінестерази

С. Ліпопротеїнліпази

D. Аланінамінотрансферази

Е. Альдолази

69. У хворого хронічним гломерулонефритом виявлені симптоми анемії. Що обумовило появу цих симптомів ?

А. Зниження синтезу еритропоетинів

В. Втрата еритроцитів з сечею

С. Збільшене руйнування нормальних еритроцитів

D. Гемоліз еритроцитів

Е. Нестача заліза для синтезу гемоглобіну

70. При оперативному втручанні з приводу пахвинної кили утворюють дуплікатуру передньої стінки пахвинного каналу. Які елементи передньої стінки живота використовуться для утворення цієї дуплікатури?

А. Внутрішня фасція

В. Поверхнева фасція

С. Власна фасція

D. Апоневроз зовнішнього косого м'яза

Е. Очеревина

71. Внаслідок тяжкої хвороби жінка середніх років повної статури різко схудла. Через деякий час з'явилися періодичні болі в поперековій ділянці. Лікар констатував опущення нирок. Послаблення якого з фіксуючих факторів нирок призвело до цього порушення?

A. Arteriae et venae renalis

В. Capsula adiposa

C. Capsula fibrosa

D. Perifoneum

E. Fascia endoabdominalis

72. При розтині жінки 33 років знайдено потовщення стінки шлунка в пілоричному відділі, на розрізі шари стінки розрізняються, з розростанням щільної білуватої тканини в підслизовому шарі і дрібними тяжами її в м'язовому шарі. Рельєф слизової оболонки збережений, складки ригідні, нерухомі. Яка макроскопічна форма пухлини в даному випадку?

А. Кіста

В. Вузол

С. Виразка

D. Інфільтрат

Е. Інфільтративно-виразкова форма

73. На гістологічному зрізі дна шлунка у складі залоз видно порівняно великі клітини з ацидофільною цитоплазмою, при електронній мікроскопії в цих клітинах визначається складна система внутрішньоклітинних канальців. Що продукують ці клітини?

А. Пепсиноген

В. Соляну кислоту

С. Слиз

D. Серотонін

Е. Гастрин

74. Пацієнт звернувся в клініку зі скаргами на загальну слабкість, ниючі болі в животі, поганий апетит, з підозрою на жовтяницю. В сироватці крові знайдено 77,3 мкмоль/л загального білірубіну і 70,76 мкмоль/л кон'югованого білірубіну. Який найбільш вірогідний вид жовтяниці?

А. Цироз печінки

В. Гострий гепатит

С. Механічна жовтяниця

D. Обтураційна жовтяниця

Е. Гемолітична жовтяниця

75. У лабораторії дослідили свіжі кров'янисто-слизові фекалії хворого з дисфункцією кишечника. Під мікроскопом виявили найпростіших, які пересувалися за допомогою випинань ектоплазми і мали всередині захоплені еритроцити. Який вид найпростіших найбільш імовірно виявили?

А. Дизентерійна амеба

В. Балантидій кишковий

С. Лямблія

D. Трихомонада кишкова

Е. Токсоплазма

76. В групі дітей, які їли солодкий соковитий кавун, у двох з'явились ознаки отруєння: різка слабкість, запаморочення, головний біль, блювання, задишка, тахікардія, синюшність губів, вух, кінчиків пальців. Лабораторний аналіз кавуна показав високий вміст нітратів. Який ведучий механізм в патогенезі отруєння тільки у двох дітей?

А. Недостатність каталази

В. Недостатність супероксиддисмутази

С. Блокада цитохромоксидази

D. Недостатність глутатіон-піроксидази

Е. Недостатність мет-НЬ-редуктази

77. У дитини 5 років при гнійному запаленні внутрішнього вуха з'явились симптоми запалення твердої мозкової оболонки. Яким шляхом інфекція могла потрапити до твердої оболонки мозку?

A. Canaliculus cochleae

В. Canaliculus tympanicus

C. Aqueductus vestibuli

D. Fenestrae vestibuli

E. Fenestrae cochleae

78. При огляді 10-річної дитини встановлено малий зріст, непропорційний розвиток тіла, недостатній психічний розвиток. Дефіцит в організмі якого гормону спричинив ці зміни?

А. Тироксину

В. Паратгормону

С. Тиреокальціотоніну

D. Адренокортикотропного гормону

Е. Окситоцину

79. У хворого на висоті чергового нападу гарячки взятий мазок крові, забарвлений за Романовським-Гімзою. При мікроскопії виявлені звивисті бактерії з 8-12 глибокими нерівномірними завитками. Які бактерії були виявлені?

А. Вібріони

В. Лептоспіри

С. Спірили

D. Боррелії

Е. Трепонеми

80. В травматологічний пункт доставлено хворого з пошкодженням м'язів нижніх кінцівок. За рахунок яких клітин можлива репаративна регенерація м'язових волокон і відновлення функції м'язів?

А. Клітин-сателітів

В. Міобластів

С. Міофібробластів

D. Фібробластів

Е. Міоепітеліальних клітин

81. Людина протягом місяця вживає близько 100 г білків на добу, а екскреція загального азоту складає 16 г на добу. Про що може свідчити такий стан азотистого обміну?

А. Посилені процеси розпаду білків

В. Підвищена швидкість синтезу білка

С. Підвищена швидкість синтезу ліпідів

D. Розпад і синтез білка рівноважні

Е. Знижена швидкість синтезу ліпідів

82. В крові пацієнта вміст глюкози натщесерце був 5,65 ммоль/л, через 1 годину після цукрового навантаження становив 8,55 ммоль/л, а через 2 години - 4,95 ммоль/л. Такі показники характерні для:

А. Хворого з тиреотоксикозом

В. Хворого з прихованим цукровим діабетом

С. Хворого з інсулінозалежним цукровим діабетом

D. Хворого з інсулінонезалежним цукровим діабетом

Е. Здорової людини

83. У жінки 63 років є ознаки ревматоїдного артриту. Підвищення рівня якого з перерахованих нижче показників крові буде найбільш значимим для підтвердження діагнозу?

А. Загального холестерину

В. Ліпопротеїдів

С. Кислої фосфатази

D. Сумарних глікозаміногліканів

Е. R-глікозидази

84. У дитини спостерігається нічне свербіння в ділянці анального отвору, розлади травлення різного ступеня важкості. На поверхні фекалій виявляються світлі черви двох розмірних груп: 9-12 мм та 3-4 мм. Який паразит присутній в організмі?

А. Ascaris lumbricoides

В. Strongiloides stercoralis

С. Enterobius vermicularis

D. Tubifex tubifex

E. Lumbricus terrestris

85. У хлопця 5 років, який раніше розвивався без відхилень від вікових норм, почало з'являтися оволосіння за чоловічим типом, збільшуватись маса м'язів. При обстеженні: виражені вторинні чоловічі статеві ознаки, але яєчка за розміром відповідають вікові. Яка найбільш вірогідна причина передчасного статевого дозрівання?

А. Гормонопродукуюча пухлина сім'яників

В. Андрогенопродукуюча пухлина наднирників

С. Збільшення вироблення гонадотропіну

D. Збільшення вироблення адренокортикотропіну

Е. Збільшення вироблення гонадоліберінів

86. У хворого виявлено значне зниження колінного рефлексу. Які сегменти спинного мозку при цьому пошкоджені?

A. І-ІІ крижові

В. III-IV поперекові

С. VII-VIII грудні

D. V-VI шийні

Е. ІХ-Х грудні

87. Хворий 50 років скаржиться на спрагу, п'є багато води, виражена поліурія. Глюкоза крові 4,8 ммоль/л, в сечі глюкози і ацетону немає, сеча безбарвна, питома вага 1,002-1,004. Яка причина поліурії?

А. Інсулінова недостатність

В. Гіпотиреоз

С. Нестача АДГ

D. Альдостеронізм

Е. Тиреотоксикоз

88. Чому пацієнту, який тривалий час отримує з приводу ревматоїдного поліартриту преднізолон, необхідно уникати контактів з інфекційними хворими?

А. Внаслідок розвитку лімфопенії

В. Ризик загострення артриту

С. Ризик тромбоемболічних ускладнень

D. Внаслідок розвитку вторинного імунодефіциту

Е. Внаслідок блокади інтерфероноутво-рення

89. У чоловіка 44 років після опіків виникла спрага. Імпульсація від яких рецепторів найбільшою мірою спричиняє розвиток спраги у зазначених умовах?

А. Дотикових рецепторів

В. Больових рецепторів

С. Температурних рецепторів

D. Осморецепторів

Е. Хеморецепторів

90. Хворий 50 років страждає на шизофренію. Йому була зроблена внутрішньовенна ін'єкція аміназину для усунення марення, після чого хворий швидко знепритомнів. Яка причина цього ускладнення?

А. Дисбаланс окислювальних процесів мозку

В. Порушення коронарного кровообігу

С. Збудження ретикулярної формації

D. Ортостатичний колапс

Е. Гальмування лімбічної системи

91. При дослідженні сироватки крові у хворого виявлено підвищення активності лактатдегідрогенази. Які зміни в організмі на клітинному рівні призводять до подібних порушень?

А. Порушення плазматичних мембран

В. Порушення енергозабезпеченості

С. Порушення міжклітинних взаємодій

D. Пошкодження генетичного апарата

Е. Гнгібування ферментних систем

92. Хворому, що страждає на тромбоемболічну хворобу, призначено штучний антикоагулянт пелентан. Антагоністом якого вітаміна є ця сполука?

А. Вітаміна С

В. Вітаміна Е

С. Вітаміна А

D. Вітаміна D

Е. Вітаміна К

93. У хворого з запальним процесом шкіри і підшкірної клітковини хронічного перебігу виявлено переважання процесів проліферації. Нестача якого гормону може призвести до цього?

А. Інсулін

В. Альдостерон

С. Кортизон

D.CTT

Е. Тироксин

94. Старіння шкіри людини характеризується утворенням зморщок та складок. Зміни у яких структурах головним чином викликають цей стан?

А. Еластичних волокнах

В. Колагенових волокнах

С. Епідермісі

D. Аморфній речовині

Е. Підшкірній жировій клітковині

95. У хворого протягом 30 хвилин не вдається купіювати нітрогліцерином різко виражені загрудинні болі, що іррадіюють в ліву руку. Які зміни в серці розвинулись у хворого?

А. Недостатність мітрального клапана

В. Патологічна гіпертрофія міокарда

С. Різке збільшення коронарного кровотоку

D. Ішемія міокарда

Е. Запалення серцевої сумки

96 При диспансерному обстеженні хлопчику 7 років встановлено діагноз дальтонізму. Батьки здорові, кольоровий зір нормальний. Але у дідуся по материнській лінії така ж аномалія. Який тип успадкування цієї аномалії?

А. Рецесивний, зчеплений зі статтю

В. Домінантний, зчеплений зі статтю

С. Неповне домінування

D. Аутосомно-рецесивний

Е. Аутосомно-домінантний

97. Клітину лабораторної тварини піддали надмірному рентгенівському випромінюванню. В результаті утворились білкові фрагменти в цитоплазмі. Який органоїд клітини візьме участь в їх утилізації?

А. Клітинний центр

В. Комплекс Гольджі

С. Рибосоми

D. Ендоплазматичний ретикулум

Е. Лізосоми

98. У хворого з нагноєною раною стегна збільшились пахові лімфовузли. При біопсії в корковому шарі лімфовузла знайдені збільшені в розмірах фолікули з вираженими реактивними центрами і підвищена кількість плазмоцитів в мозковому шарі. Який процес розвинувся в лімфовузлі?

А. Лімфогранульоматоз

В. Лімфома

С. Плазмоцитома

D. Антигенна стимуляція

Е. Абсцес

99. При аналізі ЕКГ встановлено: ритм синусовий, правильний, інтервал R-R = 0,6 сек, розташування і тривалість інших інтервалів, а також зубців і сегментів не змінена. Яка властивість серця порушена при цьому?

А. Автоматизм

В. Провідність

С. Збудливість

D. Скоротність

Е. Засвоєння ритму

100. У спортсмена на старті перед змаганнями відзначається підвищення артеріального тиску та частоти серцевих скорочень. Впливом яких відділів ЦНС можливо пояснити вказані зміни?

А. Середнього мозку

В. Довгастого мозку

С. Кори великих півкуль

D. Проміжного мозку

Е. Гіпоталамусу

101. Дитина 5 років страждає на деформацію шиї. При клінічному обстеженні виявлено такі симптоми: виражений нахил голови вліво, поворот обличчя вправо, пасивні рухи голови вправо обмежені. Вкорочення якого м'яза має місце?

А. Грудино-ключично-сосцевидного

В. Трапецієвидного

С. Ремінного м'яза голови

D. Грудино-під'язикового

Е. Довгого м'яза голови

102. Під час операції на головному мозку у хворого видалена пухлина щільної консистенції у вигляді вузла, зв'язаного з твердою мозковою оболонкою. Гістологічно вона побудована із ендотеліоподібних клітин, які тісно прилягають одна до одної і утворюють гніздні скупчення, подекуди у вигляді мікроконцентричних структур. Яка пухлина найбільш імовірна в даному випадку?

А. Астроцитома

В. Епендимома

С. Менінгеальна саркома

D. Хоріоїдна папілома

Е. Арахноендотеліома

103. У хворого, який знаходився в реанімаційному відділенні з приводу черепно-мозкової травми, раптово на фоні прояснення свідомості з'явилися корчі, а короткочасна зупинка дихання змінилася поодинокими зітханнями, які мали затихаючий характер. Який тип дихання виник у хворого?

А. Чейна-Стокса

В. Гаспінг

С. Біота

D. Куссмауля

Е. Апнейстичне

104. У хлопчика 12 років із спадковим захворюванням виявлено: екзема, в анамнезі часті запалення легень, кровотечі. В крові зменшений вміст Т-лімфоцитів, знижений рівень Ig M, нормальний рівень Ig А. Яке захворювання у дитини?

А. Синдром Віскотта-Олдріча

В. Синдром Луї-Барра

С. Хвороба Брутона

D. Пізня гіпогаммаглобулінемія

Е. Рання гіпогаммаглобулінемія

105. При мікроскопічному дослідженні серця знайдено постінфарктний транс-муральний кардіосклероз, навкруги якого кардіоміоцити збільшені у розмірах з крупними гіперхромними ядрами, багатими на ДНК. Який процес у карді-оміоцитах найбільш вірогідний?

А. Патологічна регенерація

В. Фізіологічна регенерація

С. Повна репаративна регенерація

D. Регенераційна гіпертрофія

Е. Робоча гіпертрофія

106. У дитини 7 років на шкірі розгинальних поверхонь ліктьових і колінних суглобів з'явились щільні, безболісні вузлики розміром 1-2 мм. В біопта-ті вузликів - велике вогнище фібриноїдного некрозу сполучної тканини з лімфоцитами і макрофагами по периферії. При якому захворюванні спостерігаються такі вузлики?

А. Склеродермія

В. Ревматоїдний артрит

С. Ревматизм

D. Вузликовий периартеріїт

Е. Системний червоний вовчак

107. Юнак 16 років, що страждає на цукровий діабет з 10 років, почув про можливість замінити ін'єкції інсуліну таблетками глібенкламіду. Проте лікар, до якого він звернувся, категорично йому відмовив. Чому глібенкламід не можна призначити у цьому випадку?А. Викликає гіперпродукцію гідрокортизону

В. Швидко деградує в печінці

С. Стимулює альфа-клітини

D. Не стимулює бета-клітини

Е. Посилює виділення адреналіну

108. У хворого з синдромом Іценко-Кушинга в крові збільшено вміст кортизолу. З патологією якої ендокринної залози це пов'язано?

А. Кіркова речовина наднирників

В. Мозкова речовина наднирників

С. Підшлункова залоза

D. Гіпофіз

Е. Щитовидна залоза

109. У чоловіка 60 років після крововиливу в головний мозок настав тривалий сон. Пошкодження яких структур найімовірніше привело до цього стану?

А. Кори великих півкуль

В. Гіпокампу

С. Ядер черепних нервів

D. Ретикулярної формації

Е. Чорної субстанції

110. У дитини 1,5 року з'явилась блювота, пронос, підвищилась температура. На дослідження взято фекалії, які засіяно на середовище Ендо. Через 18 годин на поверхні середовища виросли середніх розмірів, круглі, слабовипуклі червоні колонії з металевим блиском. Яке дослідження дає можливість виявити патогенну кишкову паличку серед цих колоній?

А. Фаготипування мікроорганізмів

В. Забарвлення мазка за методом Грама

С. Визначення рухливості мікроорганізмів

D. Відсів мікроорганізмів на середовище Олькеницького

Е. Реакція аглютинації з сумішшю ОВ-сироваток патогенних серогруп

111. У чоловіка 30 років збільшення вилочкової залози супроводжується розширенням підшкірної венозної сітки і набряком м'яких тканин обличчя, шиї, верхньої половини тулуба і обох верхніх кінцівок. Просвіт якого венозного стовбура звужений?

А. Зовнішньої яремної вени

В. Верхньої порожнистої вени

С. Підключичної вени

D. Внутрішньої яремної вени

Е. Передньої яремної вени

112. В інфекційне відділення лікарні госпіталізовано хворого з діагнозом бактеріальної дизентерії. Лабораторними дослідженнями встановлено, що збудник чутливий до багатьох протимікробних засобів, однак у хворого виявлені явища анемії. Який препарат протипоказаний хворому?

А. Левоміцетин

В. Фталазол

С. Фуразолідон

D. Ентеросептол

Е. Ампіцилін

113. Лабораторія одержала набір для серологічних реакцій, до складу якого входять:

а) еритроцитарний діагностикум (стабілізовані еритроцити з адсорбованим на них Vi-антигеном збудника черевного тифу);

б) буферний ізотонічний розчин;

в) стандартна сироватка з антитілами до Vi-антигена збудника черевного тифу.

Для якої серологічної реакції призначений набір?

А. Гальмування гемаглютинації

В. Зв'язування комплементу

С. Прямої гемаглютинації

D. Пасивної гемаглютинації

Е. Нейтралізації

114. Хворий 40 років госпіталізований зі скаргами на загальну слабість, судоми верхніх і нижніх кінцівок, AT - 160/100 мм рт.ст. Результати дослідження: глюкоза крові - 6,5 ммоль/л, холестерин - 6 ммоль/л, кальцій - 2 ммоль/л, фосфор - 1 ммоль/л, натрій -160 ммоль/л. Сечовиділення - 700 мл за добу. Яка патологія найбільш імовірно буде причиною такого стану?

А. Гіперпаратиреоїдизм

В. Гіпоальдостеронізм

С. Гіперальдостеронізм

D. Тиреотоксикоз

Е. Рахіт

115. Людина довгий час проживала в умовах високогір'я. Які зміни в кровоносній системі будуть у неї?

А. Збільшення кількості гемоглобіну

В. Збільшення кількості лейкоцитів

С. Зниження кількості лейкоцитів

D. Порідшання пульсу

Е. Збільшення діаметра кровоносних судин

116. У пацієнта після пересадки чужорідного ниркового трасплантату розвинулась реакція відторгнення. Які основні ефекторні клітини беруть участь в даній імунологічній реакції?

А. Плазмоцити

В. В-лімфоцити

С. Т-лімфоцити-цитотоксичні

D. Т-лімфоцити-супресори

Е. Т-лімфоцити-хелпери

117. У грудної дитини розвивається жирове переродження печінки, спостерігається галактозурія і аміноацидурія. В крові підвищений загальний білірубін. Яку речовину слід виключити з раціону дитини?

А. Сахароза

В. Жирні кислоти

С. Фенілаланін

D. Холестерин

Е. Молочний цукор

118. У хворої на третій день після тотальної тиреоектомії з'явились судоми. Які лікарські засоби слід призначити цій хворій?

А. Препарат кальцію

В. Протисудомні препарати

С. Седативні препарати

D. Нейролептики

Е. Препарат калію

119. У спортсмена 20 років в результаті постійного фізичного навантаження розвинулась функціональна гіпертрофія лівого шлуночка серця. Який морфо-функціональний процес лежить в її основі?

А. Збільшення розміру клітин і кількості скоротливих органел

В. Збільшення кількості фібробластів С. Збільшення кількості провідних кардіоміоцитів

D. Збільшення кількості сполучної тканини

Е. Збільшення кількості жирової тканини

120. При торакотомії у хворого 55 років в передньому середостінні знайдений пакет лімфовузлів, з яких взято біоптат. Мікроскопічне виявлені інфільтрати, які складаються із лімфоцитів, гістіоцитів, еозинофілів та багатоядерних клітин Березовського-Штернберга, оточені великою кількістю волокнистої сполучної тканини. Яке захворювання найбільш вірогідно відповідає цим даним ?

А. Лімфосаркома

В. Лімфогранульоматоз з пригніченим розвитком лімфоїдної тканини

С. Змішанно-клітинний варіант лімфогранульоматозу

D. Лімфогранульоматоз з переважанням лімфоїдної тканини

Е. Лімфогранульоматоз з переважанням нодулярного склерозу

121. Для проведення мікробіологічних досліджень в бактеріологічній лабораторії, запланованих на наступний день, необхідно підготувати стерильні чашки Петрі, піпетки. Який спосіб стерилізації варто застосувати в даному випадку?

А. Тиндалізацією

В. Текучим паром

С. Сухим жаром

D. Пастеризацією

Е. Кип'ятінням

122. Хворий 20 років звернувся до лікаря зі скаргами на жовтяничність шкірних покривів і склер, тупий біль у ділянці печінки. У нього запідозрено гепатит А. Такий діагноз передбачає зараження хворого через:

А. Укуси комах

В. Інфіковану кров

С. Інфіковану плазму

D. Продукти харчування

Е. Нестерильні шприци

123. В сім'ї батько захворів на трихінельоз. Які профілактичні заходи треба провести, щоб не було зараження інших членів сім'ї?

А. Не проводити ніяких заходів

В. Запобіжне щеплення

С. Госпіталізація хворого

D. Ізоляція хворого

Е. Санобробка приміщення

124. У дитини, що одужує після кору, развинулася пневмонія, викликана умовно-патогенними бактеріями. Яка найбільш вірогідна форма цієї інфекції?

А. Персистуюча інфекція

В. Реінфекція

С. Суперінфекція

D. Вторинна інфекція

Е. Госпітальна інфекція

125. У хворого після гострої посттравматичної крововтрати, яка на добу становила 15% об'єму крові, в крові з'явились поодинокі оксифільні нормобласти. При суправітальному забарвленні виявлено 25% ретикулоцитів. Яка у даного хворого анемія за здатністю до регенерації?

А. Арегенераторна

В. Регенераторна

С. Гіпорегенераторна

D. Гіперрегенераторна

Е. Гіпо- і арегенераторна

126. При трихінельозі навколо личинок трихінели у м'язах утворюється звапніла сполучнотканинна капсула. З функцією яких клітин пухкої сполучної тканини пов'язане її утворення?

А. Фібробластів

В. Макрофагів

С. Плазмоцитів

D. Тучних клітин

Е. Перицитів

127. При окисленні органічних речовин в організмі утворюється ендогенна вода, яка в умовах "сухого голодування" частково компенсує водну недостатність. Яка з наведених речовин при окисленні дає найбільшу кількість води (на одиницю маси цієї речовини)?

А. Жири

В. Білки

С.Глікоген

D. Глюкоза

Е. Гліцерин

128. У пацієнта 40 років з гострим гарячковим захворюванням невиясненої етіології на восьмий день хвороби взяли кров для серологічного дослідження. При проведенні реакції аглютинації з різними діагностикумами встановлено, що реакція Відаля позитивна в розведенні сироватки 1:100, реакція Райта - позитивна в розведенні 1:400. Який діагноз можна встановити на підставі результатів серологічного дослідження?

А. Лептоспіроз

В. Черевний тиф

С. Паратиф А

D. Паратиф В

Е. Бруцельоз

129. Після видалення щитоподібної залози у хворого з'явилися судоми. Яка структура була пошкоджена під час операції?

A. Nervus vagus

В. Nervus laryngeus inferior

C. Arteria laringea inferior

D. Glandulae parathyroideae

E. Truncus sympathicus

130. На шкірообробний завод доставили шкіру тварин з району, де реєструється сибірка. Яка реакція застосовується для виявлення термостабільного антигену збудника сибірки в шкіряній та хутровій сировині?

А. Преципітації

В. Зв'язування комплементу

С. Імунофлюоресценції

D. Аглютинації

Е. Гемаглютинації

131. У людини при пошкодженні одного з відділів ЦНС спостерігались: астенія, м'язова дистонія, порушення рівноваги. Який з відділів ЦНС може бути пошкоджений при цьому?

А. Червоне ядро

В. Чорна субстанція

С. Ретикулярна формація

D. Мозочок

Е. Кохлеарне ядро

132. У хворого напад тахікардії був припинений натисканням на очні яблука (рефлекс Даніні-Ашнера). В основі зниження частоти серцевих скорочень при цьому лежить посилення впливу на синотріальний вузол:

А. Катехоламінів

В. Симпатичних нервів

С. Автономної нервової системи

D. Симпато-адреналової системи

Е. Блукаючих нервів

133. У хворого інфаркт передньої стінки лівого шлуночка. В басейні якої судини виникло порушення кровообігу?

А. Передньої міжшлуночкової гілки лівої вінцевої артерії

В. Передсердно-шлуночкових гілок правої вінцевої артерії

С. Огинаючої гілки лівої вінцевої артерії

D. Задньої міжшлуночкової гілки правої вінцевої артерії

Е. Лівої крайової гілки лівої вінцевої артерії

134. Під час експерименту над бластулою жаби на стадії 16 бластомерів було видалено 1 бластомер. Відокремлена клітина продовжувала нормально розвиватися і започаткувала новий зародок. Яка важлива властивість бластомерів була продемонстрована?

А. Тотипотентність

В. Здатність до ембріональної індукції

С. Здатність до диференціації

D. Утворення полюсів ембріона

Е. Утворення зародкових листків

135. У хворого на ЕКГ виявлено збільшення тривалості інтервалу QT. Це може бути наслідком зменшення у шлуночках швидкості:

А. Реполяризації

В. Деполяризації

С. Деполяризації та реполяризації

D. Скорочення

Е. Розслаблення

136. У хворого на гострий гломерулонефрит виявлено підвищений вміст білків у сечі. З порушенням функції яких структур нефрона пов'язана присутність білків у сечі?

А. Базальної мембрани капілярів клубочка

В. Епітелію парієтального листка капсули клубочка

С. Епітелію тонких канальців

D. Епітелію дистальних канальців

Е. Епітелію петлі Генле

137. При аналізі електрокардіограми у людини виявлено зменшення тривалості інтервалу R-R. Які зміни в роботі серця спостерігаються при цьому?

А. Збільшення сили скорочень

В. Зменшення частоти скорочень

С. Збільшення частоти скорочень

D. Зменшення сили скорочень

Е. Зменшення частоти та сили скорочень

138. У хворого при пальпації щитовидної залози виявлено вузол діаметром 2 см. При гістологічному дослідженні визначаються великі епітеліальні клітини із світлою оксифільною цитоплазмою, які розростаються серед заповнених колоїдом фолікулів. Який найбільш вірогідний діагноз?

А. Солідна аденома

В. Фолікулярна аденома

С. Папілярна аденома

D. Фолікулярний рак

Е. Папілярний рак

139. У хворої з бронхіальною астмою підвищений артеріальний тиск. Який гіпотензивний засіб може спровокувати напад бронхіальної астми?

А. Резерпін

В. Атенолол

С. Каптопріл

D. Анаприлін

Е. Празозин

140. При обстеженні хворого була виявлена недостатня кількість імуноглобулінів. Які з перелічених клітин імунної системи виробляють імуноглобуліни?

А. Т-супресори

В. Т-хелпери

С. Т-кілери

D. Плазматичні

Е. Плазмобласти

141. Жінка 25 років скаржиться на постійний біль в області серця, задишку при русі, загальну слабість. Об'єктивно: шкіра бліда та холодна, акроціаноз. Пульс 96/хв., AT -105/70 мм рт.ст. Межа серця зміщена на 2 см вліво. Перший тон над верхівкою серця послаблений, систолічний шум над верхівкою. Діагностована недостатність мітрального клапана серця. Чим обумовлене порушення кровообігу?

А. Перевантаженням міокарда збільшеним об'ємом крові

В. Перевантаженням міокарда підвищеним опором відтоку крові

С. Пошкодженням міокарда

D. Зниженням об'єму циркулюючої крові

Е. Збільшенням об'єму судинного русла

142. Хвора звернулася в травмпункт з приводу нагноєння різаної рани. Лікар для очищення рани від гнійних виділень промив її 3% розчином перекису водню. При цьому піна не утворилася. З чим зв'язана відсутність дії препарату?

А. Низька концентрація Н2О2

В. Спадкова недостатність каталази

С. Спадкова недостатність фосфатдегідрогенази еритроцитів

D. Неглибока рана

Е. Наявність у рані гнійного вмісту

143. Через місяць після травми центра ока скалкою скла на рогівці утворилось більмо, яке різко знизило зір. За рахунок якого шару рогівки відбулося утворення більма?

А. Власна речовина рогівки

В. Задній епітелій рогівки

С. Передня погранична мембрана

D. Передній епітелій рогівки

Е. Задня погранична мембрана

144. Хворому при подагрі лікар призначив алопуринол. Яка фармакологічна властивість алопуринолу забезпечує терапевтичний ефект в даному випадку?

А. Прискорення синтезу нуклеїнових кислот

В.Збільшенняшвидкостівиведення азотвмісних речовин

С. Прискорення катаболізму піримідинових нуклеотидів

D. Уповільнення реутилізації піримідинових нуклеотидів

Е. Конкурентне інгібування ксантиноксидази

145. В гістологічному препараті стінки очного яблука визначається структура, що складається з ланцюга трьох нейронів. Тіла цих нейронів формують зовнішній, внутрішній ядерний і гангліонарний шар. Яке утворення ока має дану морфологічну будову?

А. Судинна оболонка

В. Райдужка

С. Склера

D. Сітківка

Е. Війкове тіло

146. При визначенні групи крові за системою АВО аглютинацію еритроцитів досліджуваної крові викликали стандартні сироватки І та III груп і не викликала сироватка II групи. Які аглютиногени є в цих еритроцитах?

А. А

В. В

С. С

D. А та В

Е. Аглютиногенів немає

147. Хворому, що знаходиться в гастроентерологічному відділенні, призначена гістамінова проба. З якою метою даному хворому вводять гістамін?

А. Для дослідження секреторної функції шлунка

В. Для стимуляції перетравлювання ліпідів в кишечнику

С. Для активації обмеженого протеолізу в кишечнику

D. Для оцінки активності протеолітичних ферментів підшлункової залози

Е. Для вивчення азотистого балансу

148. При мікроскопічному дослідженні ниркової біопсії виявлено проліферацію епітелію клубочкових капсул і подоцитів з утворенням структур, які нагадують півмісяць; у порожнинах капсул клубочків - накопичення фібрину, вогнищевий некроз капілярних петель, гіаліноз окремих клубочків, набряк строми нирки, дистрофія нефроцитів, гіалінові циліндри у просвітах ка-нальців. Який діагноз найбільш вірогідний?

А. Гострий інтракапілярний гломерулонефрит

В. Гострий екстракапілярний гломерулонефрит

С. Підгострий гломерулонефрит

D. Хронічний мезангіальний гломерулонефрит

Е.-

149. У недоношених новонароджених часто спостерігається синдром дихальної недостатності. Яка найбільш вірогідна причина цього?

А. Ковтання навколоплідних вод

В. Внутрішньоутробна асфіксія

С. Недосконалість нервової регуляції дихального акту

D. Незрілість альвеол легень в зв'язку з нестачею сурфактанта

Е. Внутрішньоутробна гіперкапнія

150. Хворий 45 років госпіталізований в хірургічне відділення зі скаргами на раптовий гострий біль в надчеревній області. Після обстеження встановлено діагноз: перфоративна (проривна) виразка задньої стінки шлунка. Куди вилився вміст шлунка в момент перфорації?

А. В сальникову сумку

В. В печінкову сумку

С. В передшлункову сумку

D. В лівий брижовий синус

Е. В правий брижовий синус

151. До лікаря-уролога звернулася мати хлопчика зі скаргами на відсутність правого яєчка в мошонці. Пальпаторно яєчко визначається біля поверхневого пахового кільця. З якою аномалією розвитку яєчка пов'язане таке явище?

A. Inversio testis

В. Ectopia testis pelvina

C. Ectopia testis perinealis

D. Ectopia testis femoris

E. Retentio testis

152. Хворому виконана трансплантація рогівки. Які особливості будови рогівки дозволяють сподіватися на її приживлення, а не відторгнення?

А. Відсутність кровоносних і типових лімфатичних судин

В. Наявність багатошарового переднього епітелію

С. Надмірна іннервація

D. Наявність сполучної тканини

Е. Наявність одношарового плоского епітелію

153. У хворого важка стафілококова інфекція, що загрожує життю. Який з перерахованих препаратів бажаний в даному випадку?

А. Еритроміцин

В. Ампіцилін

С. Цефазолін

D. Тетрациклін

Е. Бензилпеніцилін

154. Жінка 52 років скаржиться на виражений біль оперізуючого характеру в верхній частині живота. Діагностований гострий панкреатит. Є підозра на ускладнення панкреатиту абсцедуванням. Який тип гарячки найбільш вірогідно підтвердить появу абсцесів?

А. Переміжний

В. Постійний

С. Послаблюючий

D. Гектичний

Е. Атиповий

155. Згідно моделі подвійної спіралі ДНК, запропонованої Уотсоном і Кріком, було встановлено, що один з ланцюгів зберігається при реплікації, а інший синтезується комплементарне першим. Як називається цей способ реплікації?

А. Напівконсервативний

В. Аналогічний

С. Ідентичний

D. Дисперсний

Е. Консервативний

156. В аналізі крові хворого кількість лейкоцитів - 250 * 109/л. Який синдром має місце у даного хворого?

А. Лейкоцитоз

В. Лейкемія

С. Лейкопенія

D. Лейкемоїдна реакція

Е. Гіперлейкоцитоз

157. Хворому 50 років з метою лікування черевного тифу призначений левоміцетин, але на наступний день стан хворого погіршився, температура піднялась до 39,6°С. Чим пояснити погіршення стану хворого?

А. Алергічною реакцією

В. Дією ендотоксинів збудника

С. Нечутливістю збудника до левоміцетину

D. Приєднанням вторинної інфекції

Е. Реінфекцією

158. У хворих на алкоголізм часто спостерігається гіповітаміноз В1 який є наслідком порушення харчування. Симптомами гіповітамінозу В1 є розлади нервової системи, психози, втрата пам'яті. Чому до дефіциту вітаміна В1 особливо чутливі клітини нервової тканини?

А. Посилюється ліполіз жирової тканини

В. Порушується аеробний розпад глюкози

С. Порушується окислення жирних кислот

D. Підвищується інтенсивність гліколізу

Е. Знижується інтенсивність гліколізу

159. У чоловіка 60 років діагностований інсульт в ділянці латеральних ядер гіпоталамусу. Які зміни поведінки слід чекати при цьому?

А. Ненаситність

В. Агресивність

С. Депресія

D. Спрага

Е. Відмова від їжі

160. В лабораторії при мікроскопії харкотиння хворого на пневмонію випадково виявлені личинки. При аналізі крові виявлена еозинофілія. Який гельмінтоз можна передбачити?

А. Трихоцефальоз

В. Ентеробіоз

С. Аскаридоз

D. Парагонімоз

Е. Опісторхоз

161. У чоловіка 42 років, який довгий час знаходився в стані стресу, в сечі значно збільшено вміст 17-кетостероїдів, що в першу чергу свідчить про підвищення секреції:

А. Естрадіолу

В. Кортизолу

С. Адреналіну

D. Норадреналіну

Е. Альдостерону

162. У хворого на сечокам'яну хворобу в крові та сечі знайдено підвищений вміст сечової кислоти. Реакція сечі виявилася різко кислою. Наявність якого типу каменів можна очікувати у цього пацієнта?

А. Кальцієві

В. Оксалатні

С. Фосфатні

D. Уратні

Е. Холестеринові

163. У жінки 42 років виник напад бронхіальної астми, що супроводжувався спазмом бронхів. Підвищення тонусу ядер якого нерва сприяє виникненню бронхоспазму?

А. Язикоглоткового

В. Симпатичного

С. Блукаючого

D. Трійчастого

Е. Лицевого

164. Хвора 25 років звернулася зі скаргами на погіршення зору. При огляді виявлено порушення акомодації, зіниця розширена, не реагує на світло. Функція яких м'язів порушена?

А. М'яз, що звужує і м'яз, що розширює зіницю

В. М'яз, що розширює зіницю, війковий

С. Верхній навскісний, війковий

D. Латеральний прямий, м'яз, що звужує зіницю

Е. М'яз, що звужує зіницю, війковий

165. Жінці, що страждає патологічним ожирінням, для зниження маси тіла провели певну операцію, внаслідок чого вона втратила кілька десятків кілограмів. Що було видалено в ході операції?

А. Частина тонкого кишечника

В. Частина товстого кишечника

С. Жовчний міхур

D. Привушна слинна залоза

Е. Частина шлунка

166. У 40-річної жінки з матковою кровотечею при огляді знайдено різке потовщення слизової оболонки матки, яка мала поліпозні вирости. Мікроскопічно - в ендометрії фаза проліферації, залози видовжені, звивисті, зустрічаються кісти. Який патологічний процес найбільш вірогідний?

А. Ендометрит

В. Поліп шийки матки

С. Ендоцервікоз

D. Залозиста гіперплазія ендометрія

Е. Рак матки

167. При обстеженні хворого з доброякісною пухлиною в легенях поставлено реакцію Васермана, яка виявилась позитивною на три плюси. Яка подальша тактика лікаря?

А. Призначити реакцію мікропреципітації

В. Встановити діагноз: Сифіліс

С. Вдруге призначити реакцію Васермана

D. Вдруге призначити реакцію Вассермана з трепонемним антигеном

Е. Призначити мікроскопію біопсійного матеріалу з пухлини

168. Юнак 15 років пірнав на мілководді. Внаслідок удару голови об дно виникло пошкодження шийного відділу хребта та центрів діафрагмальних нервів з обох боків. Як змінилося при цьому дихання?

А. Не змінилося

В. Стало повільним і глибоким

С. Зупинилося

D. Стало поверхневим і швидким

Е. Стало повільним і поверхневим

169. У пацієнта, носія спадкової серповидної аномалії еритроцитів, захворювання пневмонією супроводжувалося гемолітичною кризою і розвитком анемії. Що є безпосередньою причиною гемолітичної кризи в даному випадку?

А. Гіпероксія

В. Зміна осмолярності крові

С. Гетерозиготність за Hb S

D. Мутація структурного гена

Е. Гіпоксія, викликана пневмонією

170. В гістологічному препараті представлений орган, стінка якого складається з слизової, підслизової, фіброзно-хрящової і адвентиціальної оболонок. Епітелій багаторядний війчастий, м'язова пластинка слизової відсутня, в підслизовій - білково-слизові залози, гіаліновий хрящ утворює незамкнуті кільця. Який орган має дані морфологічні ознаки?

А. Термінальна бронхіола

В. Дрібний бронх

С. Середній бронх

D. Трахея

Е. Гортань

171. При ураженні кишечника злоякісними карциномами близько 60% триптофану окислюється по серотоніновому шляху. В якому вітаміні зростає потреба у хворого на злоякісну карциному шлунка?

А. Піридоксин

В. Фолієва кислота

С. Пантотенова кислота

D. Нікотинова кислота

Е. Рибофлавін

172. Хвора гіпертонічною хворобою тривалий час лікувалась гіпотензивними засобами. В останній час стан хворої погіршився. З'явились нудота, набряк повік і губів, пронос, уртикарні висипи на шкірі, блювання, брадикардія, явища паркінсонізму. Який гіпотензивний препарат викликав ці явища?

А. Анаприлін

В. Клофелін

С. Бензогексоній

D. Резерпін

Е. Дибазол

173. У чоловіка 43 років, померлого при наростаючих явищах серцевої недостатності, на розтині знайдено дифузне розширення дуги аорти. Інтима аорти зморшкувата, горбиста з білуватими рубцевими втягнутостями. Яке запалення лежить в основі даних змін?

А. Специфічне

В. Неспецифічне

С. Геморагічне

D. Серозне

Е. Фібринозне

174. Хворий, який три місяці тому отримав травму голови, чує мову, розуміє її, але не може правильно назвати предмет. В якій частці кори великих півкуль є пошкодження?

А. Верхня скронева

В. Середня лобна

С. Нижня лобна

D. Верхня лобна

Е. Середня скронева

175. Під час операції з приводу лівосторонньої грижі у грижовому мішку була виявлена ущемлена кишка, що має поздовжні м'язові стрічки, відростки серозної оболонки, що містять жирову тканину, без слідів сальника. Який відділ кишечника був ущемлений?

A. Jejunum

В. Rectum

С. Ileum

D. Colon sigmoideum

E. Colon descendens

176. Чоловік 56 років помер раптово. При розтині - розрив аневризми висхідної частини аорти. На інтимі - білуваті горбки з рубцевими втягненнями. Гістологічно у стінці - скупчення лімфоїдних і плазматичних клітин, гігантських клітин типу Пирогова-Лангханса, фібробластів, дрібні вогнища некрозів. Еластичні волокна середньої оболонки зруйновані, в ній визначаються поля сполучної тканини. Яке захворювання спричинило виникнення аневризми?

А. Атеросклероз

В. Сифіліс

С. Системний червоний вовчак

D. Неспецифічний аортоартеріїт

Е. Системний некротизуючий васкуліт

177. Хворий 34 років звернувся зі скаргою з приводу карбункула на обличчі. Під час огляду: нещільний, безболісний набряк підшкірної клітковини, у центрі карбункула чорний струп, по периферії везикульозні висипання навколо карбункула. Бактеріологічне дослідження виявило нерухомі стрептобацили, які здатні утворювати капсули. Які мікроорганізми є збудниками даної хвороби?

A. Staptylococcus aureus

В. Bacillus anthracoides

С. Bacillus antracis

D. Bacillus subtilis

E. Bacillus megaterium

178. Під час операційного втручання сталося рефлекторне посилення впливу блукаючого нерва на серце. Що при цьому може виникнути?

А. Посилення скорочень міокарда

В. Збільшення провідності атріовентрикулярного вузла

С. Збільшення збудливості міокарда

D. Збільшення частоти серцевих скорочень

Е. Зупинка серця

179. У хворого на виразку шлунка, яка розташована у пілоричному відділі на малій кривизні, виникла кровотеча. Яку судину треба перев'язати для зупинки кровотечі?

А. Ліву шлунково-сальникову артерію

В. Ліву шлункову артерію

С. Праву шлункову артерію

D. Печінкову артерію

Е. Праву шлунково-сальникову артерію

180. У хворого вміст Na в плазмі крові 100 ммоль/л. Чим може проявитися такий стан?

А. Аритміями

В. Набряками

С. Дегідратацією організму

D. Зупинкою серця

Е. Тахікардією

181. При дії ультрафіолетових променів у людини темніє шкіра, що є захисною реакцією. Яка захисна речовина синтезується в клітинах шкіри під впливом вказаного фактора?

А. Меланін

В. Триптофан

С. ДНК

D. Артинін

Е. Вітамін D

182. Хворому з прогресуючою м'язовою дистрофією було проведено біохімічне дослідження сечі. Поява якої речовини в великій кількості в сечі може підтвердити захворювання м'язів у даного хворого?

А. Гіпурова кислота

В. Порфирини

С. Сечовина

D. Креатин

Е. Креатинін

183. В тимусі призупинено утворення Т-лімфоцитів-хелперів. Які процеси імуногенезу, що відбуваються в сполучній тканині, будуть порушуватись в першу чергу?

А. Фагоцитування чужорідних тіл

В. Фагоцитування антигенів макрофагами

С. Опсонізації

D. Перетворення В-лімфоцитів у плазматичні клітини

Е. Утворення попередників Т-лімфоцитів

184. До лікаря звернувся пацієнт зі скаргами на болі в суглобах. Візуально помітні набряк та почервоніння в області суглобів. Активність якого фермента необхідно дослідити для постановки діагнозу?

А. Креатинкінази

В. Гіалуронідази

С. Лужної фосфатази

D. Кислої фосфатази

Е. Уреази

185. У чоловіка, що при житті страждав на бронхоектатичну хворобу, пневмосклероз з вираженими явищами кахексії, на розтині серце зменшене в розмірах, стінки стоншені, в'ялої консистенції, на розрізі тканина бурого кольору. Відкладання якого пігменту спостерігається в міокарді?

А. Ліпофусцин

В. Гемосидерин

С. Гематоїдин

D. Цероїд

Е. Ліпохроми

186. У хворого з вираженими периферичними набряками застосування дихлотіазиду не призвело до суттєвого діуретичного ефекту. При біохімічному дослідженні крові було виявлено підвищення вмісту альдостерону. Який засіб є препаратом вибору в даній ситуації?

А. Манніт

В. Діакарб

С. Фуросемід

D. Кислота ектаринова

Е. Спіронолактон

187. При штовханні штанги спортсмен закидає голову назад для максимального підвищення тонусу м'язів-розгиначів верхніх кінцівок. Де розташовані центри рефлексів, які при цьому виникають?

А. В руховій корі

В. В ядрах Дейтерса

С. В базальних гангліях

D. В червоних ядрах

Е. В спинному мозку

188. Лікування дитини, хворої рахітом, за допомогою вітаміна D3 не дало позитивного результату. Яка найбільш вірогідна причина неефективності лікування?

А. Недостатність ліпідів в їжі

В. Порушення гідроксилювання вітаміна D3

С. Порушення включення вітаміна D3 в фермент

D. Підвищене використання вітаміна D3 мікрофлорою кишечника

Е. Порушення транспорту вітаміна D3 білками крові

189. Після крововиливу в області таламуса хворий звернув увагу на те, що не відчуває болю від укола голкою при внутрішньом'язових і внутрішньовенних ін'єкціях. Ураження яких ядер можна припустити?

А. Асоціативних ядер

В. Медіальних колінчатих тіл

С. Латеральних колінчатих тіл

D. Вентролатеральних ядер таламуса

Е. Червоних ядер

190. Хворий скаржиться на втрату апетиту, зниження ваги, слабкість, болі в епігастральній ділянці. На основі клінічних даних і аналізу крові було встановлено діагноз: хвороба Аддісона-Бірмера (перніційна, мегалобластична анемія). Який препарат є основним для лікування даного захворювання?

А. Фолієва кислота

В. Ціанкобаламін

С. Ферковен

D. Коамід

Е. Заліза лактат

191. Після закритої травми грудної клітки пацієнт почав скаржитись на болі у грудній клітці з іррадіацією у спину. При рентгенологічному дослідженні у прямій проекції виявлено зменшення лівого легеневого поля за рахунок розширення вліво серцево-судинної тіні, яка має рівні та чіткі контури. Аневризма якої судини утворилася?

А. Непарної вени

В. Верхньої порожнистої вени

С. Нижньої порожнистої вени

D. Грудної частини аорти

Е. Висхідної частини аорти

192. При різкому підніманні важкого предмета хворий відчув сильний біль у попереку. Він не може зігнутися, при кожному русі біль підсилюється. М'язи напружені. При ушкодженні якого утворення спостерігається така симптоматика?

А. Міжпоперечна зв'язка

В. Міжостиста зв'язка

С. Передня поздовжня зв'язка

D. Остистий відросток хребця

Е. Міжхребцевий диск

193. В гістопрепараті представлений орган з великою кількістю канальців, стінка яких утворена власною оболонкою, що складається з базального, міоїдного і волокнистого шарів. На базальній мембрані розміщуються підтримуючі клітини і сперматогенний епітелій. Який орган представлений в препараті?

А. Сім'яник

В. Придаток сім'яника

С. Сім'явиносна протока

D. Сім'яні пухирці

Е. Передміхурова залоза

194. Жінка 60 років, що страждає артритом тазово-стегнового суглоба, приймала тривало бутадіон. Після аналізу гемограм лікар відмінив цей препарат. Яке ускладнення найбільш імовірно викликав бутадіон з боку крові?

А. Лейкемія

В. Анемія

С. Агранулоцитоз

D. Тромбоемболічний синдром

Е. Порушення гемокоагуляції

195. Який харчовий продукт слід виключити з раціону при лікуванні антидепресантами-інгібіторамиМАО?

А. Тверді сири

В. Яловичину

С. Капусту

D. Картоплю

Е. Яблука

196. В гістопрепараті представлений паренхіматозний орган. Паренхіма складається з великої кількості фолікулів, стінка яких утворена шаром кубічних залозистих клітин. В просвіті фолікулів накопичується колоїд. Між фолікулами знаходяться гемокапіляри і острівці міжфолікулярного епітелію. Якому органу належать дані морфологічні ознаки?

А. Лімфатичному вузлу

В. Яєчнику

С. Щитовидній залозі

D. Гіпофізу

Е. Наднирнику

197. Від хворого з гострою одонтогенною інфекцією виділено культуру кишкової палички, яка чутлива лише до антибіотиків групи аміноглікозидів. Який з перелічених антибіотиків доцільно призначити хворому з лікувальною метою?

А. Кефзол

В. Метицилін

С. Еритроміцин

D. Ністатин

Е. Гентаміцин

198. Пацієнту літнього віку рекомендовано з метою попередження розвитку жирової інфільтрації печінки вживати в їжу сир. Яка незамінна амінокислота, необхідна для синтезу фосфоліпідів, міститься в сирі?

А. Аргінін

В. Метионін

С. Пролін

D. Валін

Е. Лізин

199. Хворий тривало приймав глюкокортикоїди. Після різкої відміни препарата скаржиться на міалгію, підвищену втомлюваність, емоційну нестабільність, головний біль, безсоння, втрату апетиту, нудоту. Розвинувся синдром відміни глюкокортикоїдів. Призначення яких препаратів показане для корекції даного стану?

А. Глюкокортикоїди

В. АКТГ

С. Мінералокортикоїди

D. Адреналін

Е. Кортикостероїди

200. В гінекологічне відділення госпіталізована жінка з діагнозом: гострий правосторонній аднексит. При додатковому обстеженні був поставлений остаточний діагноз: гострий апендицит. Яке положення займає червоподібний відросток?

А. Латеральне

В. Висхідне

С. Низхідне

D. Ретроцекальне

Е. Медіальне

Правильні відповіді на тест“Крок 1 Лікувальна справа" 2000

1.

D

21.

D

41.

C

61.

A

81.

D

101.

A

121.

C

141.

A

161.

B

181.

A

2.

A

22.

A

42.

B

62.

A

82.

E

102.

E

122.

D

142.

B

162.

D

182.

D

3.

E

23.

C

43.

A

63.

C

83.

D

103.

B

123.

A

143.

A

163.

C

183.

D

4.

C

24.

C

44.

B

64.

E

84.

C

104.

A

124.

D

144.

E

164.

E

184.

B

5.

B

25.

E

45.

E

65.

E

85.

B

105.

D

125.

D

145.

D

165.

A

185.

A

6.

A

26.

E

46.

E

66.

A

86.

B

106.

C

126.

A

146.

A

166.

D

186.

E

7.

A

27.

A

47.

A

67.

D

87.

C

107.

D

127.

A

147.

A

167.

D

187.

B

8.

E

28.

B

48.

D

68.

A

88.

D

108.

A

128.

E

148.

C

168.

C

188.

B

9.

B

29.

A

49.

A

69.

A

89.

D

109.

D

129.

D

149.

D

169.

E

189.

D

10.

D

30.

D

50.

E

70.

D

90.

D

110.

E

130.

A

150.

A

170.

D

190.

B

11.

C

31.

C

51.

B

71.

B

91.

A

111.

B

131.

D

151.

E

171.

D

191.

D

12.

A

32.

A

52.

B

72.

D

92.

E

112.

A

132.

E

152.

A

172.

D

192.

E

13.

E

33.

D

53.

B

73.

B

93.

C

113.

D

133.

A

153.

C

173.

A

193.

A

14.

B

34.

A

54.

A

74.

C

94.

A

114.

C

134.

A

154.

D

174.

C

194.

C

15.

D

35.

B

55.

D

75.

A

95.

D

115.

A

135.

C

155.

A

175.

D

195.

A

16.

D

36.

E

56.

D

76.

E

96.

A

116.

C

136.

A

156.

B

176.

B

196.

C

17.

E

37.

A

57.

C

77.

C

97.

E

117.

E

137.

C

157.

B

177.

C

197.

E

18.

C

38.

D

58.

D

78.

A

98.

D

118.

A

138.

A

158.

B

178.

E

198.

B

19.

A

39.

B

59.

C

79.

D

99.

A

119.

A

139.

D

159.

E

179.

C

199.

B

20.

C

40.

A

60.

C

80.

A

100.

C

120.

E

140.

D

160

C

180.

B

200.

C

Екзаменаційний тест 2001 р.

“Крок 1 Лікувальна справа”

1. Хворий на гіпертонічну хворобу, який лікувався гіпотіазидом, скаржиться на загальну слабість, втрату апетиту, серцебиття. Спостерігається гіпотонія м'язів, в'ялі паралічі, послаблення перистальтики кишечника. Що може бути причиною такого стану?

А. Гіпонатріємія

В. Гіперкальціємія

С. Гіпокаліємія

D. Гіперурикемія

Е. Гіперкаліємія

2. При диспансерному обстеженні хлопчику 7 років встановлено діагноз - дальтонізм. Батьки здорові, кольоровий зір нормальний. Але у дідуся по материнській лінії така ж аномалія. Який тип успадкування цієї аномалії?

А. Неповне домінування

В. Рецесивний, зчеплений зі статтю

С. Аутосомно-домінантний

D. Домінантний, зчеплений зі статтю

Е. Аутосомно-рецесивний

3. Методом непрямої калориметрії встановлено, що основний обмін досліджуваного на 40% нижче належного. Порушення діяльності якої ендокринної залози є причиною?

А. Епіфіз

В. Наднирники

С. Щитовидна залоза

D. Підшлункова залоза

Е. Тімус

4. Проводячи хірургічне втручання на жовчному міхурі лікарю необхідно визначити загальну міхурову протоку. В якому анатомічному утворенні очеревини вона знаходиться?

A. Lig. Hepatoduodenale

В. Lig. Duodenorenale

С. Lig. Hepatorenale

D. Lig. Gastrocolicum

E. Lig Hepatogastricum

5. При розтині померлої в комі молодої людини виявлено поширений тромбоемболічний інфаркт лівої півкулі мозку, велика септична селезінка, імунокомплексний гломерулонефрит, виразки в стулках аортального клапана, прикриті поліпоподібними тромбами з колоніями стафілококів. Яке захворювання викликало церебральну тромбоемболію?

А. Септичний бактеріальний ендокардит

В. Гострий ревматичний вальвуліт

С. Септицемія

D. Септикопіємія

Е. Ревматичний тромбендокардит

6. Провідником наукової експедиції по Індії був місцевий житель, який ніколи не розлучався зі своїм улюбленим собакою. Якими інвазійними захворюваннями можуть бути заражені члени експедиції при контакті з цим собакою, якщо він є джерелом інвазії?

А. Дикроцеліозом

В. Фасціольозом

С. Ехінококозом

D. Парагонімозом

Е. Теніозом

7. У хворого камінь загальної жовчної протоки перекрив надходження жовчі до кишечника. Порушення якого процесу травлення при цьому спостерігається?

А. Переварювання білків

B. Всмоктування вуглеводів

С. Переварювання вуглеводів

D. Переварювання жирів

Е. Всмоктування білків

8. Хворого з явищами енцефалопатії госпіталізували до неврологічного стаціонару і виявили кореляцію між наростанням енцефалопатії і речовинами, що надходять із кишківника до загального кровотоку. Які з'єднання, що утворюються в кишківнику, можуть викликати ендотоксемію?

А. Біотин

B. Ацетоацетат

С. Орнітин

D. Індол

Е. Бутират

9. Під час реєстрації ЕКГ хворого із гіперфункцією щитовидної залози зареєстровано збільшення частоти серцевих скорочень. Укорочення якого елементу ЕКГ про це свідчить?

А. Інтервалу R-R

В. Інтервалу P-Q

С. Інтервалу Р-Т

D. Сегменту P-Q

Е. Комплексу QRS

10. Хворий 62 років надійшов до неврологічного відділення з приводу мозкового крововиливу. Стан тяжкий. Спостерігається наростання глибини і частоти дихання, а потім його зменшення до апное, після чого цикл дихальних рухів відновлюється. Який тип дихання виник у хворого?

А. Біота

В. Гаспінг-дихання

С. Апнеїстичне

D. Кусмауля

Е. Чейна-Стокса

11. Внаслідок блокади іонних каналів мембрани клітини її потенціал спокою зменшився з -90 мВ до -70 мВ. Які канали заблоковані?

А. Кальцієві

В. Натрієві

С. Калієві

D. Хлорні

Е. Магнієві

12. У чоловіка 45 років після значного психоемоційного навантаження раптово з'явився стискаючий біль в ділянці серця з іррадіацією в ліву руку, шию, під ліву лопатку. Обличчя стало блідим, вкрилося холодним потом. Нітрогліцерин зняв напад болю. Який процес розвинувся у хворого?

А. Перфорація виразки шлунка

В. Інфаркт міокарда

С. Психогенний шок

D. Стенокардія

Е.Інсульт

13. При профогляді у людини, що не має скарг на стан здоров'я, виявлено лейкоцитоз. Причиною цього може бути те, що кров для аналізу здана після:

А. Вживання алкоголю

В. Відпочинку на курорті

С. Фізичного навантаження

D. Розумової праці

Е. Значного вживання води

14. У препараті червоного кісткового мозку людини визначаються скупчення гігантських клітин, що знаходяться в тісному контакті з синусоїдними капілярами. Які формені елементи крові утворюються з цих клітин?

А. Кров'яні пластинки

В. Лімфоцити

С. Лейкоцити

D. Моноцити

Е. Еритроцити

15. У крові пацієнта вміст глюкози натщесерце був 5,65 ммоль/л, через 1 годину після цукрового навантаження становив 8,55 ммоль/л, а через 2 години - 4,95 ммоль/л. Такі показники характерні для:

А. Хворого з прихованим цукровим діабетом

В. Хворого з тиреотоксикозом

С. Хворого з інсулінозалежним цукровим діабетом

D. Здорової людини

Е. Хворого з інсулінонезалежним цукровим діабетом

16. Жінка 30 років хворіє близько року, коли вперше з'явились болі в ділянці суглобів, їх припухлість, почервоніння шкіри над ними. Попередній діагноз -ревматоїдний артрит. Однією з імовірних причин цього захворювання є зміна в структурі білка сполучної тканини:

А. Муцину

В. Колагену

С. Міозину

D. Тропоніну

Е. Овоальбуміну

17. У результаті безконтрольного прийому вітамінного препарату у дитини виникла анорексія, нудота, блювання, пронос, гіпертермія, з'явились крововиливи на шкірі та слизових, а також явища менінгізму. Який препарат приймала дитина?

А. Рутин

В. Ретінолу ацетат

С. Нікотинамід

D. Соматотропін

Е. Токоферолу ацетат

18. У породілі через 24 години після пологів діагностована емболія легеневої артерії. Виникла задишка з частим глибоким диханням. AT - 90/40 мм рт.ст., частота серцевих скорочень - 99/хв. Газовий аналіз: рН = 7,48; рСО2 = 32 мм рт.ст.; [НСО3] = 20 ммоль/л; [BE] = +1ммоль/л. Який з перерахованих нижче розладів кислотно-лужного стану має місце у цьому випадку?

А. Декомпенсований респіраторний алкалоз

В. Компенсований метаболічний ацидоз

С. Компенсований респіраторний алкоз

D. Декомпенсований метаболічний ацидоз

Е. Респіраторний алкалоз + метаболічний ацидоз

19. Хворому на подагру лікар призначив алопуринол. Яка фармакологічна властивість алопуринолу забезпечує терапевтичний ефект у даному випадку?

А. Прискорення синтезу нуклеїнових кислот

В. Конкурентне інгібування ксантиноксидази

С. Прискорення катаболізму піримідинових нуклеотидів

D. Збільшення швидкості виведення азотвмісних речовин

Е. Уповільнення реутилізації піримідинових нуклеотидів

20. Є одноклітинний організм, що характеризується набором хромосом 2n=8 та розмножується безстатевим шляхом. Генетична різноманітність особи у популяції складе (без урахування мутацій):

А. 256 типів

В. 128 типів

С. 1 тип

D. 8 типів

Е. 32 типи

21. При розтині померлого від поширеного перитоніту в дистальних відділах тонкої кишки виявлено численні виразки овальної форми, які розташовані вздовж кишки. Дно виразок чисте, гладеньке, утворене м'язовою або серозною оболонкою, краї виразок рівні, закруглені. У двох виразках є перфоративні отвори діаметром до 0,5 см. Яке захворювання треба запідозрити?

А. Туберкульоз

В. Холера

С. Черевний тиф

D. Дизентерія

Е. Висипний тиф

22. З метою серологічної діагностики коклюшу поставлена розгорнута реакція з коклюшним та паракоклюшним діагностикумами. На дні пробірок, до яких було внесено діагностикум з Bordetella parapertussis, утворився зернистий осад. Які антитіла виявила ця реакція?

А. Опсоніни

В. Бактеріолізини

С. Аглютиніни

D. Преципітини

Е. Антитоксини

23. У дитини після перенесеної ангіни різко збільшилися лімфатичні вузли: паратрахеальні, біфуркаційні, шийні. При мікроскопії шийного лімфатичного вузла виявлені вогнища некрозу, обмежовані лімфоцитами, епітеліоїдними клітинами та клітинами Пирогова-Лангханса. Яка найбільш імовірна патологія?

А. Риносклерома

В. Туберкульоз

С. Сап

D. Саркоїдоз

Е. Сифіліс

24. У робітника хімчистки виявлена жирова дистрофія печінки. Порушення синтезу якої речовини в печінці може призвести до даної патології?

А. Холієвої кислоти

В. Тристеарину

С. Фосфатидилхоліну

D. Сечовини

Е. Фосфатидної кислоти

25. До інфекційного відділення надійшла дитина 9 років зі скаргами на біль у животі, пронос, анорексію. У випорожненнях - еритроцити. Аналіз показав наявність дизентерійної флори. Який препарат найкраще призначити хворому?

А. Бісептол

В. Фенілсаліцилат

С. Етазол

D. Норсульфазол

Е. Фталазол

26. При розтині жінки 33 років знайдено потовщення стінки шлунка в пілоричному відділі (на розрізі шари стінки розрізняються) з розростанням щільної білуватої тканини в підслизовому шарі і дрібними тяжами її в м'язовому шарі. Рельєф слизової оболонки збережений, складки ригідні, нерухомі. Яка макроскопічна форма пухлини в даному випадку?

А. Вузол

В. Інфільтративно-виразкова форма

С. Виразка

D. Кіста

Е. Інфільтрат

27. У людини зменшений діурез, гіпернатріємія, гіпокаліємія. Гіперсекреція якого гормону може бути причиною таких змін?

А. Адреналін

В. Передсердний натрійуретичний фактор

С. Вазопресин

D. Альдостерон

Е. Паратгормон

28. У хворого виявлено порушення прохідності дихальних шляхів на рівні дрібних і середніх бронхів. Які порушення кислотно-лужної рівноваги можна виявити у крові в даному випадку?

А. Метаболічний ацидоз

В. Респіраторний ацидоз

С. Респіраторний алкалоз

D.-

Е. Метаболічний алкалоз

29. При розтині жінки 40 років, яка страждала ревматоїдним артритом, знайдено збільшену щільну селезінку. На розрізі її тканина коричнево-червоного кольору зі збільшеними фолікулами, що мають вигляд напівпрозорих сірувато-білуватих зерен. Який із перелічених патологічних процесів найбільш імовірний?

А. Сальна селезінка

В. Порфірна селезінка

С. Гіаліноз селезінки

D. Глазурна селезінка

Е. Глазурована селезінка

30. Жінка 58 років. Стан тяжкий, свідомість затьмарена, шкіра суха, очі запалі, ціаноз, запах гнилих яблук з рота. Об'єктивно: глюкоза крові 15,1 ммоль/л, глюкоза сечі 3,5%. Яка найбільш імовірна причина цього стану?

А. Уремічна кома

В. Анафілактичний шок

С. Гіповалемічна кома

D. Гіперглікемічна кома

Е. Гіпоглікемічна кома

31. До травматологічного пункту доставлено хворого з пошкодженням м'язів нижніх кінцівок. За рахунок яких клітин можлива репаративна регенерація м'язових волокон і відновлення функції м'язів?

А. Міофібробластів

В. Фібробластів

С. Міобластів

D. Клітин-сателітів

Е. Міоепітеліальних клітин

32. До лікаря-інфекціоніста на прийом прийшов хворий зі скаргами на пропасницю, яка триває три дні, загальну слабість, безсоння, погіршення апетиту. Лікар запідозрив черевний тиф. Який метод лабораторної діагностики найдоцільніше призначити для підтвердження діагнозу?

А. Виділення мієлокультури

В. Виділення копрокультури

С. Виділення уринокультури

D. Виділення гемокультури

Е. Виділення білікультури

33. У хворого з запальним процесом шкіри і підшкірної клітковини хронічного перебігу виявлено переважання процесів проліферації. Нестача якого гормону може призвести до цього?

А. СТГ

В. Тироксин

С. Кортизон

D. Інсулін

Е. Альдостерон

34. Хворий на цукровий діабет уранці натще одержав призначену дозу інсуліну пролонгованої дії. Пропустив чергове приймання їжі і невдовзі відчув ознаки гіпоглікемії. Застосування глюкози стан не полегшило. Який препарат необхідно ввести для купірування даного стану?

А. Преднізолон

В. Норадреналін

С. Гідрокортизон

D. Адреналін

Е. Триамцинолон

35. При мікроскопічному дослідженні прямої кишки виявлені великі осередки некрозу слизової оболонки, некротичні маси просякнуті фібрином, утворюючи плівку. Слизова та підслизова оболонки по периферії ділянок некрозу повнокровні, набряклі із крововиливами та лейкоцетарною інфільтрацією. Яке захворювання можна передбачити?

А. Амебіаз

В. Сальмонельоз

С. Холера

D. Дизентерія

Е. Черевний тиф

36. Пацієнт звернувся до клініки зі скаргами на загальну слабість, ниючі болі в животі, поганий апетит, з підозрою на жовтяницю. У сироватці крові знайдено 77,3 мкмоль/л загального білірубіну і 70,76 мкмоль/л кон'югованого білірубіну. Який найбільш імовірний вид жовтяниці?

А. Гемолітична жовтяниця

В. Гострий гепатит

С. Механічна жовтяниця

D. Цироз печінки

Е. Обтураційна жовтяниця

37. При гепатиті, інфаркті міокарда в плазмі крові хворих різко зростає активність аланін- і аспартамамінотрасфераз. Які причини зростання активності цих ферментів у крові?

А. Зростання швидкості синтезу амінокислот у тканинах

В. Збільшення швидкості розпаду амінокислот у тканинах

С. Нестача піридоксину

D. Пошкодження мембран клітин і вихід ферментів у кров

Е. Підвищення активності ферментів гормонами

38. Хворому 50 років з метою лікування черевного тифу призначений левоміцетин, але на наступний день стан хворого погіршився, температура підвищилася до 39,6°С. Чим пояснити погіршення стану хворого?

А. Нечутливістю збудника до левоміцетину

В. Реінфекцією

С. Приєднанням вторинної інфекції

D. Алергічною реакцією

Е. Дією ендотоксинів збудника

39. При обстеженні хворого виявлені наступні клінічні прояви: шкірні покриви рожеві, теплі на дотик, сухі, ЧСС- 92/хв.,ЧД - 22/хв., температура тіла- 39,2°С. Яке співвідношення процесів утворення і віддачі тепла в описаному періоді пропасниці?

А. Теплопродукція дорівнює тепловіддачі

В. Теплопродукція перевищує тепловіддачу

С. Посилення теплопродукції без зміни тепловіддачі

D. Теплопродукція нижче за тепловіддачу

Е. Зниження тепловіддачі на фоні незміненої теплопродукції

40. У спортсмена на старті перед змаганнями відзначається підвищення артеріального тиску та частоти серцевих скорочень. Впливом яких відділів ЦНС можна пояснити вказані зміни?

А. Кори великих півкуль

В. Середнього мозку

С. Довгастого мозку

D. Проміжного мозку

Е. Гіпоталамуса

41. В експерименті на кролі через 2 тижні після звуження ниркової артерії виявлено збільшення кількості еритроцитів та гемоглобіну в крові внаслідок стимуляції еритропоезу еритропоетинами. Що посилює утворення еритропоетинів?

А. Гіперкапнія

В. Гіповолюмія

С. Гіперосмія

D. Гіпоосмія

Е. Гіпоксемія

42. Хворий, який три місяці тому отримав травму голови, чує мову, розуміє її, але не може правильно назвати предмет. В якій частці кори великих півкуль є ушкодження?

А. Середня скронева

В. Верхня скронева

С. Верхня лобна

D. Середня лобна

Е. Нижня лобна

43. Під час поділу клітини досліднику вдалося спостерігати фазу, при якій були відсутні мембрана ядра та ядерце, а центріолі знаходились на полюсах клітини. Хромосоми мали вигляд клубка ниток, які вільно розташовані у цитоплазмі. Для якої фази це характерно?

А. Інтерфази

В. Анафази

С. Телофази

D. Профази

Е. Метафази

44. Під час патронажу лікар виявив у дитини симетричну шорсткість щік, діарею, порушення нервової діяльності. Нестача яких харчових факторів є причиною такого стану?

А. Лізин, аскорбінова кислота

В. Нікотинова кислота, триптофан

С. Метіонін, ліпоєва кислота

D. Фенілаланін, пангамова кислота

Е. Треонін, пантотенова кислота

45. Хворий на гіпотонію, відчувши погіршення загального стану, прийняв без контролю лікаря кілька таблеток ефедрину через короткі проміжки часу. Проте деяке покращення спостерігав тільки після вживання першої таблетки. Яким явищем обумовлена така дія?

А. Ідіосинкразія

В. Тахіфілаксія

С. Звикання

D. Кумуляція

Е. Сенсибілізація

46. У чоловіка 60 років діагностований інсульт у ділянці латеральних ядер гіпоталамуса. Які зміни поведінки слід чекати при цьому?

А. Агресивність

В. Спрага

С. Відмова від їжі

D. Ненаситність

Е. Депресія

47. У хворого на ЕКГ виявлено збільшення тривалості інтервалу QT. Це може бути наслідком зменшення у шлуночках швидкості:

А. Реполяризацїї

В. Деполяризації

С. Деполяризації та реполяризації

D. Скорочення

Е. Розслаблення

48. Хворий скаржиться на болі у шлунку, печію. При обстеженні виявлено підвищення кислотності шлункового соку. Що необхідно призначити хворому для нейтралізації кислотності шлункового соку?

А. Альмагель

В. Прозерин

С. Папаверину гідрохлорид

D. Бензогексоній

Е. Атропіну сульфат

49. У хворих на алкоголізм часто спостерігається гіповітаміноз В1 який є наслідком порушення харчування. Симптомами гіповітамінозу В1 є розлади нервової системи, психози, втрата пам'яті. Чому до дефіциту вітаміна В1 особливо чутливі клітини нервової тканини?

А. Порушується аеробний розпад глюкози

В. Знижується інтенсивність гліколізу

С. Порушується окислення жирних кислот

D. Посилюється ліполіз жирової тканини

Е. Підвищується інтенсивність гліколізу

50. Чоловік 37 років надійшов до хірургічного відділення з явищами гострого панкреатиту: блювання, пронос, брадикардія, гіпотензія, слабість, явища зневоднювання організму. Який препарат найбільш доцільно використати в першу чергу?

А. Контрикал

В. Платифілін

С. Етаперазин

D. Но-шпу

Е. Ефедрин

51. У тимусі призупинено утворення Т-лімфоцитів-хелперів. Які процеси імуногенезу, що відбуваються в сполучній тканині, будуть порушуватись у першу чергу?

А. Опсонізації

В. Утворення попередників Т-лімфоцитів

С. Фагоцитування сторонніх тіл

D. Фагоцитування антигенів макрофагами

Е. Перетворення В-лімфоцитів у плазматичні клітини

52. Жінка 52 років, що хворіє на цукровий діабет, скаржиться на головний біль, стомлюваність, безсоння. При огляді встановлений високий рівень AT- 200/100 мм рт.ст. Який препарат найбільш доцільно використати для швидкого зниження артеріального тиску?

А. Анаприлін

В. Каптоприл

С. Папаверин

D. Празозин

Е. Резерпін

53. Хворому з закритим переломом плечової кістки накладена гіпсова пов'язка. Наступного дня з'явилася припухлість, синюшність і похолодання кисті травмованої руки. Про який розлад периферичного кровообігу свідчать ці ознаки?

А. Емболія

В. Венозна гіперемія

С. Тромбоз

D. Ішемія

Е. Артеріальна гіперемія

54. До отоляринголога звернувся хворий, у якого при огляді гортані виявлено неповне змикання голосових зв'язок при фонації. Голосова щілина при цьому набуває форми овалу. Функція якого м'яза гортані порушена у хворого?

А. М. aryttenoideus transversus

В. М. thyroaryttenoideus

С. М. cricoaryttenoideus lateralis

D. M. cricoaryttenoideus posterior

Е. М. vocalis

55. Лікування дитини, хворої на рахіт, за допомогою вітаміну D3 не дало позитивного результату. Яка найбільш імовірна причина неефективності лікування?

А. Порушення гідроксилювання вітаміну D3

В. Порушення транспорту вітаміну D3 білками крові

С. Недостатність ліпідів у їжі

D. Підвищене використання вітаміну D3 мікрофлорою кишечника

Е. Порушення включення вітаміну D3 до ферменту

56. У жінки виявлено пухлину яєчника. Показана операція. Яку зв'язку повинен перерізати хірург, щоб відділити яєчник від матки?

А. Власну зв'язку яєчника

В. Зв'язку, що підвішує яєчник

С. Широку зв'язку матки

D. Бічну пупкову зв'язку

Е. Круглу зв'язку матки

57. У хворого за добу виділяється води з організму менше, ніж надходить. Яке захворювання може призвести до такого стану?

А. Гепатит

В. Інфекційні хвороби

С. Серцева недостатність

D. Панкреатит

Е. Цистит

58. У хворого виникло збільшення щитовидної залози в 2 рази. При пальпації залоза щільна, поверхня нерівномірно горбиста. При гістологічному дослідженні - дифузна інфільтрація тканини залози лімфоцитами, плазматичними клітинами з утворенням фолікулів та посилене розростання сполучної тканини. Який найбільш імовірний діагноз?

А. Дифузний токсичний зоб

В. Зоб Хасімото

С. Зоб Ріделя

D. Ендемічний зоб

Е. Спорадичний зоб

59. У пацієнтки з постійною гіпоглікмією аналіз крові після введення адреналіну суттєво не змінився. Лікар передбачив порушення у печінці. Про зміни якої функції печінки може йтися?

А. Гліколітичної

В. Кетогенної

С. Глікогендепонуючої

D. Холестериноутворюючої

Е. Екскреторної

60. У результаті черепно-мозкової травми у хворого були виявлені такі симптоми: інтенційний тремор, дисметрія, адіадохокінез, дизартрія. Яка структура головного мозку ушкоджена?

А. Блідий шар

В. Рухова кора

С. Стріатум

D. Мозочок

Е. Чорна речовина

61. У хворого М., 45 років, при аналізі ЕКГ встановлено: ритм синусовий, число передсердних комплексів більше числа шлуночкових комплексів; прогресуюче подовження інтервалу P-Q від комплексу до комплексу; випадання окремих шлуночкових комплексів; зубці Р та комплекси QRST без змін. Назвіть тип порушення серцевого ритму.

А. Повна атріовентрикулярна блокада

В. Внутрішньопередсердна блокада

С. Сіноаурікулярна блокада

D. Атріовентрикулярна блокада II ступеня

Е. Атріовентрикулярна блокада І ступеня

62. До лікарні доставлена дворічна дитина із затримкою у розумовому та фізичному розвитку, яка страждає частим блюванням після прийому їжі. У сечі визначена фенілпіровиноградна кислота. Наслідком порушення якого обміну є дана патологія?

А. Фосфорно-кальцієвого обміну

В. Ліпідного обміну

С. Обміну амінокислот

D. Вуглеводного обміну

Е. Водно-сольового обміну

63. У жінки 42 років, яка перенесла операцію на нирці, після наркозу розвинулися явища рекураризації і припинилося дихання. Як міорелаксант був застосований дитилін. Який засіб найбільш доцільно застосовувати для відновлення тонусу м'язів?

А. Стрихніну нітрат

В. Галантоміну гідробромід

С. Кофеїн

D. Плазму крові

Е. Прозерин

64. У клініку доставлена дитина 8 років із різаною раною підошви правої ноги. При хірургічній обробці виявлена глибока рана з розтином сухожилля м'яза на підошвенній поверхні, ближче до латерального краю стопи. У хворого обмежене підняття латерального краю стопи. Функція якого м'яза, ймовірніше за все, порушена?

А. М. extensor digitorum longus

В. М. peroneus longus

С. М. quadriceps femoris

D. M. tibialis anterior

E. M. triceps surae

65. Хворий 40 років госпіталізований зі скаргами на загальну слабість, судоми верхніх і нижніх кінцівок, AT - 160/100 мм рт.ст. Результати дослідження: глюкоза крові - 6,5 ммоль/л, холестерин - 6 ммоль/л, кальцій - 2 ммоль/л, фосфор -1 ммоль/л, натрій - 160 ммоль/л. Сечовиділення - 700 мл за добу. Яка патологія спричинила такий стан?

А. Тиреотоксикоз

В. Гіперальдостеронізм

С. Гіпоальдостеронізм

D. Гіперпаратиреоїдизм

Е. Рахіт

66. Хворому на туберкульоз, в анамнезі якого була відкрита легенева форма захворювання, проведено мікроскопічне дослідження мокротиння з метою виявлення збудника. Який метод забарвлювання доцільно застосувати при цьому?

А. Метод Ціля-Нільсена

В. Метод Буррі-Гінса

С. Метод Нейссера

D. Метод Романовського-Гімза

Е. Метод Грама

67. При томографії у хворого в середостінні знайдені збільшені лімфатичні вузли. При гістологічному дослідженні в лімфовузлі виявлені циркулярні розростання сполучної тканини, яка оточувала гранульомоподібні утворення з лімфоцитів, плазмоцитів і великих двоядерних клітин Березовського-Штернберга. Який найбільш імовірний діагноз?

А. Саркоїдоз

В. Лімфолейкоз

С. Лімфосаркома

D. Туберкульоз

Е. Лімфогранулематоз

68. В експерименті на постсинаптичну мембрану нейрона подіяли речовиною, яка викликала її гіперполяризацію. Проникність для яких іонів на постсинаптичній мембрані збільшилась у даній ситуації?

А. Магнію

В. Калію

С. Кальцію

D. Натрію

Е. Марганцю

69. Вивчення відбитків виступів епідермісу пальців рук (т.з. дактилоскопія) використовується у криміналістиці для ідентифікації особи, а також для діагностики генетичних аномалій, зокрема хвороби Дауна. Який шар шкіри визначає індивідуальність відбитків?

А. Блискучий

В. Базальний

С. Роговий

D. Сосочковий

Е. Сітчастий

70. До інфекційного відділення госпіталізовано хворого зі скаргами на багаторазовий пронос та блювання, біль у м'язах ніг, слабість, запаморочення. Лікар поставив попередній діагноз - "холера". Як необхідно досліджувати матеріал від хворого для експрес- діагнозу?

А. Бактеріологічним методом

В. Пряма і непряма РІФ

С. Біологічним методом

D. Серологічним методом

Е.РА

71. Після дії мутагену в метафазній пластинці людини виявлено на три хромосоми менше за норму. Вказана мутація належить до:

А. Поліплоїдія

В. Політенія

С. Анеуплоїдія

D. Транслокація

Е. Інверсія

72. У лабораторії при мікроскопії харкотиння хворого на пневмонію випадково виявлені личинки. При аналізі крові виявлена еозинофілія. Який гельмінтоз можна передбачити?

А. Аскаридоз

В. Парагонімоз

С. Трихоцефальоз

D. Ентеробіоз

Е. Опісторхоз

73. Клітину лабораторної тварини піддали надмірному рентгенівському опроміненню. У результаті утворились білкові фрагменти в цитоплазмі. Який органоїд клітини візьме участь у їх утилізації?

А. Рибосоми

В. Лізосоми

С. Ендоплазматичний ретикулум

D. Комплекс Гольджі

Е. Клітинний центр

74. У дівчинки 12 років при розтині виявлені численні крововиливи в шкірі (переважно сідниць, нижніх кінцівок), серозних та слизових оболонках, у головному мозку. У наднирниках - вогнищевий некроз та масивні крововиливи, у нирках - некротичний нефроз, гнійний артрит, іридоцикліт, васкуліт. Який найбільш імовірний діагноз?

А. Променева хвороба

В. Висипний тиф

С. Вузликовий періартеріїт

D. Менінгококемія

Е. Системний червоний вовчак

75. На базарі громадянин А. продавав ковбасу під назвою "свиняча домашня".У держсанінспекції виникла підозра фальсифікації ковбаси. За допомогою якої серологічної реакції імунітету можна ідентифікувати харчовий продукт?

А. РНГА

В. Преципітації

С.РЗК

D. Аглютинації

Е. Імунофлюоресценції

76. У роділлі 35 років больовий синдром, пов'язаний із затримкою першого періоду пологів. Який препарат краще за все застосувати для зменшення болю?

А. Парацетамол

В. Морфін

С. Промедол

D. Кодеїн

Е. Анальгін

77. У добовому раціоні дорослої здорової людини повинні бути жири, білки, вуглеводи, вітаміни, мінеральні солі та вода. Укажіть кількість білка, що забезпечує нормальну життєдіяльність організму.

А. 10-20

В. 100-120

С.40-50

D.50-60

Е. 70-80

78. У хворого виник спазм гладенької мускулатури бронхів. Використання яких активаторів для зняття нападу буде фізіологічне обгрунтованим?

А. α- та β-адренорецепторів

В. М-холінорецепторів

С. Н-холінорецепторів

D. β-адренорецепторів

Е. α-адренорецепторів

79. Хворому з великими опіками зробили пересадку донорської шкіри. Але на 8-му добу трансплантат набряк, змінився його колір і на 11 добу почав відторгатися. Які клітини беруть у цьому участь?

А. Еозинофіли

В. Базофіли

С. Еритроцити

D. В-лімфоцити

Е. Т-лімфоцити

80. До клініки доставлений пацієнт з ознаками гострого алкогольного отруєння. Які зміни вуглеводного обміну характерні для цього стану?

А. У м'язах переважає анаеробний разпад глюкози

В. У печінці посилюється глюконеогенез

С. У м'язах посилюється аеробний разпад глюкози

D. У печінці посилюється разпад глікогену

Е. У печінці знижується швидкість глюконеогенезу

81. При дослідженні гнійних виділень з шийки матки бактеріоскопічно виявлено присутність грамнегативних бобоподібних диплококів, які знаходилися як всередині, так і поза лейкоцитами. Назвіть чинника гнійного запалення шийки матки.

A. Calymmatobacterium granulomatis

В. Haemophilus vaginalis

С. Chlamidia trachomatis

D. Neisseria gonorroeae

E. Trichomonas vaginalis

82. У дитини 7 років на шкірі розгинальних поверхонь ліктьових і колінних суглобів з'явились щільні, безболісні вузлики розміром 1-2 мм. В біоптаті вузликів - велике вогнище фібриноїдного некрозу сполучної тканини з лімфоцитами і макрофагами по периферії. При якому захворюванні спостерігаються такі вузлики?

А. Системний червоний вовчак

В. Ревматоїдний артрит

С. Вузликовий періартеріїт

D. Ревматизм

Е. Склеродермія

83. У тварини в експерименті реєструють електричну активність нейронів спірального вузла, що дозволяє аналізувати аферентну імпульсацію від рецепторів:

А. Кортієвого органа

В. Вестибулярних і кортієвого органа

С. Напівкружних каналів

D. Пристінкових

Е. Вестибулярних

84. Хвора 25 років звернулася зі скаргами на погіршення зору. При огляді виявлено порушення акомодації, зіниця розширена, не реагує на світло. Функція яких м'язів порушена?

А. Верхній навскісний, війковий

В. М'яз, що звужує зіницю, війковий

С. Латеральний прямий, м'яз, що звужує зіницю

D. М'яз, що звужує і м'яз, що розширює зіницю

Е. М'яз, що розширює зіницю, війковий

85. До клініки доставили пацієнта 32 років з масивною крововтратою внаслідок автодорожної травми. Пульс 110 уд/хв., частота дихання - 22 за 2 хв., АТ-100/60 мм рт.ст. Яка зміна крові із перелічених буде найбільш характерною через 1 годину після крововтрати?

А. Лейкопенія

В. Гіпохромія еритроцитів

С. Гіпопротеїнемія

D. Еритропенія

Е. Гіповолемія

86. Під час хірургічної операції пацієнту проведено переливання крові. На антигени якого збудника необхідно перевірити цю кров?

А. Ентеровірусів

В. Вірусу гепатиту В

С. Вірусу гепатиту Е

D. Аденовірусів

Е. Вірусу гепатиту А

87. Пацієнт скаржиться на задишку після фізичного навантаження. Об'єктивно: анемія, наявність парапротеїну в зоні гаммаглобулінів. Який показник у сечі необхідно визначити для підтвердження діагнозу мієломи?

А. Білірубін

В. Церулоплазмін

С. Антитрипсин

D. Гемоглобін

Е. Білок Бенс-Джонса

88. У хворого 45 років на правій нозі спостерігається блідість шкіри гомілки та стопи і відзначається відсутність пульсації тильної артерії стопи та задньої великогомілкової артерії. Пульсація стегнової артерії збережена. Ураження якої артерії сталося?

А. Глибокої артерії стегна

В. Низхідної колінної

С. Малогомілкової

D. Зовнішньої клубової

Е. Підколінної

89. Хворий тривало приймав глюкокортикоїди. Після різкої відміни препарату скаржиться на міалгію, підвищену стомлюваність, емоційну нестабільність, головний біль, безсоння, втрату апетиту, нудоту. Розвинувся синдром відміни глюкокортикоїдів. Призначення яких препаратів показане для корекції даного стану?

А. Кортикостероїди

В. Мінералокортикоїди

С. АКТГ

D. Адреналін

Е. Глюкокортикоїди

90. При обстеженні міміки хворого виявлено, що він не може скласти губи трубочкою, не може свистіти, під час сміху кутки рота не піднімаються догори, ротова щілина розтягується в боки (поперечна посмішка). Пацієнт страждає на міопатію - спадкове захворювання із дистрофічним ураженням м'язів. На атрофію якого м'яза вказують дані симптоми?

А. Шийного м'яза

В. Колового м'яза рота

С. Жувального м'яза

D. М'яза сміху

Е. Великого виличного м'яза

91. В експерименті на тварині видалення ділянки кори півкуль мозку усунуло раніше вироблені умовні рефлекси на світлове подразнення. Яку ділянку кори було видалено?

А. Потилична кора

В. Прецентральна звивина

С. Лімбічна кора

D. Постцентральна звивина

Е. Скронева доля

92. У чоловіка 36 років з черепно-мозковою травмою дихання послаблене, пульс ниткоподібний, рефлекси відсутні. Який шлях введення пірацетаму найбільше доцільний у даному випадку?

А. Ректальний

В. Пероральний

С. Інгаляційний

D. Підшкірний

Е. Внутрішньовенний

93. У хворого інфаркт передньої стінки лівого шлуночка. У басейні якої судини виникло порушення кровообігу?

А. Задньої міжшлуночкової гілки правої вінцевої артерії

В. Передньої міжшлуночкової гілки лівої вінцевої артерії

С. Огинаючої гілки лівої вінцевої артерії

D. Лівої крайової гілки лівої вінцевої артерії

Е. Передсердно-шлуночкових гілок правої вінцевої артерії

94. Юнак 16 років, що страждає на цукровий діабет з 10 років, почув про можливість замінити ін'єкції інсуліну таблетками глібенкламіду. Проте лікар, до якого він звернувся, категорично йому відмовив. Чому глібенкламід не можна призначити у цьому випадку?

А. Посилює виділення адреналіну

В. Викликає гіперпродукцію гідрокортизону

С. Не стимулює бета-клітини

D. Стимулює альфа-клітини

Е. Швидко деградує в печінці

95. У хворого на туберкульоз після тривалого лікування з'явились шум та дзвін у вухах, зниження слуху, висипи на шкірі, набряк слизових оболонок і порушилась координація рухів. Після відміни препарату стан хворого значно покращився. Який препарат приймав хворий?

А. Бепаск

В. Ізоніазид

С. Рифампіцин

D. Стрептоміцину сульфат

Е. Етамбутол

96. При обстеженні у хворого виявлено підвищений вміст ліпопротеїнів низької щільності в сироватці крові. Яке захворювання можна очікувати у цього хворого?

А. Ураження нирок

В. Гострий панкреатит

С. Гастрит

D. Атеросклероз

Е. Запалення легень

97. При черговому нападі бронхіальної астми хворий за допомогою інгалятора вдихнув препарат, який у вигляді інгаляцій використовується як бронхолітик, а в ін'єкціях - в акушерській практиці для попередження викиднів. Який препарат був використаний хворим?А. Ефедрин

В. Сальбутамол

С. Фенотерол

D. Адреналін

Е. Еуфілін

98. На електронній мікрофотографії біопсійного матеріалу подано легеню недоношеної дитини. Виявлено злипання стінки альвеол через відсутність сурфактанту. Порушення функції яких клітин стінки альвеоли зумовлюють дану картину?

А. Альвеолоцитів II типу

В. Альвеолярних макрофагів

С. Фібробластів

D. Секреторних клітин

Е. Альвеолоцитів І типу

99. У здорової дорослої людини проводять зондування порожнин серця і великих судин. Де знаходиться зонд, якщо протягом серцевого циклу зареєстровані зміни тиску від 0 до 120 мм рт.ст.?

А. Аорта

В. Легенева артерія

С. Передсердя

D. Правий шлуночок

Е. Лівий шлуночок

100. Пацієнту, що проживає на специфічній геохімічній території, поставлено діагноз - ендемічний зоб. Який вид посттрансляційної модифікації тиреоглобуліну порушений в організмі хворого?

А. Глікозилювання

В. Метилування

С. Фосфорилювання

D. Ацетилування

Е. Йодування

101. У хворого діагностовано гострий інфаркт міокарда, який супроводжується стійкими болями за грудиною. Неефективність попередньо введених препаратів дала підставу лікарю провести нейролептанальгезію. Який нейролептик треба використати?

А. Метаперазин

В. Резерпін

С. Галоперидол

D. Аміназин

Е. Дроперидол

102. Дитина 5 років надійшла до ЛОР-відділення з діагнозом гнійне запалення середнього вуха. Захворювання розпочалося із запалення носоглотки. Через який канал скроневої кістки інфекція потрапила в барабанну порожнину?

А. М'язовотрубний канал

В. Сонний канал

С. Барабанний каналець

D. Каналець барабанної струни

Е. Сонно-барабанні канальці

103. У дівчинки 5 років спостерігається висока температура та біль в горлі. Об'єктивно: набряк м'якого піднебіння, на мигдаликах сірі плівки, які важко відділяються, залишаючи глибокі кровоточиві дефекти тканини. Яке з нижчеперерахованих захворювань найбільш імовірне?

А. Дифтерія зіву

В. Інфекційний мононуклеоз

С. Ангіна Симоновського-Венсана

D. Лакунарна ангіна

Е. Некротична ангіна

104. При дисбактеріозах, що супроводжуються процесом гниття (протея, псевдомонад) і підвищенням рН фекалій, необхідно призначати біологічні препарати, які підкислюють середовище і проявляють антагоністичну дію. Які мікроорганізми для цього придатні?

А. Серація

В. Біфідумбактерії

С. Ентеробактер

D. Клебсієли

Е. Азотобактерії

105. Хворий скаржиться на загальну слабість, головний біль, нудоту, блювання, рідкі випорожнення із домішкою слизу і крові. При мікроскопії дуоденального вмісту і при дослідженні свіжих фекалій виявлено рухомі личинки. Встановіть діагноз.

А. Трихоцефальоз

В. Стронгілоїдоз

С. Дракункульоз

D. Анкілостомоз

Е. Ентеробіоз

106. До клініки потрапив чоловік 54 років зі скаргами на болі в правій підреберній ділянці, блювоту з кров'ю. Об'єктивно: збільшення розмірів печінки, варикозне розширення вен стравоходу і шлунка, кровотеча з них. Порушення функції якої судини, ймовірніше за все, мало місце?

A. Vena hepatica

В. Vena cava superior

C. Vena cava inferior

D. Vena porta

E. Aorta abdominalis

107. Хворий помер від інфаркту міокарда. При патогістологічному дослідженні міокарда виявлені значні контрактурні зміни в кардіоміоцитах. Це зумовлено нагромадженням в кардіоміоцитах іонів:

А. Водню

В. Натрію

С. Кальцію

D. Магнію

Е. Хлору

108. Однорічна дитина відстає в розумовому розвитку від своїх однолітків. Вранці: блювання, судоми, втрата свідомості. В крові - гіпоглікемія натще. З дефектом якого ферменту це пов'язано?

А. Аргінази

В. Лактози

С. Фосфорилази

D. Глікогенсинтази

Е. Сахарози

109.3 метою аналгезії можуть бути використані речовини, що імітують ефекти морфіну, але продукуються в ЦНС. Укажіть їх.

А. β-ендорфін

В. Окситоцин

С. Соматоліберин

D. Вазопресин

Е. Кальцитонін

110. Хворий звернувся зі скаргами на порушення зору, що супроводжується опущенням повіки, неможливість повернути око вгору та до середини. Об'єктивно: око відведене назовні, зіниця розширена та не реагує на світло, хворий не бачить близько. Який нерв ушкоджено?

А. Окоруховий

В. Зоровий

С. Відвідний

D. Трійчастий

Е. Блоковий

111. Через декілька хвилин після введення препарату хворому правцем з'явилася задишка, частий пульс, впав артеріальний тиск. Який препарат міг бути найбільш імовірною причиною ускладнення, що виникло?

А. Сульфаніламід

В. Антибіотик

С. Антитоксична сироватка

D. Анатоксин

Е. Донорський гамаглобулін

112. При розтині померлого, який хворів на атеросклероз, у головному мозку виявлено тромбоз гілки внутрішньої сонної артерії та сірого кольору осередок вологого розм'якшення тканини. Який патологічний процес виявлено в головному мозку?

А. Гематома

В. Ішемічний інфаркт

С. Геморагічна інфільтрація

D. Енцефаліт

Е. Пухлина мозку

113. При моделюванні запалення нижньої кінцівки у тварини підвищилася температура тіла, збільшився вміст антитіл та лейкоцитів у крові. Які речовини зумовили розвиток цих загальних реакцій організму при запаленні?

А. Глюкокортикоїди

В. Мінералокортикоїди

С. Соматомедини

D. Лейкотриєни

Е. Інтерлейкіни

114. Потерпілий доставлений до клініки з відкритим переломом гілки нижньої щелепи, з великою кровотечею в ділянці перелому. Пошкодження якої артерії, імовірніше за все, мало місце?

А. Коміркової нижньої

В. Лицьової

С. Висхідної піднебінної

D. Язикової

Е. Середньої скроневої

115. У потерпілого - різана рана шиї, яка знаходиться вздовж заднього краю m. sternocleidomastoideus, послаблена чутливість шкіри задньої поверхні вушної раковини. Який нерв може бути ушкодженим?

A. nn. auriculares anteriores

В. n. auricularis posterior

С. r. auricularis nervi vagi

D. n. auricularis magnus

E. n. auriculotemporalis

116. У хворого з гострим ринітом виявлена гіперемія і підвищене утворення слизу у носовій порожнині. Активність яких клітин епітелію слизової оболонки підвищена?

А. Мікроворсинчастих

В. Келихоподібних

С. Ендокринних

D. Війчастих

Е. Базальних

117. Хвора звернулася до травмпункту з приводу нагноєння різаної рани. Лікар для очищення рани від гнійних виділень промив її 3% розчином перекису водню. При цьому піна не утворилася. З чим пов'язана відсутність дії препарату?

А. Спадкова недостатність фосфатдегідрогенази еритроцитів

В. Наявність у рані гнійного вмісту

С. Низька концентрація H2O2

D. Неглибока рана

Е. Спадкова недостатність каталази

118. У результаті виснажливої м'язової роботи у робітника значно зменшилась буферна ємність крові. Надходженням якої кислої речовини до крові можна пояснити це явище?

А. 1,3-бісфосфогліцерату

В. Пірувату

С. 3-фосфогліцерату

D. Лактату

Е. α-кетоглутарату

119. Хворий безконтрольно приймав дигоксин, що викликало порушення діяльності серця: зниження скоротності, брадикардацію, шлуночкову екстрасистолію. Яка із перерахованих речовин є препаратом вибору для купірування порушень, що виникли?

А. Етмозин

В. Калію хлорид

С. Верапаміл

D. Анаприлін

Е. Новокаїнамід

120. У дитини, що народилася в пізньому шлюбі, малий зріст, відставання в розумовому розвитку, товстий "географічний язик", вузькі очні щілини, плоске обличчя з широкими вилицями. Яка найбільш імовірна причина розвитку описаного синдрому?

А. Внутрішньоутробна інфекція

В. Спадкова хромосомна патологія

С. Внутрішньоутробний імунний конфлікт

D. Пологова травма

Е. Внутрішньоутробна інтоксикація

121. До приймального відділення доставлений хворий зі скаргами на сухість у роті, світлобоязнь та порушення зору. Шкіра гіперемійована, суха, зіниці розширені, тахікардія. При подальшому обстеженні був встановлений діагноз: отруєння алкалоїдами красавки. Який із лікарських засобів доцільно застосувати?

А. Прозерин

В. Діазепам

С. Дипіроксим

D. Армін

Е. Пілокарпін

122. Хворий 50 років скаржиться на спрагу, п'є багато води, виражена поліурія. Глюкоза крові 4,8 ммоль/л, в сечі глюкози і ацетону немає, сеча безбарвна, питома вага 1,002-1,004. Яка причина поліурії?

А. Альдостеронізм

В. Гіпотиреоз

С. Тиреотоксикоз

D. Інсулінова недостатність

Е. Нестача АДГ

123. У хворого виявлена болючість по ходу великих нервових стволів та підви-щенний вміст пірувату в крові. Нестача якого вітаміну може викликати такі зміни?

А. Пантотенова кислота

В. 82

С.РР

D.BI

Е. Біотин

124. Юнак 16 років скаржиться на свербіж між пальцями рук і на животі, який посилюється вночі. При огляді на шкірі були виявлені тоненькі смужки сірого кольору і дрібненький висип. Який найбільш імовірний збудник цієї хвороби?

A. Ixodes persulcatus

В. Ornithidorus papillipes

С. Dermacentor pictus

D. Sarcoptes scabiei

E. Ixodes ricinus

125. Проводять дослідження на ізольованій збуджуваній клітині. Встановлено, що поріг сили подразнення клітини суттєво зменшився. Що із зазначеного може бути причиною цього?

А. Активація натрієвих каналів мембрани

В. Активація калієвих каналів мембрани

С. Інактивація натрієвих каналів мембрани

D. Інактивація кальцієвих каналів мембрани

Е. Блокада енергоутворення у клітині

126. Пацієнту з гострим інфарктом міокарда внутрішньовенне крапельне введено 1500 мл різних розчинів протягом 8 годин, кисень інтраназально. Смерть настала від набряку легень. Що спричинило набряк легень?

А. Нейрогенна реакція

В. Алергічна реакція

С. Інгаляція кисню

D. Зменшення онкотичного тиску за рахунок гемодилюції

Е. Перевантаження лівого шлуночка об'ємом

127. У хворого через 12 годин після гострого нападу загрудинного болю виявлено різке підвищення активності АсАТ у сироватці крові. Яка патологія з перерахованих буде найбільш імовірною?

А. Колагеноз

В. Нецукровий діабет

С. Інфаркт міокарда

D. Вірусний гепатит

Е. Цукровий діабет

128. Хворому з прогресуючою м'язовою дистрофією було проведено біохімічне дослідження сечі. Поява якої речовини у великій кількості в сечі може підтвердити захворювання м'язів у даного хворого?

А. Гіпурової кислоти

В. Порфиринів

С. Сечовини

D. Креатиніну

Е. Креатину

129. У жінки з первинним гіперпаратиреоїдизмом періодично повторюються напади ниркової коліки. Ультразвукове обстеження показало наявність дрібних каменів у нирках. Яка найбільш імовірна причина утворення цих каменів?

А. Гіперурикемія

В. Гіперкальціємія

С. Гіперфосфатемія

D. Гіперхолестеринемія

Е. Гіперкаліємія

130. У дитини із розумовою відсталістю встановлено зелене забарвлення сечі після додавання 5% розчину FeCl3. Про порушення обміну якої амінокислоти свідчить позитивний результат цієї діагностичної проби?

А. Тирозину

В. Триптофану

С. Фенілаланіну

D. Аргініну

Е. Глутаміну

131. При бактеріологічному дослідженні випорожнень чотиримісячної дитини з симптомами гострої кишкової інфекції на середовищі Ендо виросли у великій кількості червоні колонії. Які це можуть бути мікроорганізми?

А. Стафілококи

В. Сальмонели

С. Ешерихії

D. Шигели

Е. Стрептококи

132. Більша частина учасників експедиції Магеллана в Америку загинула від захворювання на авітаміноз, що проявлялось загальною слабістю, підшкірними крововиливами, випадінням зубів, кровотечею з ясен. Укажіть назву цього авітамінозу.

А. Рахіт

В. Пелагра

С. Анемія Бірмера

D. Скорбут (цинга)

Е. Поліневрит (бері-бері)

133. У 40-річного чоловіка із стенозуючим (без метастазів) раком стравоходу виявляються наступні зміни: атрофія скелетних м'язів, жирової клітковини. Шкіра землисто-коричневого забарвлення, епідерміс потоншаний, серце зменшене за розмірами. Міокард та печінка бурого кольору. Яка найбільш імовірна причина такого стану?

А. Ракова кахексія

В. Аліментарна кахексія

С. Бура атрофія

D. Хвороба Аддісона

Е. Міастенія

134. При дослідженні каріотипу у пацієнта були виявлені два типи клітин в однаковій кількості з хромосомними наборами 46XY і 47XXY. Який найбільш імовірний діагноз?

А. Синдром Патау

В. Нормальний каріотип

С. Моносомія-Х

D. Синдром Клайнфельтера

Е. Синдром Дауна

135. У чоловіка 44 років, мисливця на ондатр, підвищилася температура тіла до 38°С, з'явилися головний біль, набряк повік, гіперемія кон'юнктив, на шкірі в ділянці шиї утворилася неглибока виразка. Вкажіть імовірного збудника захворювання.

A. Leptospira interrogans

В. Bacillus anthracis

С. Brusella suis

D. Yersinia pseudotuberculosis

E. Francisella tularensis

136. Чоловіку 35 років з виразковою хворобою зроблено резекцію антрального відділу шлунка. Секреція якого гастроінтестинального гормону внаслідок операції буде порушена найбільше?

А. Холецистокінін

В. Секретин

С. Гастрин

D. Гістамін

Е. Нейротензин

137. Під час розтину в верхній долі правої легені виявлений великий клиноподібний осередок темночервоної, щільної тканини. При гістологічному дослідженні в ній виявлений некроз стінок альвеол, просвіт альвеол щільно заповнений еритроцитами. Який процес розвинувся в легенях?

А. Карніфікація легень

В. Геморагічний інфаркт легень

С. Ателектаз легень

D. Крововилив у легені

Е. Гангрена легень

138. Хворий 2 роки тому переніс операцію резекції пілоричного відділу шлунка. Спостерігається слабість, періодична поява темних кіл перед очима, задишка. В аналізі крові: НЬ - 70 г/л, ер. - 3,0 * 1012/л, к. п. - 0,7. Які зміни еритроцитів у мазках крові найбільш характерні для даного стану?

А. Мікроцити

В. Макроцити

С. Овалоцити

D. Шизоцити

Е. Мегалоцити

139. При обстеженні чоловіка 45 років, що тривалий час перебував на рослинній дієті, виявлено негативний азотистий баланс. Яка особливість раціону стала причиною цього явища?

А. Недостатня кількість жирів і білків

В. Недостатня кількість жирів

С. Недостатня кількість білків

D. Надмірна кількість вуглеводів

Е. Надмірна кількість води

140. Куди треба провести катетер для забору лімфи з грудної лімфатичної протоки?

А. У нижню порожнисту вену

В. У верхню порожнисту вену

С. У лівий венозний кут

D. У правий венозний кут

Е. У ліву пахову вену

141. У хворого 27 років виявлено патологічні зміни печінки і головного мозку. У плазмі крові виявлено різке зниження, а в сечі підвищення вмісту міді. Поставлено діагноз - хвороба Вільсона. Активність якого ферменту в сироватці крові необхідно дослідити для підтвердження діагнозу?

А. Лейцинамінопептидази

В. Карбоангідрази

С. Церулоплазміну

D. Алкогольдегідрогенази

Е. Ксантиноксидази

142. У собаки в досліді подразнювали на шиї периферичний відрізок блукаючого нерва, при цьому спостерігали такі зміни серцевої діяльності:

А. Збільшення збудливості міокарда

В. Збільшення сили скорочень

С. Збільшення атріовентрикулярного проведення

D. Зменшення частоти скорочень

Е. Збільшення частоти та сили скорочень

143. Хворому виконана трансплантація рогівки. Які особливості будови рогівки дозволяють сподіватися на її приживлення, а не відторгнення?

А. Наявність одношарового плоского епітелію

В. Надмірна іннервація

С. Наявність багатошарового переднього епітелію

D. Відсутність кровоносних і типових лімфатичних судин

Е. Наявність сполучної тканини

144. У хворої на бронхіальну астму вірусне інфікування спровокувало астматичний статус зі смертельним кінцем. При гістологічному дослідженні легень виявлено спазм і набрякання бронхіол, в їх стінках виражена інфільтрація лімфоцитами, еозинофілами та іншими лейкоцитами, а також дегрануляція лаброцитів. Який механізм гіперчутливості лежить в основі описаних змін?

А. Аутоімунний

В. Реагінова реакція

С. Імунокомплексний

D. Запальний

Е. Імунозумовлений клітинний цитоліз

145. У жінки напередодні пологів ШОЕ 40 мм/год. Така величина ШОЕ зумовлена тим, що в крові підвищений вміст:

А. Ліпопротеїнів

В. Фібріногену

С. Еритроцитів

D. Альбумінів

Е. Білків

146. У хворого 42 років при дослідженні периферичної крові виявлено: гемоглобін 80 г/л, еритроцитів 3,2 * 1012/л, лейкоцитів 25 * 109/л; лейкоцитарна формула: базофілів - 5%, еозинофілів - 9%, мієлобластів - 3%, промієлоцитів - 8%; нейтрофілів: мієлоцитів -11%, метаміелоцитів - 22%, паличкоядерних -17%, сегментоядерних - 19%, лімфоцитів -3%, моноцитів - 3%. Яка патологія крові найбільш вірогідна у хворого:

А. Панмієлофтіз

В. Мієлобластний лейкоз

С. Промієлоцитарний лейкоз

D. Хронічний мієлолейкоз

Е. Еритромієлоз

147. У хворого 17 років інтраопераційно на нижній поверхні печінки виявлена пухлина розмірами 4,5x5,0x3,5 см із субсерозною локалізацією, темно-червоного кольору, на розрізі представлена порожнинами зі значним вмістом крові. Поставте попередній діагноз.

А, Кавернозна гемангіома

В. Капілярна гемангіома

С. Лімфангіома

D. Гемангіоперицитома

Е. Гемангіоендотеліома

148. Після травми передньої поверхні верхньої третини передпліччя у хворого затруднення пронації, послаблення долонного згинання кисті і порушення чутливості шкіри 1-3 пальців на долоні. Який нерв ушкоджено?

A. n. ulnaris

В. n. cutaneus antebrachii medialis

С. n. radialis

D. n. medianus

E. n. musculocutaneus

149. У жінки 63 років є ознаки ревматоїдного артриту. Підвищення рівня якого з перерахованих нижче показників крові буде найбільш значущим для підтвердження діагнозу?

А. Кислої фосфатази

В. Сумарних глікозаміногліканів

С. Загального холестерину

D. Ліпопротеїдів

Е. R-глікозидази

150. На аутопсії померлої, яка тривалий час хворіла цистітом і дискінезією сечоводів, виявлено морфологічні ознаки уремії. Нирка була нерівномірно рубцево-зморщена. У просвіті мисок - дрібні уратні камені та пісок. Гістологічне виявлено "щитовидну нирку", вогнища інтерстиційного запалення. Який із нижчеперерахованих діагнозів є найбільш імовірним?

А. Амілоїдно змрщена нирка

В. Атеросклеротично зморщена нирка

С. Хронічний пієлонефрит

D. Первинно зморщена нирка

Е. Гострий пієлонефрит

151. У хворого на глаукому спостерігається підвищення внутрішньоочного тиску при нормальній секреції водянистої вологи циліарним тілом. З ушкодженням яких структур стінки очного яблука пов'язане порушення відтоку рідини з передньої камери?

А. Судинної оболонки

В. Циліарного тіла

С. Заднього епітелію рогівки

D. Війкового м'яза

Е. Венозного синуса

152. Послаблення кровопостачання органа зумовлює розвиток гіпоксії, яка активізує функцію фібробластів. Об'єм яких елементів нарощується в цій ситуації?

А. Лімфатичних судин

В. Нервових елементів

С. Міжклітинної речовини

D. Судин мікроциркуляторного русла

Е. Паренхіматозних елементів органа

153. У хворого на цукровий діабет змінилось значення рН та стало дорівнювати 7,3. Визначення компонентів якої буферної системи використовується для діагностики розладів кислотно-лужної рівноваги?

А. Оксигемоглобінової

В. Білкової

С. Бікарбонатної

D. Фосфатної

Е. Гемоглобінової

154. Згідно з моделлю подвійної спіралі ДНК, запропонованої Уотсоном і Кріком, було встановлено, що один із ланцюгів зберігається при реплікації, а інший синтезується комплементарно першому. Як називається цей спосіб реплікації?

А. Дисперсний

В. Ідентичний

С. Консервативний

D. Аналогічний

Е. Напівконсервативний

155. Підліток 15 років, скаржиться на брак повітря, загальну слабість, серцебиття. ЧСС - 130 уд/хв, AT - 100/60 мм рт. ст. на ЕКГ комплекс QRS нормальної форми та тривалості. Число зубців Р та шлуночкових комплексів однакове, зубець Т злитий із зубцем Е Яка аритмія серця спостерігається у підлітка?

А. Мерехтіння передсердь

В. Тремтіння передсердь

С. Синусова екстрасистолія

D. Синусова тахікардія

Е. Передсердна пароксизмальна тахікардія

156. При мікроскопічному дослідженні пухлини верхньої губи виявлено, що вона побудована з числених щілиноподібних порожнин. Стінки порожнин вистелені сплощеним ендотелієм. Порожнини заповнені рідкою кров'ю і зсідками. Встановіть діагноз.

А. Гемангіоперицитома

В. Капілярна гемангіома

С. Кавернозна гемангіома

D. Венозна гемангіома

Е. Гломус-ангіома

157. Пацієнт скаржиться на запаморочення, відчуття спраги, затруднення ковтання, погане бачення близьких предметів. Об'єктивно: часте дихання, розширені зіниці, загальне збудження, балакучість, однак мова малозрозуміла. AT- 110/70 мм рт. ст., пульс 110/хв. На передозування якого препарату можуть вказувати наведені симптоми?

А. Атропіну

В. Морфіну

С. Кофеїну

D. Аміназину

Е. Ефедріну

158. У дитини, що одужує після кору, развинулася пневмонія, викликана умовно патогенними бактеріями. Яка найбільш імовірна форма цієї інфекції?

А. Реінфекція

В. Госпітальна інфекція

С. Персистуюча інфекція

D. Вторинна інфекція

Е. Суперінфекція

159. У чоловіка, що за життя страждав на бронхоектатичну хворобу, пневмосклероз з вираженими явищами кахексії, на розтині серце зменшене в розмірах, стінки стоншені, в'ялої консистенції, на розрізі тканина бурого кольору. Відкладання якого пігменту спостерігається в міокарді?

А. Гемосидерин

В. Гематоїдин

С. Цероїд

D. Ліпофусцин

Е. Ліпохроми

160. До гастроентерологічного відділення потрапив хворий 57 років з підозрою на синдром Золінгера-Еллісона, про що свідчило різке збільшення рівня гастрину у сироватці крові. Яке порушення секреторної функції шлунка найбільш імовірне?

А. Гіпосекреція гіпоацидна

В. Гіпосекреція гіперацидна

С. Ахілія

D. Гіперсекреція гіпоацидна

Е. Гіперсекреція гіперацидна

161. До інфекційного відділення лікарні госпіталізовано хворого з діагнозом бактеріальної дизентерії. Лабораторними дослідженнями встановлено, що збудник чутливий до багатьох протимікробних засобів, однак у хворого виявлені явища анемії. Який препарат протипоказаний хворому?

А. Ампіцилін

В. Левоміцетин

С. Фталазол

D. Ентеросептол

Е. Фуразолідон

162. Мати виявила у 5-річної доньки на перианальних складках білих "черв'ячків", які викликали у доньки свербіж та неспокій, і доставила їх в лабораторію. Виявлені білі гельмінти 0,5-1 см завдовжки, ниткоподібної форми, з загостреними кінцями, у деяких вони закручені. Який діагноз можна поставити?

А. Опісторхоз

В. Ентеробіоз

С. Аскародоз

D. Теніоз

Е. Дифілоботріоз

163. Який механізм тепловіддачі найбільш ефективно спрацьовує при перебуванні людини в умовах 80% вологості повітря та температурі навколишнього середовища +35°С?

А. Теплопровідність

В. Радіація

С. Конвекція

D.-

Е. Потовиділення

164. У жінки 68 років після інсульту відсутні рухи в верхній та нижній правій кінцівках. Тонус м'язів цих кінцівок і рефлекси в них підвищені. Є патологічні рефлекси. Яка це форма паралічу?

А. Дисоціація

В. Параплегія

С. Тетраплегія

D. Моноплегія

Е. Геміплегія

165. Для гістологічного дослідження доставлена видалена на операції матка. Під слизовою оболонкою визначені численні округлої форми вузли, які чітко відмежовані від оточуючої тканини. Мікроскопічно пухлина побудована з пучків гладкої мускулатури із явищами тканинного атипізму. Який найбільш імовірний діагноз?

А. Рак матки

В. Фіброміоми

С. Лейоміосаркома

D. Лейоміома

Е. Хоріонепітеліома

166. До інфекційної лікарні надійшов хворий із гарячкою, що періодично повторюється. У препараті крові (товста крапля), забарвленому за методом Романовського-Гімза, виявлені спіралеподібні мікроорганізми з гострими кінцями синьо-фіолетового кольору. Який збудник виявлено?

А. Малярії

В. Висипного тифу

С. Поворотного тифу

D. Черевного тифу

Е. Лептоспірозу

167. До травмпункту звернувся чоловік 45 років після побутової травми плеча. Об'єктивно: відсутні функції розгинання, приведення та пронації плеча. Пошкодження якого м'яза викликало такий стан?

А. Підосний м'яз

В. Малий круглий м'яз

С. Підлопатковий м'яз

D. Великий круглий м'яз

Е. Надосний м'яз

168. Людина отримала електротравму. При цьому струм пройшов через серцевий м'яз. Які небезпечні порушення в роботі серця, що вимагають термінового втручання, можуть виникнути у цій ситуаціі?

А. Фібриляція шлуночків

В. Тахікардія

С. Фібриляція передсердь

D. Екстрасистолія

Е. Брадікардія

169. У родині зростає дочка 14 років, у якої спостерігаються деякі відхилення від норми: зріст її нижче, ніж у однолітків, відсутні ознаки статевого дозрівання, шия дуже коротка, плечі широкі. Інтелект у нормі. Яке захворювання можна припустити?

А. Синдром Шерешевського-Тернера

В. Синдром Едвардса

С. Синдром Патау

D. Синдром Клайнфельтера

Е. Синдром Дауна

170. До приймального відділення доставлений чоловік у непритомному стані. Об'єктивно: на зовнішні подразники хворий не реагує, дихання періодичне за типом Чейн-Стокса, зіниці звужені, зіничний рефлекс відсутній. Було встановлено, що дані симптоми зумовлені використанням морфіну. Призначити антидотну терапію.

А. Унітіол

В. Кальцію хлорид

С. Налоксон

D. Апоморфіна гідрохлорид

Е. Протаміну сульфат

171. У хворої 43 років на фоні септичного шоку визначається тромбоцитопенія, зменшення фібриногену, поява в крові продуктів деградації фібрину, петехіальні крововиливи. Яка найбільш вірогідна причина цих змін?

А. Екзогенна інтоксикація

В. Геморагічний діатез

С. ДВС-синдром

D. Аутоімунна тромбоцитопенія

Е. Порушення виробляння тромбоцитів

172. У хворого похилого віку спостерігали збільшення та потовщення пальців кистей, стоп, носа та нижньої щелепи. Із збільшенням виділення якого гормону пов'язані вказані порушення?

А. Тиреотропіну

В. Інсуліну

С. Паратгормону

D. Адренокортикотропіну

Е. Соматотропіну

173. При огляді 3-річної дитини виявлено порушення обвапнення кісток, деформацію хребта і грудної клітини, викривлення нижніх кінцівок. Який препарат є найбільш ефективним у даній ситуації?

А. Токоферол

В. Глюкоза

С. Тіаміну бромід

D. Ергокальциферол

Е. Адіурекрин

174. У гістологічному препараті стінки серця між ендокардом та міокардом виявляються великі клітини зі світлою цитоплазмою та ексцентрично розміщеним ядром. Які це клітини?

А. Пейсмекерові клітини

В. Скоротливі кардіоміоцити

С. Ендокринні клітини

D. Ліпоцити

Е. Клітини Пуркін'є

175. Хворий на ішемічну хворобу серця не повідомив лікаря, що у нього бувають напади бронхоспазму. Лікар призначив препарат, після прийому якого напади стенокардії стали рідшими, але почастішали напади бронхоспазму. Вкажіть, який препарат був призначений.А. Нітросорбід

В. Верапаміл

С. Дилтіазем

D. Анаприлін

Е. Атенолол

176. Хворому проведена субтотальна субфасціальна резекція щитовидної залози. У післяопераційному періоді тривалий час зберігається охриплість голосу. Який нерв ушкоджено в ході операції?

А. Язиковий нерв

В. Верхній гортанний нерв

С. Під'язиковий нерв

D. Поворотний гортанний нерв

Е. Нижньощелепний нерв

177. Хворий 45 років госпіталізований до хірургічного відділення зі скаргами на раптовий гострий біль в надчеревній ділянці. Після обстеження встановлено діагноз: перфоративна (проривна) виразка задньої стінки шлунка. Куди вилився вміст шлунка в момент перфорації?

А. В передшлункову сумку

В. В печінкову сумку

С. В правий брижовий синус

D. В лівий брижовий синус

Е. В сальникову сумку

178. У чоловіка 60 років після крововиливу в головний мозок настав тривалий сон. Пошкодження яких структур найімовірніше призвело до цього стану?

А. Чорної субстанції

В. Ретикулярної формації

С. Ядер черепних нервів

D. Кори великих півкуль

Е. Гіпокампа

179. У культурі тканин ядерним опроміненням пошкоджені ядерця ядер. Відновлення яких органел у цитоплазмі клітин стає проблематичним?

А. Лізосом

В. Комплексу Гольджі

С. Мікротрубочок

D. Ендоплазматичної сітки

Е. Рибосом

180. При відтворенні артеріальної гіпертензії у собаки через 1 місяць товщина стінки лівого шлуночка зросла в 1,7 рази, а об'єм циркулюючої крові не змінився порівняно з вихідними даними. Яка стадія гіпертрофії міокарда спостерігається у тварини?

А. Аварійна

В. Завершеної гіпертрофії

С. Прогресуючого кардіосклерозу

D. Декомпенсації

Е. Початкова

181. У біоптаті бронха хворого, який зловживає палінням, у потовщеній слизовій оболонці виявлено хронічне запалення і трансформацію одношарового війчастого епітелію в багатошаровий плоский епітелій. Який із процесів найбільш імовірний?

А. Лейкоплакія

В. Гіперплазія епітелію

С. Плоскоклітинний рак

D. Метаплазія

Е. Гіпертрофія епітелію

182. У препаратах подані зрізи органів кровотворення та імуногенезу людини, для яких характерна наявність лімфоїдної тканини, що формує різноманітні структури (лімфатичні вузлики, дольки, тяжі). Визначте, в якому з органів відбувається антигеннезалежна проліферація та диференціювання лімфоцитів?

А. Селезінка

В. Гемолімфатичні вузли

С. Тимус

D. Лімфатичні вузли

Е. Мигдалик

183. При огляді зіву у хворого на ангіну визначається гіперемія слизової оболонки піднебіння, мигдалики збільшені в розмірах, червоні, на їх поверхні помітні дрібні біло-жовті осередки. Який клініко-морфологічний варіант ангіни найбільш імовірний у даному випадку?

А. Некротична

В. Лакунарна

С. Гнійна

D. Катаральна

Е. Фібринозна

184. Хлопчик 13 років скаржиться на загальну слабість, запаморочення, втомлюваність. Спостерігається відставання у розумовому розвитку. В крові та сечі висока концентрація валіну, ізолейцину, лейцину. Сеча специфічного запаху. Який найбільш імовірний діагноз?

А. Хвороба кленового сиропу

В. Гістидинемія

С. Хвороба Аддісона

D. Тирозиноз

Е. Базедова хвороба

185. До лікарні потрапив хворий із Східного Сибіру зі скаргою на біль у печінці. У фекаліях знайдені яйця до 30 мкм, які за формою нагадують насіння огірків. Який попередній діагноз можна поставити хворому?

А. Парагонімоз

В. Гіменолепідоз

С. Дикроцеліоз

D. Опісторхоз

Е. Теніаринхоз

186. Після введення жабі стрихніну вона на найменше подразнення відповідає генералізованими судомами. Блокада якої структури ЦНС є причиною цього?

А. Збуджувальних синапсів

В. Холінорецепторів

С. Гальмівних синапсів

D. Клітин Реншоу

Е. Адренорецепторів

187. Піддослідному собаці через зонд у порожнину шлунка ввели 150 мл м'ясного бульйону. Вміст якої з наведених речовин швидко збільшиться у крові тварини?

А. Вазоінтестинальний поліпептид

В. Соматостатин

С. Інсулін

D. Нейротензин

Е. Гастрин

188. Хворий 65 років, що страждав на атеросклероз, госпіталізований до хірургічного відділення з приводу розлитого гнійного перитоніту. Під час операції діагностовано тромбоз брижових артерій. Яка найбільш імовірна причина перитоніту?

А. Ішемія ангіоспастична

В. Стаз

С. Геморагічний інфаркт

D. Ішемічний інфаркт

Е. Ішемія компресійна

189. З випорожнень хворої дитини 6-місячного віку, яка знаходилась на штучному годуванні, виділена культура кишкової палички з антигенною структурою 0-111. Який діагноз можна поставити?

А. Колі-ентерит

В. Харчове отруєння

С. Дизентерієподібне захворювання

D. Холероподібне захворювання

Е. Гастро-ентерит

190. У хворого діагностовано тиреотоксикоз. У крові знайдено антитиреоїдні антитіла. Який тип алергічної реакції за Кумбсом і Джелом спостерігається під час цього захворювання?

А. Стимулюючий

В. Гіперчутливість сповільненого типу

С. Цитотоксичний

D. Імунокомплексний

Е. Анафілактичний

191. У чоловіка 52 років через 3 роки після операції видалення шлунка вміст еритроцитів в крові складає 2,0* 1012/л, НЬ-85г/л, к.п.-1,27. Порушення засвоєння якого вітаміну викликало такі зміни?

А. С

В. ВІ2

С. Р

D. A

Е. B6

192. При пункційній біопсії в трансплантованій нирці виявлена дифузна інфільтрація строми лімфоцитами, плазмоцитами, лімфобластами, плазмобластами, а також некротичний артеріїт. Який патологічний процес розвинувся у трансплантаті?

А. Гломерулонефрит

В. Пієлонефрит

С. Тубулонекроз

D. Ішемічне пошкодження нирки

Е. Імунне відторгнення

193. При визначенні групи крові за системою АВ0 аглютинацію еритроцитів досліджуваної крові викликали стандартні сироватки І та II груп і не викликала III групи. Які аглютиногени містяться в цих еритроцитах?

А.С

В. А

С. А та В

D.B

Е. D та С

194. У людини 70 років швидкість поширення пульсової хвилі виявилась значно вищою, ніж у 25-річної. Причиною цього є зниження:

А. Еластичності судинної стінки

В. Частоти серцевих скорочень

С. Серцевого викиду

D. Швидкості кровотоку

Е. Артеріального тиску

195. У групі дітей, які їли солодкий соковитий кавун, у двох з'явились ознаки отруєння: різка слабість, запаморочення, головний біль, блювання, задишка, тахікардія, синюшність губів, вух, кінчиків пальців. Лабораторний аналіз кавуна показав високий вміст нітратів. Який провідний механізм у патогенезі отруєння тільки у двох дітей?

А. Блокада цитохромоксидази

В. Недостатність глутатіон-піроксидази

С. Недостатність каталази

D. Недостатність мет-НЬ-редуктази

Е. Недостатність супероксиддисмутази

196. У хворого через 9 діб після введення лікувальної сироватки з'явилася кропив'янка, свербіж шкіри, набряк її та слизових оболонок, припухання лімфатичних вузлів. Яке захворювання розвинулось?

А. Феномен Швартцмана

В. Феномен Овері

С. Поліноз

D. Набряк Квінке

Е. Сироваткова хвороба

197. Хворий з інфекційним мононуклеозом протягом 2-х тижнів приймав глюкокортикостероїдні препарати. Настала ремісія, проте в нього виникло загострення хронічного тонзиліту. Результатом якої дії глюкокортикостероїдів є дане ускладнення?

А. Імунодепресивної

В. Антиалергічної

С. Протишокової

D. Протизапальної

Е. Антитоксичної

198. При травмі у пацієнта 44 років виник розрив сухожилків м'язів лівої долоні, поверхневих кровоносних судин. Після оперативного втручання і видалення більшої частини некротично зміненої м'язової тканини кровотік був відновлений. За рахунок яких судин?

А. Аа. digitales palmares communes

В. Arcus palmaris superficialis

C. Aa. perforantes

D. Arcus palmaris profundus

E. Aa. metacarpeae palmares

199. При дослідженні каріотипу 5-річної дівчинки виявлено 46 хромосом. Одна з хромосом 15-ї пари довша від звичайної, тому що до неї приєдналась хромосома з 21-ї пари. Який вид мутації має місце у цієї дівчинки?

А. Інверсія

В. Транслокація

С. Делеція

D. Дуплікація

Е. Нестача

200. У хворого із значними периферійними набряками почергове застосування дихлотиазиду, етакринової кислоти і фурасеміду не сприяло значному діуретичному ефекту. У крові - значне підвищення кількості альдостерону. Вкажіть препарат вибору.

А. Амілорид

В. Спіронолактон

С. Сечовина

D. Маніт

Е. Клопамід

Соседние файлы в предмете [НЕСОРТИРОВАННОЕ]